You are on page 1of 209

PEV110

ADVANCED VERBAL ABILITY - II


Table of content
Units topics Page no

Unit 1 Sentence correction 3-36

Unit 2 Voice and Accent 37-65

Unit 3 Vocabulary and E-mail writing 66-95

Unit 4 Essay writing and Reading comprehension passages 96-136

Unit 5 Narration and Cover letter 137-173

Unit 6 Critical Reasoning 174-207

1
Unit- 1

Sentence correction
Introduction to sentence correction:-

Sentence Correction (SC) is one of question types found in the Verbal section. Sentence Correction tests
your mastery of both grammar and meaning as it applies to conventional written English.

The SC Process:-

Try the process out with the William Pereira example:

Although William Pereira first gained national recognition for his movie set designs, including those
for Reap the Wild Wind and Jane Eyre, future generations remember him as the architect of the
Transamerica Tower, the Malibu campus of Pepperdine University, and the city of Irvine.

(A) including those for Reap the Wild Wind and Jane Eyre, future generations remember
(B) like that for Reap the Wild Wind and Jane Eyre, future generations will remember
(C) like those for Reap the Wild Wind and Jane Eyre, future generations remembered
(D) including that for Reap the Wild Wind and Jane Eyre, future generations remembered
(E) including those for Reap the Wild Wind and Jane Eyre, future generations will remember

1. First Glance
First glance at a problem is, by definition, quick and superficial, but—if one gets good at this step—one can
pick up some very useful clues that will help
2
Read the original sentence with an idea already in mind of what the sentence may be testing.

For SC, pay attention to three issues during your first glance:

Clue Possible Implication

Very long underlines often signal issues with sentence structure,


meaning, modifiers, and parallelism.
1. Is the underline very
long? Veryshort?
Very short underlines (less than five words) may lead you to
compare the answers in full before reading theoriginal sentence.

The nature of the first underlined word (or the word just before)
2. What is the first can give you a clue about one of the issuestested in the sentence.
underlined word? For example, if the word has is the first underlined word, the
What is the word sentence is likely testingeither subject-verb agreement or verb
right before? tense, since hasis a verb.

There will always be at least one difference at the beginning of the


answers (as well as one at the end). It's easy to glance down the
3. What are the
first word or two of each answer, so do so. If the first word
differences amongthe
switches between has and have, for example, then you know the
first word or two of
sentenceis testing singular vs. plural. Now, you can actively look
each answer?
for the relevant subject when you read the original sentence.

Give yourself a few seconds (no more than five!) to glance at a problem, then look away and say out loud
what you noticed in those few seconds.

Afterwards, look at the full problem and remind yourself what it tests. Did your first glance unearth any of
those issues? Examine the first underlined word, the one just before, and the first words of each choice
more carefully, and ask yourself whether there are any clues, or markers, you missed. If so, write them
down on a flash card. Here's an example:

3
Sometimes, there are no good clues at the first glance level, so don't expect that this strategy will always
help you. Still, don't skip this step; good clues exist for more than 50% of problems, so this quick step is
quite valuable.

In the Pereira problem, the underline is relatively short. It begins right after a comma and the first word is
including.

The first word of the five answers will always contain at least one difference, so glance down the first word
of each choice. The “split,” or difference among the answers, here is including vs. like.

The word including is used to introduce examples. The word like is used to indicate a similarity between
two or more things. Keep these in mind as you move to your next step.

1. Read the Sentence for Meaning:-


Your default strategy is to read the entire original sentence, all the way to the period, noting possible
grammar or meaning issues along the way. The non- underlined portion contains very valuable information
that can help you decide how to proceed. Once you're done, decide which issue to tackle first. If you think
you've spotted an error in the original, verify, and then cross off any answer that repeat that same error.

Read only the original sentence (not the answers), then look away and try to articulate aloud, in your own
words, what the sentence is trying to say. (You don't need to limit your rephrase to a single sentence.)

Do actually talk out loud. You'll be able to hear the conviction in your own voice when you know what the
sentence is trying to say and you'll also know if you don't really know what the sentence means.

What does the William Pereira sentence say?

The sentence begins with a contrast word (although), so make sure the rest of the sentence does convey a
contrast. Although he gained recognition for one thing, he was remembered for other, quite different
things. That basic meaning does make sense.

2. Find a Starting Point:-


Most SC problems test multiple issues and those issues can appear anywhere in the sentence. Where do
you start?

Initially, you're likely to have one of two starting points:

4
1. You spot an error (or suspected error) in the original sentence.
2. You notice splits, or differences, in the answers.

If you think you've found an issue in the original sentence, immediately look through the answers to make
sure you're offered at least one split for that issue. If all five are identical, then you haven't actually found
an error. If you are offered splits, go ahead and tackle that issue.

You might get to the end of the original sentence without spotting an error. In this case, start comparing
the answers to find splits. If you don't know how to decide about a particular split, ignore it and find
another.
The first two steps—first glance and read for meaning—will usually help you to find your first starting
point.

For instance, in the Pereira problem, the first glance showed a split between including and like, so as you
read, ask yourself:

why does the sentence mention these two films? They represent examples of Pereira's movie set designs,
and examples should be introduced using the word including, not the word like.

4 Eliminate all incorrect choices:-

Scan down the options. Answers (B) and (C) both use like; eliminate them.

There are still three choices left, so find another starting point and repeat steps 3 and 4. After a repetition
or two, you'll either get down to one answer or get stuck. Either way, pick an answer and move on to the
next problem.

If you spot a difference but don't know how to deal with it, ignore that difference and look for some other
difference instead.

Now, where are you going to find these new starting points? You have two main options:

1. Tackle errors that you spotted in the original sentence.

2. Compare the remaining answer choices vertically, looking for differences, or splits. If you know how
to tackle a particular split, do so!

5
In the Pereira example, you might note that the answers split on that vs. those. What is at the heart of that
difference?

The two words are pronouns, but one is singular and one is plural. The pronoun is intended to refer back to

the plural word designs, so the singular that is incorrect. Eliminate answer (D).

Now, compare the last two answers, (A) and (E). The only difference is at the end: remember vs. will
remember. Pereira first gained recognition for one thing, but the author postulates that future generations
are going to remember him for something else. The future tense, will remember, fits that meaning.

Eliminate (A) and pick (E).

There are various types of Sentence errors –

1. subject-verb agreement
2. pronoun agreement
3. modifiers
4. parallelism
5. comparisons
6. Verb tenses
7. errors of participles
8. redundancy

First error – Subject verb agreement


Every sentence must have a subject and a verb. The subject is the noun that performs the action expressed
by the verb:

6
The subject, the dog, is performing the action of running. Moreover, the subject and the verb must agree
in number: in this case, both dog and runs are singular.

Subject and Verb Must Both Exist

If a sentence is missing the subject or the verb, the sentence is a fragment; in other words, it is not a
complete sentence, the correct answer must be a complete sentence, or independent clause.

The error might be disguise by dropping the verb:

Wrong: The cat sitting by the stairs.

Wait a minute, what about sitting? Sitting certainly looks like a verb. It is not, however, a working verb, a
verb that can run a sentence by itself. Here's an example of a working verb:

Right: The cat sitting by the stairs WATCHED the mouse.

In this sentence, the word watched is a working verb. Here's another example of a working verb:

Right: The cat WAS SITTING by the stairs.

In this sentence, the words was sitting make up the full verb form. The word sitting is called a present
participle and you'll see more of these “-ing” words throughout this book. For now, just remember that an
-ing word by itself is never a working verb: The cat sitting by the stairs is not a sentence.

Subject and Verb Must Agree in Number

A singular subject requires a singular verb form:

The dog runs out of the house.

A plural subject requires a plural verb form:

The dogs run out of the house.

However , the subject sometimes is hidden, making you unsure whether the subject is singular or plural. If
you do not know the number of the subject, then you will not be able to select the verb form that agrees
with it. Consider this example:

The discovery of new medicines (was/were) vital to the company's growth.

7
What is the subject, discovery or new medicines? If you ask yourself, “What is/are vital to the company's
growth?” you may be able to talk yourself into either choice. It makes as much sense to say the discovery
was vital as it does to say the new medicines were vital.

In this case, The discovery…was is the correct subject–verb pair because the noun medicines is part of the
prepositional phrase of new medicines. A noun in a prepositional phrase cannot be the subject of the
sentence.

Are these sentences both correct?

Lin and Guy drive to work.

Lin, as well as Guy, drive to work every day.

The first sentence is a correct example of a compound subject: Lin and Guy together function as the
subject of the sentence. Compound subjects are always plural because at least two nouns function as the
subject.

A compound subject must be connected by the word and, but the second sentence uses the modifier as
well as Guy. Only Lin qualifies as the subject, so the sentence is incorrect. It should read:

Right:

LIN, as well as Guy, drives to work every day.

Subject Modifier Verb

A sentence can also contain a compound verb (two or more verbs that all point to the same subject). For
example:

That last sentence contains both a compound subject and a compound verb. If the writer inserts enough
distance between the two portions of a compound subject or verb, it could be easy to make a mistake.

Second error- Pronoun Agreement


A pronoun is a word that takes the place of a noun, so that you do not have to repeat that noun
elsewhere in the sentence. For example:

8
GASOLINE has become so expensive that it now consumes as much as 16% of personal income
in some rural areas.

In the sentence above, the pronoun it takes the place of the noun gasoline. In other words, it refers to
gasoline. The noun gasoline is known as the antecedent of it.

Most common pronoun makers

The Antecedent Must Exist and Be Sensible

If the answer choices do offer different pronouns, then find the antecedent:

The park rangers discussed measures to prevent severe wildfires, which would be devastating to it.

What noun does it refer to? Logically, it should refer to the park. However, in this sentence, park is not a
noun. Rather, park is acting as an adjective in the phrase the park rangers. As a result, park cannot be the
antecedent of it.

Moreover, there is no other possible antecedent in the sentence, so this answer choice would be incorrect
on a Sentence Correction question.

Watch out for adjectives! They cannot be antecedents of pronouns. The antecedent to which you want
to refer must actually exist in the sentence as a noun. Be careful not to gloss over the meaning. For
example:

Although the term “supercomputer” may sound fanciful or exaggerated, it is


simply an extremely fast mainframe that can execute trillions of calculations
every second.

The antecedent appears to be the term “supercomputer.” Look what happens when the pronoun is
replaced with this noun:

… the TERM “supercomputer” is simply an extremely fast mainframe …

The term is not a mainframe; rather, the term refers to a mainframe. Therefore, you must change the verb
or make some other edit:

9
Although the TERM “supercomputer” may sound fanciful or exaggerated, it simply REFERS TO an
extremely fast mainframe that can execute trillions of calculations every second.

The Antecedent and Pronoun Must Agree in Number

If the answers switch between singular and plural pronouns, then check the antecedent to see whether it
is singular or plural. If the answers switch between singular and plural nouns, check either for pronouns or
for verbs to help you decide whether the noun should be singular or plural. Consider this example:

Confronted by radical changes in production and distribution, modern Hollywood


studios are attempting various experiments in an effort to retain its status as the
primary arbiter of movie consumption.

The antecedent of its is intended to be studios. However, its is singular, while studios is plural. Either the
noun or the pronoun has to change (depending upon which portion is underlined in the problem):

Right: Confronted by radical changes in production and distribution, modern


Hollywood STUDIOS are attempting various experiments in an effort to retain their
status as the primary arbiters of movie consumption.

Right: Confronted by radical changes in production and distribution, the modern


Hollywood STUDIO is attempting various experiments in an effort to retain its status
as the primary arbiter of movie consumption.

The Deadly Five: It, Its, They, Them, Their

The most common pronoun mistakes involve the singular it and its, and the plural they,
them, and their. Whenever you see one of these five pronouns, check the answers; if
differences exist, find the antecedent and check its viability.

Be careful with their, which is often used in everyday speech to refer to singular subjects:

Wrong: Whenever a STUDENT calls, take down their information.

Right: Whenever a STUDENT calls, take down his or her information.

Right: Whenever STUDENTS call, take down their information.

Some Ambiguity Is Acceptable

In theory, every pronoun in a well-written sentence should clearly refer to one antecedent. If a sentence
uses the same pronoun multiple times, every instance must refer to the same antecedent. If the first it
refers to one noun and the second it refers to another, unacceptable confusion reigns.
10
It is also preferable to have pronouns of the same class refer to the same noun. It and its are one class, and
they, them, and their are another class. This is a preference, however, not a rule. For example:

Researchers claim to have developed new “nano-papers” incorporating tiny cellulose fibers, which
THEY allege give THEM the strength of cast iron.

What nouns do they and them refer to? Logically, they refers to researchers (who claim something) and
them refers to new “nano-papers.” If another grammatically correct option exists without this mild
ambiguity, choose the other option. If, however, the other four choices all contain errors, then this choice
would be correct.

An answer choice could avoid potential ambiguity by not using a pronoun in the first place:

Right: Researchers claim to have developed new “nano-papers” incorporating tiny cellulose fibers,
which give THESE MATERIALS the strength of cast iron, according to the researchers.

If you spot a split between a pronoun and a regular noun, chances are good that the correct answer will
use the regular noun, since that usage will prevent any possible misreading of a pronoun.

Third error- Modifier


A modifier describes or provides extra information about something else in the sentence.

Tired out from playing basketball, CHARLES decided to take a nap.

The modifier Tired out from playing basketball describes the noun Charles. It provides additional context as
to why Charles decided to take a nap. Many modifiers answer the questions who, what, when, where, or
why. Incorrectly used modifiers can lead to ambiguity or illogical meaning.

Common errors when using modifiers

Because modifiers can exist in the form of groups of words—phrases and clauses—it is easy to misplace
them in writing. Just like adjectives and adverbs need to be in the correct location with respect to the
nouns and verbs they modify, adjectival and adverbial clauses, prepositional phrases, and other modifiers
need to be placed such that they correctly modify their intended words/groups of words.

1. Dangling modifier

After watching the movie, the book still seems better.

 The adverbial phrase “after watching the movie” incorrectly modifies “the book.” This doesn’t
make sense since a book cannot watch a movie. The modifier is dangling because the word it is
supposed to modify—the name of or pronoun for the person who watched the movie—is not there.
 A correction: After watching the movie, he still thinks the book is better.

2. Misplaced modifier

Joe thought about throwing a party for his dad while he walked in the park.
11
 As it is, this sentence says that Joe thought about throwing the party while his dad walked in the
park. That doesn’t make sense—his dad wouldn’t be there—and it is unclear.
 The modifier “while he walked in the park” is misplaced. Is it referring to Joe or his dad?
 A correction: While he walked in the park, Joe thought about throwing a party for his dad

3. Ambiguous (squinting)
Explaining your premises clearly enhances your argument.
 In this sentence, “clearly” could modify “explaining your premises,” as in explaining them clearly, or
it could modify “enhances your argument,” as in clearly enhancing it.
 A correction: Explaining your premises will clearly enhance your argument.

Following are four rules of placement that can correct the majority of modifier problems:

1. Simple adjectives precede.

2. Adjective phrases and clauses follow.

3. Adverbs move around.

4. Limiters precede.

1. Simple adjectives precede.

In the following, aromatic modifies tea, fluffy modifies socks, and cold modifies feet:

Mai set a cup of aromatic tea to steep on the counter while she pulled fluffy socks over her cold feet.

In general, an adjective modifier precedes the noun it modifies. In other words, it is aromatic tea, not "tea
aromatic." Nor is it "...cup of tea to steep aromatic on the counter," or "pulled socks over her fluffy cold
feet." If you are a native speaker of English, you probably knew this instinctively.

2. Adjective phrases and clauses follow.

We tend to associate phrase modifiers with the nearest preceding noun. The following example has a
comic effect because we read the modifier, with a deep tan, as belonging to the nearest noun:

Incorrect:

The lifeguard dove into the surf with the deep tan. (The surf with the deep tan?)

Revised:

The lifeguard with the deep tan dove into the surf.

Adjective phrases like this not only follow the nouns they describe, they must follow very closely to make
the proper sense.

12
Treat adjective clauses similarly. These are word groups that contain both a subject and verb, but are not
complete because they also contain a dependent-making word (that or which, for example). They explain
or otherwise expand on information in the sentence and, like adjective phrases, should immediately follow
their headwords:

Incorrect:

Her sorority sponsored a blood drive to assist the disaster relief effort that they had spent almost six months
planning. (Did the sorority plan the entire disaster relief effort? Or just the blood drive?)

Revised:

Her sorority sponsored a blood drive that they had spent almost six months planning to assist the disaster
relief effort.

3. Adverbs move around.

Adverbs modify verbs, adjectives, and other adverbs. In English, adverbs are allowed to move around quite
a bit in a sentence. In the following example, the adverb quickly modifies the verb calculated, or the verb
graphed, or both - but without causing a great deal of confusion no matter where it is placed.

Quickly she calculated the slope and graphed the result.

She quickly calculated the slope and graphed the result.

She calculated the slope and quickly graphed the result.

She calculated the slope and graphed the result quickly.

However, misplaced adverbs can cause ambiguity. If this happens, simply move the adverb to place it next
to the headword it is intended to modify.

Incorrect:

Mikail followed the instructions for setting up the computer network carefully. (Followed the instructions
carefully? Or were the instructions to set up the network carefully?)

Revised:

Mikail carefully followed the instructions for setting up the computer network.

Or:

Mikail followed the instructions for carefully setting up the computer network.

Because of the ability of adverbs to float around freely in a sentence, they are sometimes said to squint. A
squinting adverb is one which seems to modify two things at the same time. Like an optical illusion, a
sentence with a squinting adverb seems to mean first one thing and then another. In the following example
the prepositional phrase is modifying a verb or verbal-but which?

13
Incorrect:

She agreed after the Rooks' game to meet her friends at Moxie's. (Did she agree to this after the game? Or
did she agree to meet following the game?)

Revised:

She agreed to meet her friends at Moxie's after the Rooks' game.

Or:

After the Rooks' game she agreed to meet her friends at Moxie's.

Adverb clauses express relationships such as time, cause, purpose, and condition, using words like when,
because, in order that, and if. Adverb clauses can move around in a sentence without much problem. The
following adverb clause clearly modifies the verb will erode, whether the adverb clause is placed first or
last:

If the Sacramento River rises fast enough, some farmland will erode along the banks.

Some farmland will erode along the banks if the Sacrament River rises fast enough.

If you think a reader may misunderstand, feel free to move the clause or revise the sentence to eliminate
confusion.

4. Limiters precede.

Limiters are words like only, almost, just, nearly, or hardly. Place these words in front of their headwords.
Consider the difference in meaning in the following two examples:

He does math homework almost every day.

Or:

He almost does math homework every day.

The difference - did you catch it? - is that the first subject does his math, while the second does not. In the
first example, almost modifies every. In the second, almost modifies do. But one either does math, or
doesn't; "almost doing" math is nonsense, unless the intended meaning is that the second speaker thinks
about doing math, and gets ready to do math, but never actually does math.

Fourth error- Parallelism


According to the principle of parallelism, comparable sentence parts must be structurally and logically
similar. What does this mean in practice?

14
Here's an example:

The employees were upset by the company's low pay, poor working conditions, and that they did not
have enough outlets for their creativity.

This sentence has three comparable parts: the three things that upset the employees. The structure of the
first two parts is similar; both parts consist of a noun phrase (centered on the nouns pay and conditions,
respectively).

However, the third part has a different structure altogether: it is a clause containing a subject, a verb, and
an object. In order to make the sentence parallel, all three must either center on nouns or be clauses. In
the sentence below, all three center on nouns:

Right: The employees were upset by the company's low pay, poor working conditions, and shortage of
outlets for employees’ creativity.

It is not the case that every word has to be parallel within the three parallel phrases. However, the main
word or words in each element must be parallel. In this case, the most important words in the three items
in the list—pay, conditions, and shortage—are all nouns. As a result, the phrase the company's can now
apply to each of the parallel parts:

 the company's low pay


 the company's poor working conditions
 the company's shortage of outlets for employees’ creativity

The portion before the list starts (The employees were upset by the company's…) is called the root phrase.
Each parallel element could finish the sentence started by the root phrase, so if you're not sure what the
different elements are, try to plug each into the root phrase.

Parallelism Markers:-

How do you know when parts of a sentence need to be parallel to each other? Your clues will lie in
parallelism markers that link or contrast items and that force those items to be parallel. Markers can be
open, with a word or words between the two parallel elements, or closed, with a word or words between
the two elements as well as before the first element. For example:

15
The most common parallel markers are the three conjunctions: and, but, and or. Of these, and is the most
common. If you see any type of conjunction or XY idiom (such as either X or Y), ask yourself whether
parallelism is an issue in the sentence.

Parallel Elements:-

Almost any part of a sentence can be made parallel to another part—though you only need to do so if a
parallelism marker exists. The main word or clause in each element is required to be parallel, and each
element has to connect to the root phrase of the sentence.

In the table below, the parallelism marker is capitalized and the main parallel elements are underlined.

Element Example Marker


(open or
closed)

Nouns Her expression reflected BOTH anger ANDrelief. both X and Y


(closed)

Adjectives The park was NEITHER accessible NOR neither X nor


affordable. Y (closed)
We collected BOTH second- AND third- grade books. both X and Y
(closed)
Working Verbs The custodian cleaned the basement AND washed the X and Y (open)
windows.
Infinitive Verbs We would like NOT ONLY to hear your side of the story not only X but also
BUT ALSO to provide a response. Y (closed)
Participle The actor left quickly, waving to fans BUT ducking into a not only X but also
Modifiers car. Y (closed)
Prepositional It was important to leave the money in the drawer X rather than Y
Phrases RATHER THAN on the table. (Note: the prepositions do (open)
not necessarily have to be the same.)
Subordinate They contended that the committee was biased BUT that X but Y (open)
Clauses it should not be disbanded.

16
Often, the two parallel phrases or clauses may begin with the same signal word in order to remove
ambiguity about where the parallelism begins:

Wrong: I want to retire to a place WHERE I can relax AND I pay low taxes.

Right: I want to retire to a place WHERE I can relax AND WHERE I pay low taxes.

Without the repetition of the subordinator where, the first sentence could be read I want to retire … and I
pay low taxes. Repeating the where eliminates ambiguity. The signal words do not have to be the same
word. For example:

Right: There are many people WHO speak English BUT WHOSE parents do not.

When a closed marker is used, anything after the first portion of the marker applies only to the X element:

Wrong:Ralph likes BOTH those who are popular AND who are not as well-liked.

Right: Ralph likes BOTH those who are popular AND those who are not as well-liked.

Because parallelism starts with the marker word both, the X element is those who are popular. The main
parallel words, those who, cannot carry over to the Y element; they have to be repeated.

Some verbs or forms derived from verbs have more than one word: was opening, can lose, to increase. If
an open marker is used, the sentence can split apart these expressions, so that the first word or words
count across all of the elements:

In sum, when an open marker is used, check for ambiguity with respect to which words play the role of the
X element; if ambiguity exists, eliminate that choice. When a closed marker is used, make sure that both
the X and the Y elements contain the necessary starting words to form a structurally correct sentence

Fifth Error – Comparison

Comparisons are a subset of parallelism, but they get their topic because they require an additional check
in order to ensure that they are valid.

For example, what's wrong with the sentence below?

Wrong: Like Mary, Adam's car is green.

Poor Mary! She must be pretty sick if she's green, like Adam's car!
17
Right: LIKE Mary's car, Adam's car is green.

Common Errors with Comparisons :-

Common Error 1: Using the comparative instead of the superlative

 INCORRECT: He is the happier person I know.


 REVISION: He is the happiest person I know.
 INCORRECT: She is the more thoughtful person I know.
 REVISION: She is the most thoughtful person I know.

Common Error 2: Doubling up comparisons or superlatives

 INCORRECT: His car is more faster than mine.


 REVISION: His car is faster than mine.
 INCORRECT: His car is the most fastest.
 REVISION: His car is the fastest.

Common Error 3: Using empty comparisons (part of the comparison is missing)

 INCORRECT: The participants were more experienced.


 REVISION: The participants were more experienced than the previous participant pool.
 INCORRECT: The line moved more slowly.
 REVISION: The line moved more slowly than the line next to it.

Common Error 4: Using ambiguous comparisons (the comparison has more than one possible meaning)

 INCORRECT: She likes pizza better than her husband. (Does this mean that pizza is better than her
husband?)
 REVISION: She likes pizza better than her husband does. (Now it is clear that the comparison is who
likes pizza more.)
 INCORRECT: Her suitcase is bigger than John. (Does this mean that the size of the suitcase is larger
than another person?)
 REVISION: Her suitcase is bigger than John’s. (Now it is clear that the comparison is about two
suitcases, not about John.)

Common Error 5: Missing the article “the” in the superlative

INCORRECT: Finishing quickly was least important task.

REVISION: Finishing quickly was the least important task.

INCORRECT: The youngest girl was also littlest.

REVISION: The youngest girl was also the littlest.

18
Sixth Error- Verb Tense
There are three types of tenses, Past, Present and Future. While framing a sentence, there is often
confusion in verbs and clauses. Also, students often commit errors in tenses such as confusing between
past and present or future tense in a single sentence, or within the tense, such as confusion between
present perfect tense and present continuous tense.

Types of Errors in Tenses:-

1. Confusing the Present Tense:-


The present tense has some intimidating labels: simple present tense, present perfect tense,
present continuous tense, and present perfect continuous tense. Here are the examples on the
present tense:
Simple Present Present Perfect Tense

My friend works there. My friend has worked there.

Present Continuous Present Perfect Continuous Tense

My friend is working there. My friend has been working there.

so, what's the difference between, "My friend works there" and, "My friend is working there"? The
difference is slight. "My friend works there" tells us it's a habitual action. Beyond that, it is a
statement of fact.

"My friend is working there" refers to an action taking place right now. It's continuous, or ongoing,
at this very moment. This leaves room for interpretation, as it's not to say the friend will be working
there tomorrow or next week.

2. Confusing the Past Tense:-

Past tense. Past perfect tense. It sounds pretty heavy. In truth, it's nothing too foreign! They're phrases
we use day in and day out, without even thinking about them. Let's take a look:

Past Tense Past Perfect Tense

I walked the dog. I had walked the dog.

Past Continuous Tense Past Perfect Continuous Tense

I was walking the dog. I had been walking the dog.

Notice the addition of the word "continuous" in the bottom two rows? All that means is the action was
taking place in the past at a continuous rate. It was on-going. Think of it like this: in the past, I walked
the dog. Even before that, I had walked the dog. It's kind of like varying degrees of past tense.

19
If you mention something that happened in the past and then want to reference something that
happened even earlier than that, be sure to use the past perfect tense. If you think you'll run into past
tense mistakes, consider writing your story in the present tense.

3. Switching in and out of Present and Past Tense:-

Sometimes, we write the way we speak. And, in our everyday speech, it's easy to slip by with the wrong
tense. However, the written word lives on forever (and faces the possibility of being dissected by book
clubs and lit classes everywhere). Take a look at this example:

"Gracie! Get down here right now!" Nana yelled. Nana was a fierce drill sergeant and is always nagging me
about punctuality.

Did you catch it? Is there anything wrong with that sentence? Well, is Nana dead or alive? Is she an active
sergeant or a retired sergeant? If she's alive and well, and still active duty, you wouldn't say, "Nana 'was' a
fierce drill sergeant."

Rather, you'd use the verb "is." Also, note the change in tense again from "yelled" (simple past) to "is
always nagging" (present continuous). This bit of text would do well to remain in the simple past or present
tense.

4. Misusing the Past Continuous Tense:-

Let's think back to the past continuous tense. That's the tense that indicates something happened in
the past, at a continuous rate. The past continuous tense is something along the lines of, "I was walking
the dog." The past perfect continuous tense is something along the lines of, "I had been walking the
dog."

The trouble with the past continuous tense is, if it wasn't an ongoing affair in the past, all those helping
verbs and -ing words are only going to make your prose appear cumbersome. Take a look at these two
examples:

Simple Past Tense - "He tampered with the alarm system before Nora walked into the kitchen."

Past Continuous Tense - "He was tampering with the alarm system before Nora walked into the
kitchen."

One isn't "right" and one isn't "wrong." It's just a matter of whether or not you want to signify a
continuous event. So, had this man finished tampering with the alarm system before Nora walked into
the kitchen? Or was he continuously tampering with the alarm system?

5. Altering the Future Tense:-

One thing we've learned is if you start in one tense, you should generally remain in that tense.
However, the future tense likes to play by a different set of rules. Take a look at this example. See if you
can spot the error:
20
I will hire you full-time when your degree will be complete.

From the standpoint of uniformity, you might think this is the proper construct. "Will hire" is in the
future tense, so shouldn't the subordinate clause also be in the future tense? Not in this case. The
proper construct is:

I will hire you full-time you when your degree is complete.

When the verb in the main clause is in the future tense, the verb in the subordinate clause should shift
to the present tense.

Seventh Error- Error of Participle


A PARTICIPLE is a verbal modifier ending in "-ing" or "-ed," depending on whether it is a PRESENT
PARTICIPLE or a PAST PARTICIPLE. The "-ing" ending is also an ending used in the continuous tense form of
verbs, and an "-ed" ending is used with the perfect tense form of verbs. This provides a clue as to the role
of a participle: it describes something involved in an ongoing or completed activity. For example, "a rolling
barrel" or "a tuned piano" use the verbs "roll" and "tune," respectively, to describe the active state of a
barrel and a piano.

PRESENT PARTICIPLES versus GERUNDS

Novices in the concept of verbals find it difficult to distinguish a present participle from a gerund because
both of them use the continuous tense ending "-ing." (See below for more about the verb tense.) This is
understandable, and the more students practice recognizing nouns and adjectives in their reading and in
their own writing, the more easily they may distinguish participles from gerunds. The simplest way to
determine the difference is to apply the basic questions that identify nouns and adjectives:

If it answers the question "What?" or "Who?" then it's a noun, and all gerunds are nouns.

Formal dancing is making a comeback.

What is making a comeback? Dancing is. "Dancing" is a gerund.

If it answers the question "What's it like?" "Which?" or "What kind?" then it's an adjective, and all
participles are adjectives.

That formally dancing couple are ballroom competition champions.

Which couple? The dancing couple, so "dancing" is a participle.

PAST PARTICIPLES

Past participles are blessedly simple in comparison to present participles. They are derived from the perfect
verb tense, which answers the question, "What has occurred?" and always starts with a form of the
auxiliary verb "have" then finishes with a past tense form of the base verb.

I have completed the assignment.

21
A past participle verbal will utilize only the perfect tense base verb, not its auxiliary, and it does so in a way
that describes in answer to the question, "Which?" or "What kind?"

I submitted the completed assignment at the start of the next class period.

In this example "completed" is not part of the action of the clause; rather, "submitted" is. Instead,
"completed" describes which assignment, or what kind of assignment was submitted. As with all
participles, this past participle modifies a noun ("assignment") instead of conveying action or carrying over
onto an object. Diagramming the placement of a past participle will help to demonstrate its modifying role
more clearly.

A common error

Read the following sentence.

Having bitten the postman, the farmer decided to shoot the dog.

This means that it was the farmer who bit the postman and not the dog! To avoid confusion, the sentence
should be rewritten as follows.

The dog having bitten the postman, the farmer decided to shoot it.

The participle is a verbal adjective. It should be related to a proper subject of reference. If the subject is
lacking or if a wrong subject is used, the whole sentence would be wrong. Other examples are given below.

Being a rainy day, we had to abandon the match. (Wrong)

It being a rainy day, we had to abandon the match. (Right)

Being a small cot, he could not sleep on it. (Wrong)

It being a small cot, he could not sleep on it. (Right)

Being too costly for him, he could not buy the coat. (Wrong)

It being too costly for him, he could not buy the coat. (Right)

Eighth Error – Redundancy


Redundancy means superfluity or using words unnecessarily or using words for a second time.

Example-1:

• If all of us cooperate together, we will succeed.

In this sentence, the words cooperate and together have been used. But both these words convey the
same meaning.

One of the two words should be dropped in order to make the sentence a correct one.

• If all of us cooperate, we will succeed.

22
• If all of us work together, we will succeed.

Both these sentences are correct ones.

Example-2:

• The accused was guilty of false misstatement.

This sentence uses false and misstatement whereas both these words convey the same meaning.

The correct sentence is:

• The accused was guilty of misstatement.

Example-3:

• It was the general consensus of opinion that we must go to the movie.

The two words which convey the same meaning are consensus and opinion.

One word should be removed to make this sentence correct one.

The correct sentence is:

• It was the general opinion that we must go to the movie.

Example-4:

• The three brothers had nothing in common with each other.

Here also two phrases in common and with each other have been used to convey the same meaning.

The correct sentence is:

• The three brothers had nothing in common.

These examples might have made it clear for you how to avoid Redundancy in your sentences.

Example-5:

• I am enclosing herewith my bio-data.

Enclosing and herewith are the two words which convey the same meaning.

The correct sentence is:

• I am enclosing my bio-data.

23
Tutorial
Intermediate :-
Q1. The primary drama of Homer's famed Iliad revolves around anger amongst the renowned and wrathful
warrior Achilles and Agamemnon, the King of Mycenae and the leader of Achaean army.

A) revolves around anger amongst the renowned and wrathful warrior Achilles and Agamemnon, the
King of Mycenae and the leader of Achaean army

B) revolve in the anger between the renowned and wrathful warrior Achilles and the King of Mycenae
and the leader of the Achaean army Agamemnon

C) revolves around the anger between Achilles, a renowned and wrathful warrior, and Agamemnon,
king of Mycenae and the leader of the Achaean army

D) revolves in the anger between Achilles, a renowned and wrathful warrior, and Agamemnon, king of
Mycenae and the leader of the Achaean army

E) revolve around Achilles, a renowned and wrathful warrior, and his anger toward Agamemnon, king
of Mycenae and the leader of the Achaean army

Q2. Some officials both at the Treasury Department and the Securities and Exchange Commission recently
said in off-the-record conversations that it may be a good idea to require that all large and highly leveraged
banks should decrease their debt and should increase their asset base.

A) to require that all large and highly leveraged banks should decrease their debt and should increase
their asset base

B) requiring that all large and highly leveraged banks decrease their debt and increase their asset base

C) to require of all large and highly leveraged banks the decreasing of debt and an increase in their
asset base

D) requiring that all large and highly leveraged banks to decrease their debt and to increase their asset
base

E) to require all large and highly leveraged banks to decrease their debt and increase their asset base

Q3. Environmentalists associated with the United Nations Environment Programme predict that if the
current trends associated with global warming continue, thousands of acres of pristine land is in danger of
undergo potentially irrevocable changes that could alter the planet's ecosystem forever.

A) is in danger to undergo

B) are in danger of undergoing

C) is in danger of undergoing

D) are in danger to undergo

24
E) are in danger for undergoing

Q4. Among the litany of threats that many Israelis face, the potential for a nuclear-armed Iran is perhaps
the more scary as this scenario could engulf the region in a violent war. This would likely result in
historically unseen amounts of destruction, even for a region whose history is marred by perennial
violence.

A) perhaps the more

B) perhaps the most

C) possibly, perhaps the most

D) possibly the greatest

E) possibly the great

Q5. Warning that terrorists remain determined to strike the country, senior government officials urged
citizens to remain vigilant and report suspicious behavior, taking actions to speed up the recovery from an
economic catastrophe should terrorists succeed in launching an attack.

A) to remain vigilant and report suspicious behavior, taking actions necessary to speed up the recovery
from an economic catastrophe

B) to remain vigilant, reporting suspicious behavior and take actions to speed up the recovery from an
economic catastrophe

C) to be remaining vigilant, reporting suspicious behavior, and taking actions necessary to speed up
the recovery from an economic catastrophe

D) to remain vigilant, report suspicious behavior, and take actions to speed up the recovery from an
economic catastrophe

E) to be more vigilant in reporting suspicious behavior and taking actions necessary to speed up the
recovery from an economic catastrophe

Q6. According to a recent study, financial problems, together with their serious ramifications, ranks as one
of the high causes of marital stress in America.

A) ranks as one of the high causes of marital stress in America

B) rank as one of the leading causes of marital stress in America

C) rank as one of the most high causes of marital stress in America

D) ranks as one of the leading causes of marital stress in America

E) rank as one of the most leading causes of marital strife in America

Q7. After years of working on Wall Street, an apartment in lower Manhattan still felt like his home for
famed investment banker John E. Callan, even though he also owned a house in Rye and in East Hampton.

25
A) an apartment in lower Manhattan still felt like his home for famed investment banker John E.
Callan, even though he also owned a house in Rye and in East Hampton

B) famed investment banker John E. Callan still considered his lower Manhattan apartment home,
even though he also owned a house in Rye and in East Hampton

C) an apartment in lower Manhattan still felt like home for famed investment banker John E. Callan,
even though he also owned a house in Rye and in East Hampton

D) John E. Callan, a famed investment banker, still considered his lower Manhattan apartment home,
even though he too owned a house in Rye and in East Hampton

E) famed investment banker John E. Callan still considered his lower Manhattan apartment home,
even despite his owning of a house in Rye and in East Hampton

Q8. A month ago, a well respected sell-side analyst released a report predicting that prices in equity
markets will reach record highs in the next 12 months. Immediately after the release of the report, some
institutional investors who feared a continuing loss in shareholder value publicly expressed optimism in a
strong near-term future for equities.

A) who feared a continuing loss of

B) who had feared the continuation of losses of

C) who had feared continuing losses in

D) who feared continuing losses in

E) who had feared a continuing loss of investors'

Q9. One of the most important choices facing the new administration at this moment is whether they
should continue to advance contractionary fiscal policy.

A) this moment is if they should continue to advance contractionary fiscal policy

B) this present moment is whether it should continue to advance contractionary fiscal policy

C) this moment is whether it should continue to advance contractionary fiscal policy

D) this moment is whether they should continue to advance contractionary fiscal policy

E) this present moment is whether it should continue the advancement of contractionary fiscal policy

Q10. As tensions rose in the days leading up to the President’s re-election bid, one hostile commentator
who decried the President’s controversial war policies exclaimed: "If I was the President, I would be
ashamed of myself and also resign immediately." Ironically, this actually emboldened the President’s allies.

A) If I was the President, I would be ashamed of myself and also resign immediately

B) If I were the President, I would be ashamed of myself and resign immediately

C) If I was the President, I would be ashamed of myself and resign immediately


26
D) If I were the President, I would be ashamed of myself and also would resign immediately

E) If I was the President, I would be ashamed of myself and would resign immediately

Q11. Under the provisions of the United States Constitution and the laws of the United States, the Federal
government cannot detain an American citizen indefinitely without cause and is required either to bring
charges against the individual being held, in which case he is entitled to a lawyer, or that the government
must release him.

A) that the government must release him

B) release him

C) to proceed in releasing him

D) the government must release him

E) they must release him

Q12. he prosecutor’s argument, which hinged on a little known provision in a 1972 law, differed
considerably from the plaintiff, who contended that a landmark 1999 Supreme Court decision supported
his argument.

A) the plaintiff

B) that of the plaintiff

C) those from the plaintiff

D) that espoused by the plaintiff

E) that from the plaintiff

Q13. AMD employees used their creativity, intellect, and ingenuity to develop faster processors than those
offered by many competitors, to enable them to gain substantial market share from rival firm Intel.

A) to enable them to gain substantial market share from rival firm Intel

B) enabling AMD to gain substantial market share with rival firm Intel

C) enabling them to gain substantial market share from rival firm Intel

D) to enable AMD to gain substantial market share with rival firm Intel

E) enabling AMD to gain substantial market share from rival firm Intel

Q14. In 1986, socialist President Mitterrand was forced to appoint conservative Jacques Chirac as prime
minister, resulting in a period known as the Cohabitation, where him and Chirac governed together..

A) resulting in a period known as the Cohabitation, where him and Chirac governed together.

B) resulting in a period known as the Cohabitation, where them governed together.

27
C) where him and Chirac governed together, resulting in a period known as the Cohabitation.

D) where he and Chirac, resulting in a period known as the Cohabitation, governed together.

E) resulting in a period known as the Cohabitation, where he and Chirac governed together.

Q15. A ground-breaking report written by a major group of scientists has indicated that much of the
previously untraceable pollutants in stream water known to kill fish and harm humans comes from polluted
rain water and irresponsible chemical dumping by large corporations.

A) much of the previously untraceable pollutants in stream water known to kill fish and harm humans
comes from

B) much of the previously untraceable pollutants in stream water known to kill fish and harm humans
come from

C) many of the previously untraceable pollutants in stream water known to kill fish and harm humans
comes from

D) much of the previously untraceable pollutants in stream water known to kill fish and harming
humans come from

E) many of the previously untraceable pollutants in stream water known to kill fish and harm humans
come from

Q16. Many United States Congressmen recently voted to give the National Security Agency new powers
enabling them to eavesdrop on telephone calls without a court warrant and pass along evidence from
those calls to other government agencies.

A) enabling them to eavesdrop on telephone calls without a court warrant and pass along evidence
from the calls

B) enabling it to eavesdrop on telephone calls without a court warrant and pass along evidence from
the calls on which it eavesdropped

C) enabling it to eavesdrop on telephone calls without a court warrant and pass along evidence from
those calls

D) enabling them to eavesdrop on telephone calls without a court warrant and be passing along
evidence from those calls

E) enabling it to eavesdrop on telephone calls without a court warrant and be passing along evidence
from those calls

Q17. Despite the influx of international aid to a particularly war-torn region of East Africa during the
waning months of 2006, in early 2007, many indigenous people unable to find food left their home to
travel west where other tribal groups historically experienced fewer difficulties producing food, water, and
land.

A) their home to travel west where other tribal groups historically experienced less difficulties

28
B) home to travel west where other tribal groups historically experienced fewer difficulties

C) their home to travel west where other tribal groups historically experienced fewer difficulties

D) home to travel west in search of other tribal groups who historically had experienced fewer
difficulties

E) home to travel west where other tribal groups historically had experienced fewer difficulties

Q18. A recent decision made by the United States Attorney for the Southern District of New York, who
many consider a leading contender for a cabinet appointment, is seen by most legal observers as both a
savvy political maneuver and an abrogation of professional responsibility.

A) who many consider a leading contender for a cabinet appointment, is both seen by most legal
observers as a savvy political maneuver and an abrogation of professional responsibility

B) whom many consider a leading contender for a cabinet appointment, is seen by most legal
observers as both a savvy political maneuver and an abrogation of professional responsibility

C) whom many consider a leading contender for a cabinet appointment, is both seen by most legal
observers as a savvy political maneuver and an abrogation of professional responsibility

D) who many consider a leading contender for a cabinet appointment, is seen by most legal observers
as both a savvy political maneuver and an abrogation of professional responsibility

E) whom is seen as a leading contender for a cabinet appointment, is also seen by most legal
observers as both a savvy political maneuver and an abrogation of professional responsibility

Q19. Added to the increase in monthly wages discussed last spring, the dining hall employees are currently seeking
improved insurance coverage.

a) Added to the increase in monthly wages discussed last spring, the dining hall employees are currently
seeking improved insurance coverage.

b) Added to the increase in monthly wages which had been discussed last spring, the employees of the dining
hall are currently seeking an improved insurance coverage.

c) The dining hall employees are currently seeking improved insurance cover- age added to the increase in
monthly wages that were discussed last spring.

d) In addition to the increase in monthly wages that were discussed last spring, the dining hall employees are
currently seeking improved insurance cover- age.

e) In addition to the increase in monthly wages discussed last spring, the em- ployees of the dining hall are
currently seeking improved insurance coverage

Q20. Because chickens lack teeth, they need another way to break apart the food they eat before reaching the
stomach, and for this reason, chickens have a gizzard in which stones they swallow are used to grind their food.

A) before reaching the stomach, and for this reason, chickens have a gizzard in which stones they swallow are
used to grind their food.

29
B) before it reaches the stomach, and for this reason, chickens have a gizzard in which stones they have
swallowed is used to grind their food.

C) before it reaches the stomach, and for this reason, chickens have a gizzard in which stones they swallow are
used to grind their food.

D) before reaching the stomach, and for this reason, chickens have a gizzard in which stones they have
swallowed is used to grind their food.

E) before it reaches the stomach, and for this reason, chickens have a gizzard in which stones they have
swallowed are used to grind their food.

Advance:-
Q1. The catastrophic San Francisco Earthquake at the turn of the century destroyed numerous buildings
and many were led to believe that the city had become a permanent disaster zone.

(A) many were led to believe that the city had become

(B) many had been led to believing of the city as if it were

(C) the belief this led to was that the city had become

(D) led many to the belief of the city as if it were

(E) led many to believe that the city had become

Q2. Riddled with bullets, shattered by bombs, and hidden in alleys, the historic build- ings in Lodz, Poland,
were long ignored by tourists, traveling instead to more well-known memorial museums.

(A) the historic buildings in Lodz, Poland, were long ignored by tourists, travel- ing

(B) the historic buildings in Lodz, Poland, were long ignored by tourists, who traveled

(C) tourists long ignored the historic buildings in Lodz, Poland, traveling

(D) tourists long ignored the historic buildings in Lodz, Poland and traveled

(E) tourists long ignored the historic buildings in Lodz, Poland; they depended

Q3. New high-combustion models of engines show the potential of being able to pro- duce high
horsepower and performance without the costly requirements of main- tenance and consuming of special
racing fuel by earlier high horsepower models.

(A) requirements of maintenance and consuming of special racing fuel by earlier high horsepower models

(B) requirements by earlier high horsepower models of consuming of special racing fuel and maintenance

(C) requirements for consuming of special racing fuel and maintenance of earlier high horsepower models

30
(D) consumption of special racing fuel and maintenance that was required by earlier high horsepower
models

(E) maintenance and consumption of special racing fuel that were required by earlier high horsepower
models

Q4. Declining values for bonds, the financial vehicles against which investors hedge to get through the
bear market, is going to force currency trading to increase.

(A) the financial vehicles against which investors hedge to get through the bear market, is

(B) which investors use as financial vehicles to hedge against to get through the bear market, is

(C) the financial vehicle which is hedged against by investors to get through the bear market is

(D) which investors use as financial vehicles to hedge against to get through the bear market, are

(E) the financial vehicles against which investors hedge to get through the bear market, are

Q5. The sharp contrast in sales of sports memorabilia seen in sports in which most of the participants are
male and such sales in sports in which most of the partic- ipants are female have demonstrated that
women’s sports are still lacking dedicated fans.

a) seen in sports in which most of the participants are male and such sales in sports in which most of the
participants are female have

b) seen in sports in which most of the participants are predominately male over those that are
predominately female have

c) that favors sports in which most of the participants are male over sports in which most of the
participants are female have

d) that favors sports in which most of the participants are male over sports in which most of the
participants are female has

e) seen is sports in which most of the participants are male and such sales in sports in which most of the
participants are female has

Q6. A male musician can find a career as a solo performer, an orchestra member, or a music teacher after
he graduates from college with a degree in music, depending on his talent.

a) A male musician can find a career as a solo performer, an orchestra member, or a music teacher
after he graduates from college with a degree in music, depending on his talent.

b) After graduating from college with a degree in music, depending on his tal- ent, a male musician can
find a career as a solo performer, an orchestra member, or a music teacher.

c) After graduating from college with a degree in music, a male musician’s tal- ent will determine if he
can find a career as a solo performer, an orchestra member, or a music teacher.

31
d) Talent determines whether a male musician, after graduating from college with a degree in music,
can find a career as a solo performer, an orchestra member, or a music teacher.

e) The talent of a male musician, after graduating from college with a degree in music, will determine
whether he can find a career as a solo performer, an orchestra member, or a music teacher.

Q7. Recently discovered gravitational lensing around certain proximate stars strongly suggests that the
nine planets of our solar system are a common phenomenon in the universe rather than developing
incidentally from a unique galactic phenomenon several billion years ago.

(A) rather than developing incidentally from

(B) rather than a type that developed incidentally from

(C) rather than a type whose development was incidental of

(D) instead of developing incidentally from

(E) instead of a development that was incidental of

Q8. During gladiator matches, the unfair match-up between a prisoner with a short sword and ten soldiers
with horses and whips can drive the prisoner to a state of manic frenzy, like a rampaging bull whose rage
increases when its hide is pierced with swords.

(A) like a rampaging bull whose rage increases when its hide is pierced with swords

(B) like the increased rage of a rampaging bull when its hide is pierced with swords

(C) like a rampaging bull that increases rage while rampaging with its hide pierced with swords

(D) just as a rampaging bull that increases rage by piercing its hide with swords

(E) just as a rampaging bull’s rage increases when it is pierced with swords

Q10. Over the last 10 years, the population of deer in America increased dramatically over the past decade,
while the number of drivers who report car accidents with deer on highways is more than four times what
it was.

(A) increased dramatically over the past decade, while the number of drivers who report car accidents
with deer on highways is more than four times what it was

(B) increased dramatically, while the number of drivers reporting car accidents with deer on highways
is more than quadruple what it was at that time

(C) has increased dramatically, while the number of drivers reporting car acci- dents with deer on
highways has more than quadrupled

(D) has increased dramatically over the past decade, while the number of drivers reporting car
accidents with deer on highways is more than four times what it was at that time

32
(E) has increased dramatically over the past decade, while the number of drivers who are reporting car
accidents with deer on highways are more than quadruple what they once were

Q11. Erasmus's tomb lies inside the Basel Munster, located in Switzerland, an architectural monument
which having survived medieval earthquakes, and remains one of Switzerland's most well-known buildings
to this day.

A) Erasmus's tomb lies inside the Basel Munster, located in Switzerland, an architectural monument which
having survived medieval earthquakes, and

B) Erasmus's tomb lies inside Switzerland's Basel Munster, an architectural monument that survived
medieval earthquakes and

C) Switzerland's Basel Munster, an architectural monument that survived medieval earthquakes, houses
Erasmus's tomb,

D) The Basel Munster, in Switzerland, an architectural monument which, having survived medieval
earthquakes, is now home to the tomb of Erasmus and

E) The tomb of Erasmus, being housed inside Switzerland's Basel Munster, is an architectural monument
that survived medieval earthquakes and

Q12. An analysis of sixteenth century probate inventories in the major English towns show that even some
artisans and yeomen owned silver spoons, cups or salt cellars.

A. show that even some artisans and yeomen owned silver spoons, cups or

B. show that some artisans and yeomen even owned silver spoons, cups or

C. show that even some artisans and yeomen owned silver spoons, cups and

D. shows that more number of artisans and yeomen owned even silver spoons, cups and

E. shows that even some artisans and yeomen owned silver spoons, cups or

Q13. Because Albert is the most experienced and he is therefore the best ballet dancer in the company, he
is being increasingly viewed by the director as the best candidate for the role of the Nutcracker.

(A) and he is therefore the best ballet dancer in the company, he is being increasingly viewed

(B) he is therefore the best of ballet dancers, and it has increased the view

(C) and therefore the best ballet dancer, he is being increasingly viewed

(D) and therefore he is the best of ballet dancers, there is an increasing view

(E) therefore being the best of ballet dancers, it is increasingly viewed

Q14. The political and social forces that may facilitate a dictator’s rise to power include sudden crashes in
the economy, discrimination and other methods of finding scapegoats, inciting the masses to rebellion, and
their protesting that the cur- rent government may still be inadequate.

33
(A) inciting the masses to rebellion, and their protesting that the current government may still be
inadequate

(B) inciting the masses to rebellion, and a protest that the current government may still be inadequate.

(C) an incitement of the masses to rebellion, and a protesting that the current government may still be
inadequate.

(D) an incitement of the masses to rebellion, and a protest of the still inadequate current government

(E) an incitement of the masses to rebellion, and a protest that the current government may still be
inadequate

Q15. The disciplinary decisions teachers make are less strict for girls than they are for boys because they
usually cause less trouble and are more repentant.

(A) The disciplinary decisions teachers make are less strict for girls than they are for boys because they
usually cause less trouble and are more repentant.

(B) Because they usually cause less trouble and are more repentant, the disci- plinary decisions
teachers make are less strict for girls than the disciplinary decisions are for boys.

(C) The disciplinary decisions teachers make are less strict for girls than boys because they usually
cause less trouble and are more repentant.

(D) Because girls usually cause less trouble and are more repentant than boys, the disciplinary decisions
that teachers make for girls are less strict than boys.

(E) The disciplinary decisions teachers make are less strict for girls than they are for boys because girls
are usually less troublesome and more repentant than boys are.

Q16. Thought to emanate from a tiny gland on the underside of their bodies, ants leave behind pheromone
trails that can be used as signals or messages for other ants.

(A) Thought to emanate from a tiny gland on the underside of their bodies, ants leave behind
pheromone trails that can be used

(B) Ants leave behind pheromone trails that are thought to emanate from a tiny gland on the underside
of their bodies, and they can use this

(C) Thought to emanate from a tiny gland on the underside of ants’ bodies, pheromone trails left
behind can be used

(D) Emanating it is thought from a tiny gland on the underside of their bodies, ants leave behind
pheromone trails they can use

(E) Emanating, it is thought, from a tiny gland on the underside of their bodies, pheromone trails are
left behind by ants that can be used

34
Q17. The sharp contrast in sales of sports memorabilia seen in sports in which most of the participants are
male and such sales in sports in which most of the participants are female have demonstrated that
women’s sports are still lacking dedicated fans.

(A) seen in sports in which most of the participants are male and such sales in sports in which most of
the participants are female have

(B) seen in sports in which most of the participants are predominately male over those that are
predominately female have

(C) that favors sports in which most of the participants are male over sports in which most of the
participants are female have

(D) that favors sports in which most of the participants are male over sports in which most of the
participants are female has

(E) seen is sports in which most of the participants are male and such sales in sports in which most of
the participants are female has

Q18. Astronomical occurrences can be viewed in a religious light; many people are known to rekindle their
faith after the observation of a meteor shower.

(A) many people are known to rekindle their faith after the observation of a meteor shower

(B) many people are known to have rekindled their faith once a meteor shower has been observed

(C) there are many known people who have rekindled their faith once a meteor shower has been
observed

(D) after a meteor shower is observed, there are many known people who have rekindled their faith

(E) rekindling their faith is known for many people after a meteor shower is observed

Q19. Standardized test scores of minorities are well below that of white students in spite of economic
differences that are shrinking between the races.

(A) well below that of white students in spite of economic differences that are shrinking

(B) much below that of white students’ despite economic differences shrinking

(C) much below white students in spite of shrinking economic differences

(D) well below those of white students in spite of shrinking economic differences

(E) below white students’ despite their economic differences that are shrinking

Q20. Because many different cultures have different cultural norms, misunderstandings among different
cultures are far greater as that among individuals from the same culture: slurping one’s soup, in Japan a
gesture of appreciation for the cook, is unforgivably rude in America.

(A) among different cultures are far greater as that among individuals from

35
(B) among different cultures are far greater than that among individuals from

(C) among different cultures are far greater than those among individuals of

(D) between different cultures are far more than that between individuals of

(E) between different cultures are greater by far than is that between individuals from

36
UNIT 2

Voice and Accent


What is accent?
An accent is a part of a dialect and refers to the way people pronounce specific words and phrases of the
same language differently from each other. For example, an American person will say “Hello,” while a
French person will say “ello,” omitting the first “H” due to their native French language.

Statistics show that the most spoken language worldwide is English, with almost two billion people being
able to speak it, of which only 375 million are native speakers. This means that people are much more
likely to speak English with someone for whom English is a second or foreign language than with a native
English speaker. Due to this, people who have not been exposed to different accents and dialects might
have trouble understanding people, depending on where they are and who they are talking to.

The Sounds of English:-


In the English alphabet there are 26 letters, but these letters produce 44 sounds. For this reason, one letter
is used to produce more than one sound. In order to know the correct pronunciation certain symbols
denoting these sounds have been devised and standardized. By learning these symbols you will be able to
find the correct pronunciation of any word in a standard dictionary. These symbols will also help you to go
through the book with guidance for correct pronunciation of words and conversations. These sounds are
classified into two types:

What is a vowel sound?


There are only 5 vowel letters in English (a,e,i,o,u), but the sounds indicated by these 5 letters are 20. A
vowel sound is produced when the air comes out of the mouth freely without any blockage or closure in
the mouth cavity by the tongue, teeth, lips, etc. The vowel sounds are of two types:

Single vowel sounds: When these sounds are produced, the tongue remains in the same position even
when the sound is prolonged. They consist of one sound only and are called pure vowels as in ‘sit’, ‘pot’,
‘heat’, etc. (the underlined letters indicate the single vowel sounds)
37
Double vowel sounds: These vowel sounds are a combination of two single vowel sounds and in
pronouncing them the tongue moves from one position to another. For instance, in the word ‘light’ the
sound of ‘i’ is a combination of the vowel sounds /a:/ as in ‘art’ and /i/ as in ‘it’. The words ‘hair’, ‘toy’ and
‘poor’ also contain double vowel sounds.

S.No Symbol Examples with Common Examples with Other Spellings


. Spellings

1 /iː/ Sheep tea, me, fi_eld, key, receive

2 /ɪ / bit, igloo gym, bucket, cabbage

3 /e/ egg, net, bet bread, said

4 /æ/ bat, cat, rat, sat _

5 /ʌ/ but, cut, shut _

6 /ɑː/ car, large, march clerk, heart, guard

7 /ɔ/- hot, pot, otter swan, pause

8 /ɔː/ caught, taught bought, paw

9 /u/ or /ʊ/ put, truth hood, could

10 /uː/- } uniform, unity, cube cue, shoe, new, boot

11 /ə:/or /3:/ fern, perfect shirt, purple, earth, worm

12 /ə/ about, alike _

13 /eɪ/ tray, pray train, great, eight, skate

38
14 /aɪ/ kite, bite, night, I tie, fly, height

15 /ɔɪ/ boy, toy noise

16 /əu/ home, bone, nose know, grow

17 /au/ how, now _

18 /iə/ ear, fear deer

19 /eə/ chair, pair there, their, square, bear, wear

20 /uə/or/ʊə/ poor tour ,sure

What is a consonant sound?


A consonant sound is one in which the air stream coming out of the lungs is modified in the mouth cavity
by some blockage created by the tongue, lips, etc. When you say the sound of ‘p’ as in ‘pen’ a blockage is
created in the outgoing breath by closing both the lips and thus, a consonant sound is produced.

We have learnt earlier that the speech sounds can be divided into two types: the vowel sounds and the
consonant sounds.

The consonant sounds are 24 in number. A brief description of these is given below. These may be grouped
according to:

a) the way they are pronounced

b) the place of pronunciation in the mouth, and

c) whether they are voiced or unvoiced (a sound is ‘voiced’ if accompanied by vibrations in the vocal
cords and it is ‘unvoiced’ when there are no vibrations in the vocal cords.)

We shall deal with consonant sounds in groups based on the first classification i.e. the way the sounds are
produced. This classification includes;

1. Plosives: /p/, /b/, /t/, /d/, /k/, /g/.

2. Affricates: /tʃ/, /dʒ/.

3. Fricatives: /f/, /v/, /θ/, /ð/, /s/, /z/, /ʃ/, /ʒ/, /h/.

4. Lateral: /l/.

5. Frictionless Continuant: /r/. 6. Nasals: /m/, /n/, /ŋ/.

7. Semi - Vowels: /w/, /j/.

S.No. Spelling Symbol Examples with common Examples with other

39
spellings spellings

1 p /p/ pit, pin -

2 b /b/ bit, bat -

3 t /t/ time, ten -

4 d /d/ door, dog -

5 k /k/ kite, kit cat, duck, queen, chemistry

6 g /g/ get, go -

7 ch /tʃ/ chop, chick -

8 j /dʒ/ judge, jam giraffe

9 f /f/ fan, fun telephone, rough

10 v /v/ van, vine -

11 th /θ/ think, thought -

12 th /ð/ that, this -

13 s /s/ send, see circle

14 z /z/ zip, zoo pins

15 sh /ʃ/ shop, shoe invitation, sure, chef,

precious, permission

16 s /ʒ/ leisure, pleasure -

17 h /h/ hen, happy -

18 m /m/ man, monkey -

19 n /n/ nice, night knife, gnome

20 ing /ŋ/ ring, sing, English tongue

21 l /l/ leg, long -

22 r /r/ rat, run write

23 w /w/ wet, wine -

24 y /j/ yet, yes -

40
Place of articulation:-
Place of articulation is defined as the location at which two speech organs approach or come together in
producing a speech sound, as in the contact of the tongue and the teeth to form a dental sound.

1. Bilabial
Bilabials are consonant sounds produced by using both lips together.
Read this word out loud and notice how you're using both lips to pronounce the letters in bold: bump.

2. Labiodental

Labiodentals are also pretty straightforward; they are articulated by using both the lower lip and the upper
front teeth.

Examples of these sounds in English are pretty much in any word that contains the letters F and V.

Pronounce the word favour and notice the point of articulation.

41
3. Dental

Some languages have dental consonants where only the tongue and the teeth are used.

Words such as : thing and this.

4. Alveolar

The point of articulation of alveolar consonants is situated near the alveolar ridge, which is the area laying
between the upper front teeth and the palate.

Pronounce words such as tow and zap

5. Post-Alveolar

Post alveolar are consonants articulated with the tongue near or touching the back of the alveolar ridge,
placing them a bit further back in the mouth than the alveolar.

Try the words “ship", "'chill", "vision", and "jump"

42
6. Palatal

We are now getting to a depth in the mouth where portions like the back of the tongue are starting to be
used to produce sounds.

The approximant [j], found in the word yet, is also a palatal. Notice that it is the back of the tongue that
comes into near contact with the palate when pronouncing the Y in that word.

7. Velar

There are a few velars in English, so it should be pretty straightforward to learn what their point of
articulation is. read out loud the word king, which has both the velars *k+ and *ŋ+ as both the first and last
consonant sounds, respectively.

8. Glottal

The glottis, as much as we might not realize it, is used for the h sound in English; [h].

Just pronounce the words happy and heat and notice how far down the throat these sounds come from.

43
Syllable:-
Definition of syllable: a part of a word pronounced with a single uninterrupted sounding.

 bat has one vowel sound and therefore one syllable


 sticky has two vowel sounds and therefore two syllables
 south has one vowel sound—/ow/—and therefore one syllable

Ways to Count Syllables

1. Clap your hands as you say a word.

Try to form a steady beat for every individual sound you make.

For example, ba (clap) - na (clap) -na (clap). This word has three claps, one for every syllable.

Words have a natural emphasis when you say them regularly. The word "banana" sounds like banana
because it stresses the middle syllable. Coordinate your claps with the natural stresses and emphases of a
word.

Start with bigger words that you know have more than one syllable, like "hippopotamus." The more
syllables there are, the longer and more rhythmic your beat will be, making it easier to divide the word.

2. Write a word down on a piece of paper.

Use big letters and leave some room between letters to separate syllables.

Write the word in a blue. You will later be drawing vertical lines to separate syllables, which you can color
in red. This will provide visual contrast and help you picture how words are broken up.

Look at where the vowels fall in your word. Every syllable will have a vowel, but may not always have a
consonant.

3. Identify any prefix in a word.

Prefixes are sets of letters added to the beginning of a word to give it a specific meaning.

44
For example, the word "play" is different than the word "replay." "Play" means to show something once;
"replay" means to show something again.

Many prefixes are usually one syllable, for example rewind, predetermine, and postmortem.

Some prefixes have more than 1 syllable. If your prefix has more than one vowel and the vowels are
separated by consonants, it is more than one syllable. For example, "anti" is a prefix with two syllables.
"Ant" is one syllable and "i" is the other.

Prefixes with more than one vowel that are not separated by consonants most likely are only 1 syllable. For
example, "eu" in "eulogy" is only 1 syllable.

4. Identify any suffix of a word.

Suffixes are like prefixes, but they are added to the end of the word to make them mean something
different.

For example: "calculate" means something different than "calculator. Calculate is when you find the
amount of something; calculator is a tool for helping you find that amount.

Like prefixes, suffixes are usually one syllable but can sometimes be more than 1. Common suffixes include
"s," "ed," and "ing," as in "computers," "computed," and "computing."

Stress:-
When we pronounce a word having more than one syllable, we speak these syllables with a variation in the
breath force. Some syllables of a word are more prominent (pronounced more loudly) while others are
uttered with weak force of breath. The syllables which are uttered with greater force of breath are called
stressed syllables. Thus, we can say that stress is the degree of force used to pronounce a syllable.

Word stress rules

Rule 1 - ’Front weight” in nouns and adjectives.

There seems to be a very strong tendency in English for what is called core vocabulary to have stress on
the first syllable. This means that many common nouns and adjectives will have stress on the first syllable.

 water people
 woman butter
 Finger pretty
 Father mother

Rule 2- Two- and three-syllable words which have a prefix.

In words with prefixes such as ’be-’, ‘in-’, ‘dis-’, ‘ex-’, ’un-’, etc., the stress is at most always on the second
or third syllable, i.e. prefixes are not stressed in English words. Note that the majority of these words are
verbs.
45
Examples: repeat, begin, increase, exhaust, and conclude

Rule 3- Words with suffixes.

If we examine English words with suffixes, a similar tendency is revealed: suffixes are never stressed.

Examples:

-ly quietly

-al original

-ive administrative

-ent/ant equivalent

-ic automatic

Rule 3.1 Certain suffixes determine on which of the other syllables the stress will fall. There are very many
suffixes which cause the syllable before the suffix to be stressed. These are:

-ive (impressive) -iate (deviate)

-ient (incipient) -iary (pecuniary)

-iant (deviant) -iable (negotiable)

-ial (substantial) -ish (diminish)

-ion (invention) -ify (identify)

-ic (geographic) -ium (premium)

-ian (median) -ior (superior)

-ious (infectious) -io (radio)

-ical (economical) -iar (familiar)

-ity (opportunity) -ible (impossible)

It doesn’t matter if the stress was on a different syllable in the form of the word without the suffix
(sometimes called the ’base’ word); the stress will move from wherever it was to the syllable before the
suffix when any of these suffixes are added.

Rule 3.2 The suffix ’-able’ usually does not change the stress pattern of a word to which it is added. So in
’commend’ the stress is on the second syllable, in ‘commendable’ it remains on the second syllable.

Examples:

46
 adapt adaptable
 rely reliable
 knowledge knowledgeable
 detest detestable

Exceptions:

 Demonstrate demonstrable
 admire admirable
 prefer preferable

Rule 3.3 The following suffixes cause the stress to be placed on the fourth syllable from the end of the
word (this applies, of course, only to words of four or more syllables):

-ary (vocabulary)

-ator (investigator)

-mony (alimony)

-acy (intimacy)

-ory (category)

Rule 4

Stress change according to function (use of the word in the sentence)

Different syllables of the same word can be stressed depending on its use in the sentence (that is, as a
noun, adjective, verb, etc.)

Read the following sentences and note the underlined syllables which are stressed in the words.

He speaks perfect English.

She needs to perfect her skills before taking up the job.

His previous criminal record is an obstacle in his future life.

The papers record that the inflation has dropped.

There has been very little progress in this semester.

Their work progresses slowly but steadily.

He was absent from the class last week.

You should not absent yourself from the class.

His friends defended his conduct.

He will conduct the test.

47
They could see every detail in the picture.

He couldn’t detail all the facts.

Note: In the underlined words the pattern of stress changes. For example in the first sentence, we see that
the stress is at the beginning of the word ‘perfect’ but, in the second sentence the stress is on the second
syllable of the same word.

The pattern of stress in words depends upon its use in the sentence. If the word is used as a noun or
adjective the stress is on the first syllable of the word but, if it is used as a verb the stress is on the second
syllable. This can be seen in the words ‘perfect, record and progress, absent, conduct, and detail’ used as
different parts of speech in these sentences.

Stress in compound words

Words composed of two separable words are called compound words. There may or may not be a hyphen
between them. In compound words generally the stress is on the first word or at the beginning.

Read the following compound words and note that the stress in them falls on the first part.

blackbird, blackboard, rain-coat, postman, school-bus, mailbag, footprint, bathroom, lampshade,


sandbox.

There are some compound words in which the stress is on the second part e.g.: herself, whatever,
themselves.

There are some compound words in which the stress is on both the parts of the word e.g.:

|after|noon, |home-|made, |good-|looking, |bad-|tempered.

Stress in Sentences

How can you know which words to stress in a sentence? Again, there are no hard-and-fast sentence stress
rules, but there are some general principles that will help you use stress properly when speaking in English.
You can often tell which words should be stressed based on the parts of speech and where the words fall in
a sentence.

Content words (nouns, adjectives, adverbs, and main verbs) are usually stressed.

Function words (determiners, prepositions, and conjunctions) are usually unstressed unless you want to
emphasize their role(s) in a sentence.

Question words (who, what, when, where, why, and how) are usually unstressed unless you want to
emphasize their role(s) in a sentence.

48
Subject pronouns (I, You, He, She, We, They) are usually unstressed, while object pronouns (me, you, him,
her, us, them) are usually stressed.

 I ran quickly to the desk. (emphasizes who is doing the running)

 I ran quickly to the desk. (emphasizes what action is being done)

 I ran quickly to the desk. (emphasizes the way in which you ran, but does not fundamentally change
the meaning of the sentence)

 I ran quickly to the desk. (inappropriate sense stress, but emphasizes the direction in which you
ran)

 I ran quickly to the desk. (inappropriate sense stress, but emphasizes that it was a specific desk)

 I ran quickly to the desk. (emphasizes the object or location to which you ran)

Sentence Stress in a Question

Like the previous example, the sentence stress here also denotes the natural rise and fall of the word
combination. However, you could still ask this question six different ways to convey six slightly different
meanings:

 Who will come to the party? (you want to know who the party attendees are)

 Who will come to the party? (you want to know who will definitely be attending the party)

 Who will come to the party? (you want to know who will attend the party, but this form does not
change the standard meaning of the question)

 Who will come to the party? (inappropriate sense stress, but emphasizes the location of the party)

 Who will come to the party? (inappropriate sense stress, but emphasizes which party you’re talking
about)

 Who will come to the party? (you want to emphasize the party, possibly in contrast to a separate
event)

Intonation:-
Intonation refers to pattern of pitch in a sentence. When we hear someone speaking we realize that he does
not speak on the same note. We notice variation in the pitch of his voice. Sometimes his pitch rises and at

49
other times it falls. This pattern of variation in the pitch of voice is called intonation. There are mainly two types
of intonation patterns followed in English language. These are:

a. Falling tone: indicated by the symbol

b. Rising tone: indicated by the symbol

Falling intonation pattern


The tone in these sentences usually falls before the last word/segment of utterance. This is used:

- In ordinary statements:
 It’s seven o’ clock.
 The milkman hasn’t come. I didn’t do it.
 The water is cold.
 The postman was looking for you.
- In questions beginning with the question words — what, when, where, how etc. — when asking for
information in a matter-of-fact manner.

 Why did you do it?


 Where are they going?
 How do they come here?
 When are they shifting?
 What’s the time?
- In commands
 |
Do as I say.
 |
Shut the door.
 |
Go away.
 |
Sit down.
 |
Bring a „glass of water.
- In tag questions
 It’s a lovely day, isn’t it?
 You can swim well,
can’t you?

 He has a lot of books,


hasn’t he?

 He didn’t come, did he?


 You’re there, aren’t you?
 This is the one you asked for,
Isn’t it?
50
 You’re not going to play,
are you?

 Tomorrow is |Monday, isn’t it?

Rising intonation patterns


Here the voice rises towards the end of the sentence. It is used in the following cases:

- Incomplete utterances
 It’s seven o’ clock (but the milkman hasn’t come yet)
 The house is empty (for many years in the past)
 When I reached there (he had left)
- In questions which demand an answer in ‘yes’ or ‘no’

 Are|they going?
 Will he be there?
 Have they done it?
 Is she studying?
 Has he started?
 Have you heard the news?
 Do you know?
 May I try?

- In questions which begin with the question words but said in a polite or friendly manner.

It is also used when we politely ask a person to repeat the information which we have not clearly heard or
understood, e.g.

 A: They are coming from Delhi tomorrow.


 B: When are they coming? (Asking for repetition)
 How is your sister?
 Why didn’t you come to the party?
 What is wrong with you?
 When’re you coming?
- In polite requests:
 Would you open the door?
 May I have your pen?
 Would you mind |giving me your pencil?
 Will you sit down?
 Please sit quietly.
- In subordinating clauses (but falling tone is used in the main clauses as shown below).
 When he came I asked him to wait.
51
 In spite of the rain they all came as they had promised.
 If you like I will send the car for you.
 Since you re fuse help I must do it all alone .
 When you are ready we can go.

Cecily Berry Vowels Drill: Speak these words loudly:

OOT OHT AWT AHT AYT EET EHT

OOD OHD AWD AHD AYD EED EHD


OON OHN AWN AHN AYN EEN EHN

OOB OHB AWB AHB AYB EEB EHB


OOP OHP AWP AHP AYP EEP EHP

OOM OHM AWM AHM AYM EEM EHM

OOG OHG AWG AHG AYG EEG EHG

OOF OHF AWF AHF AYF EEF EHF

OOV OHV AWV AHV AYV EEV EHV

OOS OHS AWS AHS AYS EES EHS

OOZ OHZ AWZ AHZ AYZ EEZ EHZ

OOTH OHTH AWTH AHTH AYTH EETH EHTH

CECILY BERRY DRILL-EXERCISE FOR MOUTH MUSCLES

OOKH OHKH AWKH AHKHT AYKHT EEKHT


T T T
OOGD OHGD AWGD AHGD AYGD EEGD

OOPT OHPT AWPT AHP AYPT EEPT


T
OOBD OHBD AWBD AHBD AYBD EEBD

OOM OHMD AWM AHMD AYMD EEMD


D D
OOND OHND AWN AHND AYND EEND
D
OOLT OHLT AWLT AHLT AYLT EELT

OOLD OHLD AWL AHLD AYLD EELD

52
D

OOLZ OHLZ AWLZ AHLZ AYLZ EELZ

OOTH OHTH AWT AHTHT AYTHT EETHT


T T HT
OOTH OHTH AWT AHTHD AYTHD EETHD
D D HD
OOST OHST AWST AHST AYST EEST

OOST OHSTS AWST AHSTS AYSTS EESTS


S S
OOZD OHZD AWZ AHZD AYZD EEZD
D
OOFT OHFT AWFT AHFT AYFT EEFT

OOFT OHFT AWFT AHFTS AYFTS EEFTS


S S S
OOVD OHVD AWV AHVD AYVD EEVD
D
OOKS OHKS AWKS AHKST AYKST EEKST
T T T
OOTT OHTT AWT AHTTH AYTTH EETTH
H H H
CECILY BERRY DRILL-CONSONANT SOUND PRACTICE
BOO BOH BAW BAH BAY BEE BIE

COO COH CAW CAH CAY CEE CIE

DOO DOH DAW DAH DAY DEE DIE

FOO FOH FAW FAH FAY FEE FIE

GOO GOH GAW GAH GAY GEE GIE

HOO HOH HAW HAH HAY HEE HIE

JOO JOH JAW JAH JAY JEE JIE

KOO KOH KAW KAH KAY KEE KIE

LOO LOH LAW LAH LAY LEE LIE

MOO MOH MAW MAH MAY MEE MIE

NOO NOH NAW NAH NAY NEE NIE

POO POH PAW PAH PAY PEE PIE

ROO ROH RAW RAH RAY REE RIE

53
SOO SOH SAW SAH SAY SEE SIE

TOO TOH TAW TAH TAY TEE TIE

VOO VOH VAW VAH VAY VEE VIE

WOO WOH WAH WAH WAY WEE WIE

YOO YOH YAW YAH YAY YEE YIE

ZOO ZOH ZAW ZAH ZAY ZEE ZIE

54
Tutorial
Q. Treasure hunt
Help Reena to find the hidden treasure. You can move along the squares with words containing the /z/ (zip,
zoo, Pins) sound only.

Zest Jest Page Geography Join

Buzz Zoo Raise Jump Forge

Jew Wage Gaze Cage Noise

Rage Bars Reason Joy Freeze

Change Increase Biology Zebra Rise

Region Boys Engineer Huge Magic

Jealous Ways Zealous Cheese Plans

55
Exercise 1
Q1 Which of these is NOT a place where the constrictions and obstructions of air occur.
A. Glottal
B. Larynx
C. Velar
D. post-alveolar
Q2. Some ________ are made as a result of the lips and tongue altering the airstream.
A. Vowels
B. sounds
C. consonants
D. places of articulation
Q3. Labiodental consonants occur when you block airflow out of the mouth by bringing your lips together.
A. True
B. false
Q4. Which of these consonants is NOT bilabial
A. /p/
B. /b/
C. /f/
D. /m/
Q5. The post-alveolar consonants are produced where your teeth meet your gums.
A. true
B. false

Exercise 2

Q1. How many syllables in tail?

A. 2
B. 1
C. 3
D. 4

Q2. How many syllables in environment?

A. 3
B. 4
C. 5
D. 6

Q3. How many syllables in water?

A. 1
B. 3
C. 4
D. 2
56
Q4. How many syllables are there in the word GODLINESS?

A. One syllable.
B. Two syllables.
C. Three syllables.
D. Four syllables.

Q5. How many syllables are there in the word INCAPACITY?

A. Two syllables.
B. Three syllables.
C. Four syllables.
D. Five syllables.

Q6. How many syllables are there in the word EXTERNAL?

A. One syllable.
B. Two syllables.
C. Three syllables.
D. Four syllables.

Q7. How many syllables are there in the word INCIDENT?

A. Two syllables.
B. Three syllables.
C. Four syllables.
D. Five syllables.

Q8. How many syllables are there in the word EVOLUTION?

A. Two syllables.
B. Three syllables.
C. Four syllables.
D. Five syllables.

Q9. How many syllables are there in the word DANGEROUS?

A. One syllable.
B. Two syllables.
C. Three syllables.
D. Four syllables.

Q10. How many syllables are there in the word INCREDIBLE?

A. Three syllables.
B. Four syllables.
C. Five syllables.
D. Six syllables.

57
Exercise 3

Look and read the words then tick the answer with the correct stress.
1. really
A: really
B: really
C: really
2. report
A: report
B: report
3. remember
A: remember
B: remember
C: remember
4. yesterday
A: yesterday
B: yesterday
C: yesterday
5. exam
A: exam
B: exam
6. revision
A: revision
B: revision
C: revision
7. difficult
A: difficult
B: difficult
C: difficult
8. opinion
A: opinion
B: opinion
C: opinion
9. possible
A: possible
B: possible
C: possible
10. business

A: business

B: business

Exercise 4

For each of the six questions choose the one correct answer.
58
1. Which syllable is stressed in the word 'interesting'?

a) interesting

b) interesting

c) interesting

d) interesting

2. Which syllable is stressed in the word 'photographer'?

a) photographer

b) photographer

c) photographer

d) photographer

3. Which syllable is stressed in the word 'development'?

a) development

b) development

c) development

d) development

4. Which syllable is stressed in the word 'ordinary'?

a) ordinary

b) ordinary

c) ordinary

d) ordinary

5. Which syllable is stressed in the word 'vegetable'?

a) vegetable

b) vegetable

c) vegetable

d) vegetable

6. Which syllable is stressed in the word 'unfortunately'?

a) unfortunately

b) unfortunately
59
c) unfortunately

d) unfortunately

Exercise 5

Look at the words below and write them in two groups – content and function words:

Exercise 6

Q1. Choose the sentence stress that best answers the following question:

Who's going to the supermarket?

A. He's GOING to the supermarket.


B. HE'S going to the supermarket.
C. He's going to the SUPERMARKET.

Q2. Choose the sentence stress that best answers the following question:

Where is he going?

A. He's going to the SUPERMARKET.


B. HE'S going to the supermarket.
C. He's GOING to the supermarket.

Q3. Choose the sentence stress that best answers the following question:

What is she going to buy?


60
A. She's going to BUY some bread.
B. SHE'S going to buy some bread.
C. She's going to buy some BREAD.
D. SHE'S GOING TO BUY SOME BREAD.

Q4. Choose the sentence stress that best answers the following question:

How often does he practice English?

A. He studies English EVERY DAY.


B. HE studies English every day.
C. He studies ENGLISH every day.
D. He STUDIES English every day.

Q5. Choose the sentence stress that best answers the following question:

What does Tom practice every day?

A. TOM studies English every day.


B. Tom STUDIES English every day.
C. Tom studies ENGLISH every day.
D. Tom studies English EVERY DAY.

Q6. You are walking along a shopping street with a friend. You stop in front of a shop window and look at
the display there. You see a multi-coloured sweater on sale and tell your friend you want to go in and buy
it. Your friend replies "You want that?"

If your friend stressed the word you, you might understand that:

A. He has certain opinions about you and you are surprising him now
B. He thinks that he actually wants it
C. He thinks the salesperson wants it
D. He thinks the sweater is ugly

Q7. You are walking along a shopping street with a friend. You stop in front of a shop window and look at
the display there. You see a multi-coloured sweater on sale and tell your friend you want to go in and buy
it. Your friend replies "You want that?"

If he stressed the word want, you might understand that:

A. He thinks the sweater won’t fit you


B. He thinks the sweater isn’t warm enough
C. He thinks you have strange taste
D. He thinks that even on sale, it’s not a good price

Q8. You are walking along a shopping street with a friend. You stop in front of a shop window and look at
the display there. You see a multi-coloured sweater on sale and tell your friend you want to go in and buy
it. Your friend replies "You want that?"

61
If he stressed the word that, you might understand that:

A. He thinks you should buy the sweater


B. He thinks the sweater is ugly
C. He thinks colours are out of style
D. He isn’t sure what you’re talking about

Q9. You go to a party where your friend Lisa is bringing her new boyfriend. You and your friend Kathy see
him, and Kathy says, "That’s Lisa’s boyfriend?"

If she stressed the word that's, you might understand that:

A. Kathy thought he was Lisa's brother


B. Kathy thought another guy was Lisa's boyfriend
C. Kathy had a different image of Lisa's boyfriend in her mind
D. Kathy forgot her glasses at home

Q10. You go to a party where your friend Lisa is bringing her new boyfriend. You and your friend Kathy see him, and
Kathy says, "That’s Lisa’s boyfriend?"

If she stressed the word Lisa's, you might think that:

A. Kathy thought he was your boyfriend


B. Kathy doesn’t think he is Lisa's type
C. Kathy doesn’t remember who Lisa is
D. Kathy is jealous of Lisa

Exercise 7

Q1. What is the intonation marker for a question in English?

A. Rising pitch towards the end of the sentence.


B. Falling pitch towards the end of the sentence.
C. Completely flat pitch throughout the sentence.
D. It depends on the relative social statuses of the speakers.

Q2. You and your friend are having a conversation in English. Your friend asks you:

Do you want a new sweater for your birthday?

You respond:

No, I want a new scarf for my birthday.

In your sentence, which word will have a change in intonation to show that it is the focus of the sentence?

A. No
B. Want
C. Scarf
D. My
E. Birthday
62
Q3. Which of the following patterns of intonation is used in commands_____________?

A. The falling
B. The rising
C. The fall__rise
D. None of the above

Q4. Which of the following patterns of intonation, is used in W.H. questions asked in a warm and friendly way
indicating extra politeness ?

A. The falling
B. The rising
C. The fall__rise
D. None of the above

Q5. Which of the following patterns of intonation is used in declarative sentences uttered as ordinary statements ?

A. The falling
B. The rising
C. The fall__rise
D. None of the above

Practice intonation

Intonation Exercise #1: Yeah

First, we’re going to start with the word “yeah.” (It’s a little more common in everyday speech than the
word “yes.”)

Yeah (neutral)

Yeah. (falling)

Yeah? (rising)

Yeah! (enthusiastic)

Yeah!! (excited)

Yeah!!! (super excited)

Yeah?? (uncertain)

Yeah??? (surprise)

Yeah… (disinterested)

Please note that there are more emotions you can show here, like annoyance, doubt, disbelief, or
uncertainty.

Intonation Exercise #2: No

63
Next, let’s move on to “no.”

No (neutral)

No. (falling)

No? (rising)

No! (authoritative)

No!! (forceful)

No?? (uncertain)

No?? (disbelief)

Intonation Exercise #3: Okay

Next, let’s move on to the word “okay.” As in many languages, we use this all the time.

Let’s practice different emotions you can show through your intonation on the word “okay.”

Okay (neutral)

Okay. (falling)

Okay. (agreeing)

Okay? (rising)

Okay! (annoyed)

Okay!! (irritated)

Okay?? (uncertain)

Okay??? (insecure)

As you’re probably noticing as I go through the examples, your facial expressions and body language can
also help you produce the right emotion.

Intonation Exercise #4: Right

Next, let’s move on to “right.”

Right (neutral)

Right. (falling)

Right? (rising)

Right! (authoritative)

Right!! (aggressive)

64
Right?? (uncertain)

Right??? (insecure)

Intonation Exercise #5: Hey

Next, let’s move on to the word “hey,” another word we use all the time in everyday speech.

Hey (neutral)

Hey. (falling)

Hey. (disinterested)

Hey! (authoritative)

Hey!! (aggressive)

Hey? (rising)

Hey! (enthusiastic)

Hey!! (excited)

Intonation Exercise #6: Maybe

Moving on, let’s talk about the word “maybe.”

Maybe (neutral)

Maybe. (falling)

Maybe. (unconvinced)

Maybe? (rising)

Maybe?? (uncertain)

Maybe… (noncommittal)

Intonation Exercise #7: Please

Last but not least, let’s talk about the word “please.”

Please (neutral)

Please. (falling)

Please! (authoritative)

Please!! (forceful)

Please? (rising)

Please?? (pleading)
65
Please??? (begging)

66
UNIT -3

Vocabulary
Cloze Test – Introduction to the Concept
What is a Cloze Test?

A Cloze test is an amalgamation of ‘comprehension’ and ‘fill in the blanks’ where your reading, analysing
and vocabulary skills are all tested at once. A comprehension or a passage is given in the question which
comprises blanks. Candidates need to choose the most appropriate word from the given options, which
can most suitably be placed in the blank.

There are various benefits for a candidate if a cloze test is asked in the question paper. This is because to
answer these questions, it does not take anything more than 30 seconds and anybody who has a good
command over Grammar can easily score high for this topic, thereby increasing the overall scores.

There are two major patterns based on which a cloze test is asked in the examinations:

Basic Fill in the Blanks Format: The simplest format for the cloze test is that a passage is given with
multiple blanks and candidates need to choose one correct answer from the given options, which would be
appropriate and logically fill the blank.

Replacing words: With the increased competition in the various exams, a new pattern for cloze test
frequently appears in the examination. Instead of giving blanks, multiple words are given in bold in the
passage and candidates need to answer if the word given in bold is appropriate or if it can be replaced with
another word from the given options.

In either of the two cases, candidates need to have good command over vocabulary and grammar as only
then can they answer these questions easily and efficiently.

Tips and Tricks to Solve the Cloze Test

The exam pattern for the various competitive exams may be different but the syllabus under each subject
is the same. So, candidates who are preparing for any exam must be aware that questions from the cloze
test may be asked in the examination.

To ensure that you do not lose marks in the cloze test passage, given below are a few tips to solve the test-
based questions:

Overview of the Passage: Take a quick glance at the passage to know what the topic deals with as this will
help you apprehend which of the given options most suitably fits in the blank.

Look for Contextually Correct Options: The options provided under cloze test may seem to be similar in
meaning, which makes selecting the appropriate option even tougher. Thus, look for an option which aptly
fits with the context of the passage.

67
Eliminate Options in case confused: If you are unable to reach an answer, try eliminating the options
which seem to be incorrect. This will lessen your options and may help to arrive at the correct answer
easily.

Link the Sentences while answering: The cloze test is a passage based on a particular topic. So, if you are
unable to find the answer, try linking the sentences with each other to simplify the context of the passage.

Understand the tone of the passage: Once a candidate understands the tone of the passage, selecting the
appropriate option becomes easier.

Sentence synonyms
A synonym is a word or phrase, which has exactly or nearly the same meaning as another word or phrase
in the same language, e.g., Happy is a synonym of glad, joyful, etc. Such words which have the same or
related meaning are known as synonyms of one another.

Sentence synonyms are type of vocabulary questions which often asked in competitive and placement
exams. In these types of questions you will be given a sentence and one word would be underline. You
have to find synonym of the word in given option and replace it with that without changing the meaning of
the sentence.

Example

Q. In the following sentence replace the underlined word with its synoynm.

The prince abdicated the crown and returned to his castle.

A. Gave up.
B. Sold.
C. Reinvested into.
D. Auctioned.

Correct Answer – A. gave up

Explanation

To abdicate means to relinquish or give up a throne, power, right, or responsibility. It does not mean to sell
(B), reinvest in (C), or auction (D).

Tips and Tricks to Solve Sentence synonyms

1. Reread the sentences surrounding the word they’re asking about. Understand the contextual
meaning of the word.
2. Restate the sentence, inserting your own word(s). Don’t worry about the answer choices at this
point. Just find a word or a phrase that makes sense.
3. Locate an answer choice that fits your definition. Suppose you came up with decorate for this step.
In the five answer choices, you see beautify, which matches your definition well enough.
4. Insert your answer choice into the sentence. If it fits, you’re done. If not, try another word.

68
Image
Images as a part of visual aids are used as a technique in teaching English Vocabulary. Pictures can be used
to explain the meaning of vocabulary items. In teaching and learning process, the role of using Images or
pictures as media has become the best choice in teaching English especially in vocabulary. Pictures are
important to help the students in learning English vocabulary and memorizing the meanings.

In this semester we will learn vocabulary by using Images.

Q. Four Pictures, One word:-

In question below you have given four pictures and Count of letter. You need to take pictures as a clue to
come up with the correct word. The word must have 8 letters.

Answer and explanation –

Look at the picture, what is the first thing comes to your mind. Family ,maybe But we know that we need
to give the answer in eight letter word. As the first picture is showing family laughing, others are telling us
that something is going on outdoors. Looking at second picture of people skinning, third people on
mountain and lastly a couple looking at map, it becomes quite evident that the answer is VACATION

Apply the meaning to a real life situation


Don’t settle after you learn a new word by reading it or looking it up in the dictionary: these are good starts,
but it’s by using the new words that you truly commit them to your long-term memory.

Be creative and try to use your newly learned words in as many ways as possible:

 Write them down

 Say them aloud

 Create sentences with them, mentally or in writing


69
 Try to use them in a conversation

 Discuss them with friends

It’s also important to be aware of your own language style: every time you catch yourself saying common
or nonspecific words such as “nice,” try coming up with richer and more precise expressions instead.

Following are some of examples of the words divided into themes of real life situation

Religious Words:-
1 Cardinal

Meaning- Cardinal means of primary importance, fundamental.

Example - Most cultures consider gambling a cardinal sin and thus have outlawed its practice.

2.Syncretic

Meaning - Syncretic-more generally speaking-can refer to any amalgam of different schools of thought.

Example- I have created a syncretic religion, one that combines elements of different religion.

3. Parochial

Meaning- This word comes from parish, a small district, usually located in the country.

To be parochial is to be narrow-minded in one's view.

Example - Since the professor tends to be parochial, he is often unwilling to listen to theories other than
his own.

4. Iconoclast

Meaning - This is an interesting word. i.e. an iconoclast is one who destroys religious images. An
iconoclast- more broadly is somebody who attacks cherished beliefs or institutions.

Example - The church leader was deemed an iconoclast when he spoke out against his religion’s views.

5. Apostasy

Meaning- Some believers turn against their faith and renounce it. We call this act apostasy, and those who
commit it, apostates. Today the word carries a slightly broader connotation in that it can apply to politics
as well.

Example - After a short apostasy, during which he supported Henry Raspe, landgrave of Thuringia, Conrad
returned to the side of the Hohenstaufen and aided Conrad IV.

Political scandals :-
1.Malfeasance

70
Meaning- Malfeasances wrongdoing, usually by a public official. Oftentimes, you hear the term corporate
malfeasance- this type of wrongdoing occurs when somebody in the business world is up to no good.
Though, malfeasance is used in the context of politics.

Example - The government wants to draw a line under the glut of financial malfeasance.

2. Embroiled

Meaning- To become caught up in a scandal is to become embroiled in it.

Example -In the last couple of months, a few well-known politicians have become embroiled in scandals.
From the verb embroiled, we get the noun imbroglio. which is an embarrassing, confusing situation.

3. Prevaricate

Meaning- If you've ever seen a politician caught in a lie and that person is trying to wiggle their way out of
a pointed question, he (or she) is prevaricating.

Example- In order to get his bill passed, the politician went out of his way to prevaricate about the release
of the environmental study.

4. Lascivious

Meaning- Lascivious, like lecherous, prurient, and libidinous, all refer to perversion.

Example- The lascivious person was put in prison.

5. Turpitude

Meaning- A synonym for depravity, turpitude is only reserved for those acts deemed to be downright
wicked and immoral.

Example- He was considered unfit to hold office because of moral turpitude.

Preposterous Prepositions:-
1. Untoward

Meaning- You may think that untoward has something to do with a direction. Untoward is an adjective
meaning not favourable, inconvenient. Synonym for untoward is inauspicious.

Example- Unless anything untoward happens we should arrive just before midday.

2. Upbraid

Meaning- Upbraid means to scold or berate, a meaning it shares with many other words: reprimand,
reproach, chide, and castigate.

Example- The police officer did not hesitate to upbraid the man for driving without insurance.

3. Underwrite

71
Meaning- If you are writing below the margins of a paper you are not underwriting-you are simply writing
below the margins of a piece of paper. Underwrite means to support financially.

Example- He then offered to underwrite all of my costs for July

4. Overweening

Meaning- To be overweening is to be presumptuously arrogant.

Overweening can also refer to ideas/ opinions/ appetites that are excessive or immoderate.

Example- The more we nourish widespread ambition, the less we have to fear the overweening power of
mild despotism.

Money
1. Profligate

Meaning- This word means spending recklessly almost to the point of immorality. This word often pops up
in politics, when some charge that government is spending wastefully. Profligate is also a person known for
his or her profligacy.

Example- She is well-known for her profligate spending habits.

2. Prodigal

Meaning- To be prodigal is to squander or waste wealth (it doesn't necessarily have to be familial wealth).
This word should not be confused with prodigious, which means vast or immense.

Example- His eldest brother being a prodigal he succeeded to the paternal estate, but threw the will into
the fire on his brother's promising to reform.

3. Avarice

Meaning - One of the seven deadly sins, avarice means greed. Of note, this word doesn't necessarily mean
greed for food but usually pertains to possessions or wealth.

Example- The chief ingredient in the making of a criminal is avarice.

4. Sybarite

Meaning - This is a person who indulges in luxury. And though the word doesn't directly relate to wealth,
most of the times a sybarite have to be wealthy.

Example- Someone who has a sybaritic way of life spends a lot of time relaxing in a luxurious way.

5. Impecunious

Meaning - The word pecuniary means relating to money. Impecunious, on the other hand, means not
having any money.

72
Example- Well-born but impecunious younger brothers kidnap heiresses and roguishly attempt to
persuade them into matrimony.

Tutorial
EXERCISE 1- CLOZE TEST

Read the following information carefully and answer the questions given below:

Four of the most popular forms ie Facebook, Twitter, Snapchat, Instagram of social media ___1____ great
damage to young people’s mental health. These four platforms have a negative effect because they
can____2____ children’s and young people’s body image worries, and worsen bullying, sleep problems,
anxiety, depression, and loneliness. The findings follow growing ____3____among politicians, health
bodies, doctors, and parents about young people suffering harm as a result of cyber bullying and social
media ____4____ feelings of self-loathing and leading them to commit suicide. However, the leader of the
UK’s psychiatrists said these findings were too simplistic and they ____5____ blamed social media for the
complex reasons why the mental health of so many young people is suffering

Q1. Mark the correct option for blank _(1)_.

(A) desecrate

(B) impious

(C) inflict

(D) perpetrating

(E) None of the above

Q2. Mark the correct option for blank _(2)_.

(A) mitigate

(B) exacerbate

(C) alleviate

(D) effectuate

(E) None of the above

Q3. Mark the correct option for blank _(3)_.

(A) bridle

(B) consternate

(C) poise

73
(D) concern

Q4.Mark the correct option for blank _(4)_.

(A) reinforcing

(B) ebbing

(C) augment

(D) accentuate

(E) None of the above

Q5. Mark the correct option for blank _(5)_.

(A) legitimately

(B) outlandish

(C) unfairly

(D) impartially

(E) None of the above

In the following passage, some of the words have been left out, each of which is indicated by a letter.
Find the suitable word from the options given against each letter and fill up the blanks with appropriate
words to make the paragraph meaningful.

This year, the world’s largest democracy, India, and the biggest country by _____ 6 _____ , Russia, are
celebrating the 70th anniversary of establishment of diplomatic relations between them. Russia continues
to be among India’s major politico-diplomatic and defence partner nations. While India has _____ 7 _____
separate strategic partnership pacts with more than two dozen countries, the Indian and Russian
governments in December 2010 _____ 8 _____ their bilateral ‘Strategic Partnership’ to what they termed a
“Special and Privileged Strategic Partnership.” The New Delhi-based _____ 9 _____ ‘Foundation for
National Security Research’, which did a comparative assessment of India’s strategic partnerships —
meaning, ‘political-diplomatic, defence and economic cooperation’ (during the 10-year period prior to
November 2011), had said, “Russia emerges as the most important strategic partner of India (followed by
the U.S., France, the U.K., Germany and Japan in that order).” The November 2011 report had found that
Russia had provided strong political and diplomatic support to India and helped enormously in building
India’s defence capability. However, it warned that the “economic content of the (India-Russia) partnership
is extremely weak, ” and recommended that “urgent and _____ 10 _____ steps need to be taken to
improve economic relations if this (India-Russia) partnership is to be sustained and made durable.”

Q6. Mark the correct option for blank (6)

A. Population B. Density C. Area D. Democracy E. Economy

74
Q7. Mark the correct option for blank (7)

A. Inked B. Considered C. Contemplated D. Refuted E. Revoked

Q8. Mark the correct option for blank (9)

A. Called B. Elevated C. Refreshed D. Nullified E. Revived

Q9. Mark the correct option for blank (9)

A. NGO B. Startup C. Personnel D. Think tank E. Avenue

Q10. Mark the correct option for blank (10)

A. Various B. Precarious C. Vigorous D. Minuscule E. Exhilarating

Directions (11-15): In the following passage, some of the words have been left out, each of which is
indicated by a letter. Find the suitable word from the options given against each letter and fill up the
blanks with appropriate words to make the paragraph meaningful.

The blockade of the national highways leading to the Manipur valley, called by the United Naga Council
(UNC), has been in place since November 1. This has severely affected life in the State, with shortages and
_____ 11 _____ costs of essential supplies such as fuel and food, even as demonetisation has _____ 12
_____ problems. Blockades like this are not new to Manipur. In 2011, there was initially a hundred-dayplus
blockade enforced by Kuki-led groups, and countered later by Naga groups, which together had a _____ 13
_____ effect on life in Manipur. This time the blockade is in place to oppose the creation of new districts by
the Okram Ibobi Singh government. On December 9 it issued a _____ 14 _____ notification for the creation
of seven new districts by bifurcating seven (of a total of nine) districts. This decision had as much to do
with long-pending demands — in particular, for a new Kuki-majority district to be carved out of the larger
Senapati hill district — as with easing administrative access to _____ 15 _____ areas from the district
headquarters. 11. A. escalating B. upfront C. diminishing D. varying E. considerable

Q11. Mark the correct option for blank (11)

A. escalating B. upfront C. diminishing D. varying E. considerable

Q12. Mark the correct option for blank (12)

A. exceed B. pacified C. exacerbated D. vindicated E. exemplified

Q13. Mark the correct option for blank (13)

A. asting B. counter C. mounting D. befitting E. debilitating

Q14. Mark the correct option for blank (14)

A. special B. live C. gazette D. detail E. retentive

Q15. Mark the correct option for blank (15)

A. offshore B. far-flung C. outskirts D. remotely E. secluded

75
Directions (16-20): In the following passage, some of the words have been left out, each of which is
indicated by a letter. Find the suitable word from the options given against each letter and fill up the
blanks with appropriate words to make the paragraph meaningful.

The BCCI and its State associations will probably _____ 16 _____ the day the Indian Premier League (IPL)
spot-fixing scandal began in 2013, but in all honesty, there have been so many opportunities for them to
stem the rot that it is difficult to pinpoint how it all went so wrong. Increasingly, the BCCI has become
_____ 17 _____ and waged a battle that has seemed strategically unsound. There was a major disconnect
between how far the Board really believed the court would go and the ground reality. What may also have
_____ 18 _____ the BCCI’s notice is the changing perception of governance in sport and not just in India.
The Federation Internationale de Football Association (FIFA) has seen an _____ 19 _____ overhaul
recently, and in India, the Sports Ministry’s vocal _____ 20 _____ of the Indian Olympic Association’s
controversial appointments is a case in point.

Q16. Mark the correct option for blank (16)

A. remember B. recall C. justified D. rue E. rejoice

Q17. Mark the correct option for blank (17)

A. victim B. isolated C. superpower D. unparalled E. invincible

Q18. Mark the correct option for blank (18)

A. indicated B. caught C. escaped D. come E. incurred

Q19. Mark the correct option for blank (19)

A. rigorous B. reverse C. inward D. unending E. invasive

Q20. Mark the correct option for blank (20)

A. chastising B. appreciation C. criticised D. espressions E. nagging

EXERCISE 2- SENTENCE SYNONYMS

Q1. Read the following sentence and replace the underlined word with its synonym.

The convicted criminal absconded prior to the sentencing phase of the trial.

A. Touched the jury.


B. Reported immediately.
C. Left after discussion.
D. Departed secretly.

Q2. Read the following sentence and replace the underlined word with its synonym.

The aural component of balance is critical for postural control during ambulation.

76
A. Eye.
B. Ear.
C. Nose.
D. Mouth.

Q3. Read the following sentence and replace the underlined word with its synonym.

The old man was benevolent with his fortune.

A. Secretive.
B. Stingy.
C. Kind.
D. Careful.

Q4. Read the following sentence and replace the underlined word with its synonym.

The extra dirt was a key buttress to the foundation.

A. Limiting factor.
B. Support.
C. Overwhelming condition.
D. Obstacle.

Q5. Read the following sentence and replace the underlined word with its synonym.

The cathode of a battery was removed.

A. Positive pole.

B. Negative pole.

C. Neutral pole.

D. Opposite pole.

Q6. Read the following sentence and replace the underlined word with its synonym.

The doctor was known as a charlatan over the years of his practice.

A. Quack.
B. Knowledgeable physician.
C. Procedural physician.
D. Medical examiner.

Q7. Read the following sentence and replace the underlined word with its synonym.

The wound exhibited signs of copious drainage requiring medical intervention.

A. Minimal.
B. Clear.
C. Maximal.
77
D. Foul.

Q8. Read the following sentence and replace the underlined word with its synonym.

The attorney accused the witness of defaming the defendant.

A. Killing.
B. Badgering.
C. Suffocating.
D. Slandering.

Q9. Read the following sentence and replace the underlined word with its synonym.

The detective was able to derive the facts of the case.

A. Desist.
B. Deter.
C. Devise.
D. Deduce

Q10. Read the following sentence and replace the underlined word with its synonym.

The scientist was able to evoke powerful emotions from her audience.

A. Sell.
B. Calm.
C. Call forth.
D. Exaggerate.

Q11. Read the following sentence and replace the underlined word with its synonym.

The judge was fallible during deliberation.

A. Careful not to err.


B. Falsely accused.
C. Loyal to his supporters.
D. Capable of mistakes.

Q12. Read the following sentence and replace the underlined word with its synonym.

The chemist collected the germane data during the experiment.

A. Relevant.
B. Obscure.
C. Limited.
D. Usual.

Q13. Read the following sentence and replace the underlined word with its synonym.

The desperados held up in a grotto in New Mexico during the escape.

78
A. Large cave.
B. Small cavern.
C. Hotel.
D. Motel.

Q14. Read the following sentence and replace the underlined word with its synonym.

The official exhibited a heedless attitude when dealing with the dignitaries.

A. Thoughtless.
B. Pleasant.
C. Friendly.
D. Bitter.

Q15. Read the following sentence and replace the underlined word with its synonym.

The Sherman tank commander noted innumerable troops moving forward against his position.

A. Limited.
B. Weary.
C. Countless.
D. Harmless.

Exercise 3- IMAGE

Q1. Look at the given for picture and write the word associated with it. 8 letter word

________

Q2. Look at the given for picture and write the word associated with it. 8 letter word

79
________

Q3. Look at the given for picture and write the word associated with it. 8 letter word

________

Q4. Look at the given for picture and write the word associated with it. 8 letter word

________

Q5. Look at the given for picture and write the word associated with it. 8 letter word

80
________

Q6. Look at the given for picture and write the word associated with it. 9 letter word

_________

Q7. Look at the given for picture and write the word associated with it. 9 letter word

_________

Q8. Look at the given for picture and write the word associated with it. 9 letter word

81
_________

Q9. Look at the given for picture and write the word associated with it. 9 letter word

_________

Q10. Look at the given for picture and write the word associated with it. 9 letter word

EXERCISE 4- Apply the meaning to a real life situation

Q1. ___________ is the process by which a group adjusts to living within a dominant culture while at the
same time maintaining its original cultural identity.

A. Assimilation
B. Acculturation
C. Syncretism
D. Cultural pluralism

Q2. One who destroys images or attacks popular beliefs?

A. imagist
B. misanthropist
C. iconoclast
D. masochist
82
Q3. Downright wicked and immoral behaviour is :

A. misdemeanor
B. moral turpitude
C. felony
D. petty offense

Q4. Impecunious means

A. Wealthy
B. Cautious
C. Hungry
D. Tardy

Q5. One trait that dominates a personality so much that it influences nearly everything a person does is a:

A. Global Trait
B. Cardinal Trait
C. Specific trait
D. Central Trait

Q6. Infosys is ____________in another controversy after a group said that the company was indulging in
“unethical practices” to boost short term revenues and profits.

A. Consolidated
B. Embroiled
C. Revived
D. Disinvesting
E. Engrossed

Q7. Synonym of PREVARICATE is _____________?

A. Truth
B. Early
C. Lie
D. Delay

Q8. Antonym of LASCIVIOUS is _____________?

A. fluid
B. stable
C. devout
D. chaste

83
Email Writing

Writing a Formal Email


In the information age, email has become the dominant form of communication. Being able to write a
polished, professional email is now a critical skill both in college and the workplace. Below are some key
distinctions between formal and informal writing, as well as some guidelines to follow when composing a
formal email to a superior (professor, current or prospective employer, etc.) or someone who does not
know you. Informal vs. Formal

Informal:

• Written to friends and family

• Accuracy and grammar (spelling and punctuation) are not important

• You can make up your own rules

Example:

Hi Anne, I miss you so much! Can’t wait to see you on Friday!! We haven’t hung out in so long! I miss my
bestie! Maybe we can go to the movies or dinner or just chill and watch TV and catch up…idc, whichever
you want. Love ya, Jules

Formal:

• Written to a professor, colleague, boss, etc.

• Must always be professional

• Accurate grammar, punctuation, and spelling necessary

Example:

Dear Professor Johnson,

I was unable to attend class today due to a doctor’s appointment. When you have a moment, could please
let me know what I missed and what homework I need to have completed for Friday?

Thank you, Julia Smith

Structural components of e-mail


The basic elements of professional email writing:

1. Subject line

84
2. Salutation

3. Email opening

4. Email body

5. Email ending

6. Email Sign off

7. Email signature/footer

1. Subject lines

Your subject line will be the single most important element in your formal email writing. It is the first thing
your recipient will see and, unless you convince her then and there that your email is safe, relevant and
high priority (in that order) it may never be opened. If this happens, any effort you put into the rest of the
email elements will go to waste.

Your subject line will depend on the purpose or content of your email, but overall, you want it to be
something engaging enough for a recipient to click on.

Email subject line guidelines:

 Be clear and specific – avoid using generic or click bait subject lines that say nothing or make
unrealistic promises, like “find out how to double your business in a week”.

 Be original – avoid using those all too common subject line templates you find on the internet.
Instead make original subject lines that are relevant, personal, and concise.

 Add relevancy – address something that the recipient will recognize, like an acquaintance name or
a an article/ show/ book they appeared on.

Studies have shown that personalized subject lines are 26% more likely to be opened. You also want
to tailor your email subject line to your email goal, whether it’s a sales email, a personal email, a
newsletter, or something else.

2. Salutation

The salutation of a formal email is similar to the salutation of a letter. When writing to someone you do not
know by name, you put “To Whom it May Concern.” When applying for a job, you would address the
person by, “Dear Hiring Manager.” If you do know the recipient’s name, you put “Dear Mr./Ms. Smith.” For
a formal salutation, you should not use the recipient’s first name or the informal greetings “Hello” or “Hey.

3. Email opening

85
The next most important way to write a good email is by writing a strong email opening line. Like your
subject line the email opening is mostly used as another filtering stage for most people. If it fails to meet
the promise made in the subject line, your readers will ditch.

Therefore, it’s extremely important to define your main point in 1 or 2 paragraphs tops. If you clearly
convey your request or question and your reader feels it’s relevant and interesting, then they’ll continue
reading your email. If you manage to get them to stay after this point, in most cases, they’ll return your
email.

4. Email body

The body of your email is where you get into your main message. Whether you’re composing an email to
establish a new business connecting or just following up on a meeting, the body of your email should be
detailed enough that the reader isn’t confused, but also brief and to-the-point. No one wants to sit and
read a long-winded email when they have dozens of other unattended messages in their inbox.

Email body writing guidelines:

 Be concise – detail only what’s needed to get your point across.

 Use words that convey (authentic) positive personal emotional – words like “glad”, “excited”,
“intrigued”, “confident”.

 Use the word “because” when asking for something – it’s been scientifically shown that people are
more easily convinced to do something if told why, and more so if the reason is important for them.

 Show don’t tell – if you can’t explain something in few words, see if you can add a screenshot, a
video or a link that explain it better.

 Use headings to split you content if your email is long – if you can’t avoid writing a long email,
make sure to break it up into subsections with headings. This will help your time scarce readers to
scan and find their points of interest.

 Add your concrete request or question in bold text – to ensure your reader does not misses the
most important piece of content (your request or question) – set it in a separate line and put it in
bold.

5. Email ending

After you’ve addressed all your main points in the body of your email, you’ll want to end it with a
respectful and brief salutation. You can either invite your recipient to reach out for more questions, wish
them success, or ask a question. It all depends on the motive for your email. If it was a long email it could
also be a good idea to gently reiterate your main request, question or motivation.

6. Email sign-off

86
When closing your email, you’ll want to choose a suitable email sign off. There are different sign offs you
can use for each occasion, such as “best regards,” “sincerely,” or “with love,” but you obviously wouldn’t
want to send the last one to your manager. Make sure your signoff is appropriate to your email content
and your recipient.

7. Email signature/footer

Your email signature (or footer) is your wave of goodbye. The way you do this can affect the impression
you’ve made up to this point. If you make this moment memorable, organized and aesthetic you can get
some extra credit and positive attitude from your reader. On the other hand if you mess this up, your
entire message or offer may be put in doubt. So, make sure your email signature looks visually appealing
and well organized.

Consider creating a professional email signature to nail a positive lasting impression. Use the simple text
email signature we all used back when email started at your discretion. Whichever you choose, be sure to
include all your professional and contact information. It would also be a good idea to add links to your
website, social media, or a landing page.

Do’s and don’ts of e-mail writing


The improvised back-and-forth pattern we are comfortable with in social media conversations differs
greatly from the pre-planned, more self-contained messages most professionals expect in the workplace.

If you are planning an outing with friends, you expect multiple rapid exchanges asking for clarification and
providing corrections on the fly. Since you are usually texting somebody you already know well, about a
shared interest, you don’t need to provide much context.

But most professionals do not want to engage in a leisurely back-and-forth in order to get their work done.
They want to clear this item from their inbox, perhaps by passing it to an assistant or kicking it upstream,
without having to ask the sender “I dunno, what do you think?” or “What did you mean by that emoji?”

1. Write a meaningful subject line

2. Keep the message focused

3. Identify yourself clearly

4. Proofread

5. Distinguish between formal and informal situations

6. Respond Promptly

7. Show Respect and Restraint

1. Write a meaningful subject line.

Before you hit “send,” take a moment to write a subject line that accurately describes the content, giving
your reader a concrete reason to open your message. A vague or blank subject line is a missed opportunity
to inform or persuade your reader.
87
Remember — your message is not the only one in your recipient’s mailbox. A clear subject line will help a
busy professional to decide that your email is worthwhile.

Subject: [Blank]

A blank subject line suggests that your name in the “From” line is all your recipient should need in order to
make you message a top priority. That could come across as arrogant, or at the very least, thoughtless. A
well-chosen subject line is an important opportunity to inform and persuade your reader.

Subject: “Important! Read Immediately!!“

Rather than brashly announcing that the secret contents of your mystery message are inexplicably
important…

Subject: “All Cars in the Lower Lot Will Be Towed in 1 Hour.”

…write a functional subject line that actually conveys the important idea.

Subject: “Quick question.“

If the question is quick, why not just ask it in the subject line? This subject line is hardly useful.

Subject: “Follow-up about Friday“

Fractionally better — provided that the recipient remembers why a follow-up was necessary.

Subject: “That file you requested.“

Many email users get scads of virus-laden spam with vague titles like this. The more specific you are, the
more likely your recipient’s spam-blocker will let your message through.

Subject: “10 confirmed for Friday… will we need a larger room?“

Upon reading this revised, informative subject line, the recipient immediately starts thinking about the size
of the room, not about whether it will be worth it to open the email.

88
2. Keep the message focused.

Why are you writing? Are you responding to a request? Apologizing for an error on your part? Asking for
the recipient to take some action for you?

 Purpose: Any textbook on business and professional writing will include examples of complaint and
adjustment letters, proposal letters, progress reports, application letters, and so forth.

 Directness: You probably don’t need to open with “Dear Ms. Jones,” engage in personal chit-chat,
and close with “Yours Truly.” (If you really want to be that formal, send a letter on paper instead.)

 Organization: Readers will often get partway through a complex message, hit “reply” as soon as
they have something to contribute, and forget to read the rest. That’s human nature.

 Number your points in more complex message. (Start with a clear statement of how many
parts there are to your message.)

 Split unrelated points into separate, purposeful emails.

 If you send all your employees a message that only relates to some of them, a lot of
people will waste time reading the whole thing, in order to determine whether any
part of it applies to them.

 Other people will give up as soon as they find any detail that does not apply to them.
(Again, this is human nature.)

 Politeness: Please and thank-you are still important, but wordiness wastes your reader’s time
(which is rude).

Indirect and wasteful: “Dearest Arnold: I would be very much obliged if, at your earliest
convenience, you could send me the current password for the website. I look forward to
your response. Have a nice day! Yours Truly, Philomena.”

Blunt to the point of rudeness: “Need the password for the website.”

If you get a message like this, you might assume the sender trusts you and really needs your help;
however, if you send a message like this, you might appear needy and panicky. Is that how you
want to come across? Think about it.

Urgent, yet polite: “Site is down, but I can’t troubleshoot without the new password. Do
you know it?”

help your reader focus on your message: keep your text readable.

89
 Proofread, especially when your message asks your recipient to do work for you. All-caps comes
across as shouting, and no-caps makes you look like a lazy teenager. Regardless of your intention,
people will respond accordingly.

 If you are in middle school, a gushing statement “thx 4 ur help 2day ur gr8!″ may make a
busy professional smile — or shudder.

 Often, the sweetness of the gesture won’t be enough. u want ur prof r ur boss 2 think u cant
spl? LOL ;-)

 Write short paragraphs, separated by blank lines. Most people find unbroken blocks of text boring,
or even intimidating. Take the time to format your message for the ease of your reader.

 Avoid fancy typefaces. Don’t depend upon bold font or large size to add nuances. Your recipient’s
email reader may not have all the features that yours does. In a pinch, use asterisks to show
*emphasis*.

3 . Identify yourself clearly.

If you telephoned someone outside your closest circle, someone who probably wouldn’t recognize your
voice, you would probably say something like “Hello, Ms. Wordsworth, this is Sally Griffin.” A formal “Dear
Ms. Wordsworth” salutation is not necessary for routine workplace communication.

When we send text messages to our friends, we expect a lot of back-and-forth. But professionals who use
email don’t enjoy getting a cryptic message from an email address they don’t recognize.

While a routine email does not require a formal salutation such as “Dear Ms. Wordsworth,” ask yourself
whether the person you are writing knows you well enough to recognize your email address.

To: Professor Blinderson


From: FuZzYkItTy2000@hotmail.com
Subject: [Blank]Yo goin 2 miss class whats the homework

Professor Blinderson will probably reply, “Please let me know your name and which class you’re in, so that I
can respond meaningfully. I don’t recognize the address FuZzYkItTy2000@hotmail.com.”

To: Professor Blinderson


From: m.ponsybil@gmail.com
Subject: EL227 Absence, Oct 10Hello, Prof. Blinderson. This is Morris Ponsybil, from EL227 section
2.This morning, I just found out that the curling team has advanced to the playoffs, so I’m going to
be out of town on the 10th.According to the syllabus, it looks like I will miss a paper workshop and
the discussion of Chapter 10. May I email you my Chapter 10 discussion questions before I leave
town? And could I come to your office hour at 2pm on the 12th, in order to discuss the paper? I’ve
asked Cheryl Jones to take notes for me.Thank you very much. I’ll see you in class tomorrow.

90
If you are asking the other person to do you a favor, providing the right information will give him or her a
good reason to decide in your favor. In this case, Morris Ponsybil shows his professor he cares enough about
the class to propose a solution to the problem his absence will cause.

When contacting someone cold, be polite and brief. If you are asking for a stranger to do something for
you for free, be prepared to hear nothing in response.

Even if you already have a connection with the person you are contacting, a little context is helpful. Every
fall, I get emails from “bad_boy2315@yahoo.com” or “FuZzYkItTy2000@hotmail.com” who ask a question
about “class” and don’t sign their real names.

4. Proofread.

If you are asking someone else to do work for you, take the time to make your message look professional.

While your spell checker won’t catch every mistake, at the very least it will catch a few typos. If you are
sending a message that will be read by someone higher up on the chain of command (a superior or
professor, for instance), or if you’re about to mass-mail dozens or thousands of people, take an extra
minute or two before you hit “send”. Show a draft to a close associate, in order to see whether it actually
makes sense.

5. Distinguish between formal and informal situations.

When you are writing to a friend or a close colleague, it is OK to use “smilies” :-) , abbreviations (IIRC for “if
I recall correctly”, LOL for “laughing out loud,” etc.) and nonstandard punctuation and spelling (like that
found in instant messaging or chat rooms).

These linguistic shortcuts are generally signs of friendly intimacy, like sharing cold pizza with a family
friend. If you tried to share that same cold pizza with a first date, or a visiting dignitary, you would give off
the impression that you did not really care about the meeting. By the same token, don’t use informal
language when your reader expects a more formal approach.

Always know the situation, and write accordingly.

6. Respond Promptly.

If you want to appear professional and courteous, make yourself available to your online correspondents.
Even if your reply is, “Sorry, I’m too busy to help you now,” at least your correspondent won’t be waiting in
vain for your reply.

7. Show Respect and Restraint

Many a flame war has been started by someone who hit “reply all” instead of “reply.”

While most people know that email is not private, it is good form to ask the sender before forwarding a
personal message. If someone emails you a request, it is perfectly acceptable to forward the request to a
person who can help — but forwarding a message in order to ridicule the sender is tacky.

91
Use BCC instead of CC when sending sensitive information to large groups. (For example, a professor
sending a bulk message to students who are in danger of failing, or an employer telling unsuccessful
applicants that a position is no longer open.) The name of everyone in the CC list goes out with the
message, but the names of people on the BCC list (“blind carbon copy”) are hidden. Put your own name in
the “To” box if your mail editor doesn’t like the blank space.

Be tolerant of other people’s etiquette blunders. If you think you’ve been insulted, quote the line back to
your sender and add a neutral comment such as, “I’m not sure how to interpret this… could you
elaborate?”

Tutorial
EXERCISE 1

Q1. Formal vs. Informal

State whether the sentence would be classified as either formal or informal. If informal, change it to
formal.

Example: Hi y’all! _informal;

1.) I am pleased to inform you that you have won our grand prize. _____________

2.) I hope all is well with your new career choice. _____________

3.) I shouldn’t have gone and missed with it!! _____________

4.) I can’t help you with that cuz it’s too hard. _____________

5.) Hi, how are you?_____________

Q2. Putting It All Together

Find and correct the errors in the following emails:

1.) Hello Professor Smith,

I’m sorry to tell you but im sick and will not be able to come to class. See ya Wednesday.

Jason

______________________________________________________________________________
______________________________________________________________________________
______________________________________________________________________________
______________________________________________________________________________
______________________________________________________________________________
______________________________________________________________________________

92
2.) Dear Sally Blue,

I read online that you’re selling business cards. I was wondering how much if i only wanted 500? Is color
and a logo extra? Can I see an example before all are shipped or will that cost extra? You seem to have a
great business so I hope you can help.

Thanks, Jess Higgins

______________________________________________________________________________
______________________________________________________________________________
______________________________________________________________________________
______________________________________________________________________________
______________________________________________________________________________
______________________________________________________________________________
______________________________________________________________________________

3.) Dear Sir/Madam,

I am a graduate from menlo college. I got a degree in business and would now like to use it. Your company
looks interesting. Can I come in for an interview? I have alot of experience from my schooling and extra
cirriculers. I think I can help the company alot. Please respond to my email to let me know.

Thanks, Max Oates


______________________________________________________________________________
______________________________________________________________________________
______________________________________________________________________________
______________________________________________________________________________
______________________________________________________________________________
______________________________________________________________________________
______________________________________________________________________________
_____________________________________________________________________________
______________________________________________________________________________
______________________________________________________________________________
______________________________________________________________________________
______________________________________________________________________________

EXERCISE 2

Q1. Using the following phrases, write an email with minimum of 50 words to the customer Mr. Bill Roy
explaining delay to the project, sign the email as Reese:

Payment processing system – schedule – 10th May (Friday) – Unexpected power outage – 3 days – Overall
delay-7 days – includes recovery of lost work – will not recur

Q2. As a student representative of your department, write an email to your batch mates, suggesting a
party for Head of Department Prof.Sunil who is retiring next month. Sign the email as Sam.

93
inform – retire – plan – surprise – party – host – family – exceptional teacher – guide – mentor – groom –
students – helpful – together – memorable

Q3. As a recent buyer of their car, write an email to the Manager of Smart Automotive company,
Mr.Ahmed, regarding the poor quality of service facility available in the city. Sign the email as Chopra.

Outline: very few – service centers – complaints – pending problems – maintenance – cost – time –
delivery – increase – customer satisfaction

Q4. You are the project leader for a team of 20 members. As the team members are not submitting the
weekly time sheets regularly, you need to email them stressing the need to submit without fail.

Using the following phrases, write an email with a minimum of 70 words and a maximum of 100 words to
your team members informing the same can be accessed online

– lead to loss of pay-every week – do not default-used to bill client-actual working hours – by Friday –
failure to adhere-time sheet filling application.

Q5. As an intern at ABC consulting Pvt.Ltd, write an email to your internship Project Manager, Mr.Ramesh,
informing about the progress that you are making and some difficulties that you are encountering. Sign the
email as Ben.

Outline: Thank – challenging – progress – tight schedule – support – report – analytics – guidance – access
– doubt – requirements – design.

Exercise 3

1. Email content should ideally be presented as-

A. One long paragraph

B. Bullet points

C. Story format

D. Text and pictures

2. Most effective way of closing an email is

A. With a social message

B. With a clear actionable

C. Repeating the subject line

D. With a proverb

3. How should the subject line be

A. Long and descriptive

B. Short and precise

94
C. A greeting

D. Does not matter

4. Email is a reflection of

A. Designation and pride

B. Professionalism, Values, Attention to detail

C. Authority and status

D. Nothing. It is used for getting work done

5. Email font should be

A. Colourful and fancy

B. Black in colour, easy to read, capital alphabets only where required

C. Bright colours, easy to read

D. Bold and all capital alphabets

6. SMS language ( dey, u, tmrw) in the email should be-

A. Used with selected few clients

B. Can be used sometimes if you feel like.

C. Should not be used

D. Used while writing to juniors

7. Important points/words can be

A. Mentioned repeatedly for impact

B. Need not be highlighted

C. Highlighted in bold/underlined

D. Highlighted with multiple colours

8. Usage of all capital alphabets on email is considered as -

A. Showing importance

B. Looks fancy

C. Screaming/Insulting

D. Irrelevant

9. It is important to read the email completely before responding.


95
A. Only if you have time

B. Not important at all

C. Always

D. Only if it is from seniors

10. Proofreading emails before hitting send is-

A. Can be done sometimes

B. Not important

C. A best practice

D. Waste of time

96
UNIT 4

ESSAY WRITING
What is an essay?

An essay can be thought of as an extended answer to a question. Most importantly, it presents your
position on a topic, which is often referred to as your argument. The essay is your opportunity to
demonstrate that you can think critically about complex issues, and can clearly communicate the
conclusions you have reached as a result of your research.

Essays are usually assessed on how deeply you have engaged with the topic, how clearly you have
presented your ideas and argument, and how well you have drawn on relevant evidence to support your
ideas.

Types of essays

1. Narrative Essay-

A narrative essay achieves a certain purpose through telling a story, which makes it interesting to the
reader and also results in getting some point across. For example, you might write a story about meeting
someone special to you. The purpose may be to tell how meeting this person affected your decision to
attend college or to entertain the reader with a funny story about that person. The purposes of writing
narratives include the following: to create a sense of shared history, to provide entertainment, to offer
instruction, and to provide insight. Some examples of narrative essay topics include a special person,
event, or place. After choosing a topic, you should determine the main point you want to make about that
topic and then concentrate on that throughout the rest of the paper.

Guidelines for writing a narrative essay:

1. Make a specific point. Determine what the purpose of your essay is. State that point in your thesis
statement, or controlling idea, and build upon it throughout the essay by using examples, stories,
and other details that all relate back to the main idea.
2. Involve readers in the story and create a visual picture by using dialogue and physical description, which is
achieved through telling events that happened and presenting problems that arose. Also include specific
descriptive details so that the reader can easily picture the scene in each of the events.
3. Sequence events. Often this is done chronologically (the order in which events happened), but using
flashbacks (writing the story from one point in time but then talking briefly about something that happened
in the past) and foreshadowing (briefly talking about something that will happen in the future) can make the
story more interesting.
4. Decide which point of view, first-person (from the author’s point of view, using “I”) or third person (using
“he,” “she,” “them,” etc. instead of “I”), works best for your essay, and tell the story from that viewpoint.
2. Descriptive Essay:-

A descriptive essay tells about a certain topic or story, using details to appeal to the five senses. It gives readers
the ability to vividly imagine the situation or scene and feel as if they are experiencing it first-hand. To write a
descriptive essay, you could choose a person, place, event, object, or experience and describe it thoroughly
using many sensory details.

97
Guidelines for writing a descriptive essay:

1. Use sensory details. Appeal to sight, sound, smell, taste, and touch.
2. Use active verbs (in which the subject is doing the acting instead of being acted upon by something or
someone else, such as “Cindy hit the ball.” instead of “The ball was hit by Cindy.”). Also, use a variety of
sentence types, such as a simple sentence (“Tom went to the store.”), compound sentence (“Tom went to
the store, and he bought a loaf of bread.”), complex sentence (“While Tom was at the store, he bought a loaf
of bread.”), or compound complex sentence (“While Tom was at the store, he bought a loaf of bread, but he
forgot to buy a gallon of milk.”).
3. Avoid vague language, such as “pretty,” “really,” “a lot,” and “very.”
4. Create a dominant impression (overall attitude, mood, or feeling about the subject), such as causing the
readers to sympathize with you about a sad event that happened or to agree with you strongly on a certain
controversial topic. Choose the details and descriptions that will help accomplish this impression.
5. Use comparisons. In order to help make the descriptions even more vivid to the reader, use similes (such as
“His emotional state was like a roller coaster.”), metaphors (such as “Her smile was a ray of sunlight in the
dark sea of unfamiliar faces.”), personification (giving human characteristics to an object, such as “The wind
howled in my ears and beckoned me to walk further into the forest.”), and analogies (brief stories that relate
to the topic). Determine the method of organization that works best for your essay: spatial (top to bottom,
inside to outside, near to far), chronological (the order in which events happened), least-to-most, or most-
to-least.

3. Comparison and/or Contrast Essay:-

A comparison and/or contrast essay tells about two or more main subjects by pointing out similarities and/or
differences. One way to write this type of essay would be to choose two or more objects, people, places, events,
experiences, or ideas and compare and/or contrast them according to a few specific points.

Guidelines for writing a comparison and/or contrast essay:

1. Have a clear purpose and a main point (to express ideas, to inform, to persuade, etc.). Make sure your
subjects have something specific on which you can base your comparison and/or contrast.

2. Decide the order that best suits your essay. The two main ways you can choose to organize a comparison
and/or contrast essay are point-by-point organization and subject-by-subject organization.

point-by-point: switching back and forth between the subjects, comparing/contrasting


them according to several main points

subject-by-subject: writing about all the main points of one subject and then writing about all the main points
of another subject, and so on, referring back to the other subject(s) in a comparing/contrasting wa

3.In your thesis statement, or controlling idea, include the subjects; identify whether you will be talking about
similarities, differences, or both; and state your main point

4. Have a sufficient number of significant characteristics and details (which are the various points you use to
compare and/or contrast the subjects), and give an equal amount to all sides.

4.Cause and Effect Essay (also called causal analysis):-

A cause and effect essay analyzes what causes certain things to happen or why things are a certain way, the results
brought about by certain events, or both. For example, you may want to write an essay about an event that
happened and then tell how it affected your life. This would tell the cause (the event) and the effect (how it affected
98
your life). To begin writing a cause and effect essay, you may want to first decide on a topic, such as an event or a
person, and then decide what you will tell about that topic, such as what happened to make that specific event take
place or what made the person have a certain attitude or personality. You could also tell about a specific topic and
what effects it had on future events.

Guidelines for writing a cause and effect essay:

1. Decide whether you will focus on causes, effects, or both.


2. There can be multiple causes of one effect. (For example, because you saved your money, shopped around for
the best deal, and saw an ad in a newspaper, you were able to buy the car of your dreams.) There can also be
one cause of multiple effects. (For example, your decision to stay out late one night may have caused you to
miss an important phone call, forget to study for a test, and made you tired the next day.) There can also be a
chain of events in which each effect turns out to be the cause of another effect. (For example, one event may
have affected your decision to travel to Europe, which affected your decision to study a foreign language, which
helped you get a job as a travel agent.)
3. Have a clear purpose (to tell information, to persuade readers, or both). This purpose is achieved through a
main idea, which should be included in the thesis statement, or controlling idea.
4. Choose a logical organization. For example, you may want to use chronological (the order in which events
happened), most-to-least (such as most effective to least effective or most difficult to least difficult), least-to-
most (such as least costly to most costly or least obvious to most obvious), or order of importance (such as a
belief that is most important to you, then a belief that is a little less important, and so on).
5. Explain cause and/or effect fully, using details to support your explanations

5.Argumentative Essay:-

An argumentative essay makes a claim and then gives examples and evidence to prove that point. You can begin
writing an argumentative essay by deciding on a certain topic, such as something about a belief, idea, or
controversial issue. Then, do research to obtain detailed information. The information can be statistics, stories,
examples, observances, and other facts. Use the information throughout the essay to argue your point and convince
the reader

Guidelines for writing an argumentative essay:

1. Choose a narrow issue.


2. Tell your reader what you believe about the issue very clearly and specifically in your thesis statement, or
controlling idea.
3. Be aware of your audience and anticipate the reader’s views. This means that if you know your readers will
most likely not believe a certain statement because it goes against traditional beliefs or their personal
viewpoints, make sure you give solid evidence to back it up.
4. Use convincing details as evidence to support your claim. These should be from believable and reliable
sources. For example, if you’re stating something that has to do with human health, quote a doctor who
knows about that topic, or if you are talking about a trend in society, include statistics from a well-known
organization
5. Choose the most appropriate way of arguing your point (inductive or deductive). inductive=start with
stating all the evidence and then move on to one conclusion deductive=start with one basic statement of
belief and move on to the supporting points of evidence
6. Acknowledge opposing viewpoints to prove you have thought about the topic thoroughly yet still find that
your point is correct.

99
Structure of the essay

Writing an introduction:-

The introduction guides your reader into the paper by introducing the topic. It should begin with a hook
that catches the reader’s interest. Every essay or assignment you write must begin with an introduction. It
might be helpful to think of the introduction as an inverted pyramid. In such a pyramid, you begin by
presenting a broad introduction to the topic and end by making a more focused point about that topic in
your thesis statement. The introduction has three essential parts, each of which serves a particular
purpose.

1. Attention-grabber

The first part is the "attention-grabber." You need to interest your reader in your topic so that they will
want to continue reading. You also want to do that in a way that is fresh and original. For example,
although it may be tempting to begin your essay with a dictionary definition, this technique is stale
because it has been widely overused. Instead, you might try one of the following techniques:

o Offer a surprising statistic that conveys something about the problem to be addressed in the paper.

o Perhaps you can find an interesting quote that nicely sums up your argument.

o Use rhetorical questions that place your readers in a different situation in order to get them
thinking about your topic in a new way.

o If you have a personal connection to the topic, you might use an anecdote or story to get your
readers emotionally involved.

o For example, if you were writing a paper about drunk drivers, you might begin with a compelling
story about someone whose life was forever altered by a drunk driver: "At eighteen, Michelle had a
lifetime of promise in front of her. Attending college on a track scholarship, she was earning good
grades and making lots of friends. Then one night her life was forever altered…"
100
2. Background information or linking statement

2. From this attention grabbing opener, you would need to move to the next part of the introduction,
in which you offer some relevant background on the specific purpose of the essay. This section helps
the reader see why you are focusing on this topic and makes the transition to the main point of your
paper. For this reason, this is sometimes called the "transitional" part of the introduction.

o In the example above, the anecdote about Michelle might capture the reader's attention, but the
essay is not really about Michelle. The attention grabber might get the reader thinking about how
drunk driving can destroy people's lives, but it doesn't introduce the topic of the need for stricter drunk
driving penalties (or whatever the real focus of the paper might be).

o Therefore, you need to bridge the gap between your attention-grabber and your thesis with some
transitional discussion. In this part of your introduction, you narrow your focus of the topic and explain
why the attention-grabber is relevant to the specific area you will be discussing. You should introduce
your specific topic and provide any necessary background information that the reader would need in
order to understand the problem that you are presenting in the paper. You can also define any key
terms the reader might not know.

o Continuing with the example above, we might move from the narrative about Michelle to a short
discussion of the scope of the problem of drunk drivers. We might say, for example: "Michelle's story is
not isolated. Each year XX (number) of lives are lost due to drunk-driving accidents." You could follow
this with a short discussion of how serious the problem is and why the reader should care about this
problem. This effectively moves the reader from the story about Michelle to your real topic, which
might be the need for stricter penalties for drinking and driving.

3. Thesis statement

Finally, the introduction must conclude with a clear statement of the overall point you want to make in
the paper. This is called your "thesis statement." It is the narrowest part of your inverted pyramid, and
it states exactly what your essay will be arguing.

 In this scenario, your thesis would be the point you are trying to make about drunk driving. You
might be arguing for better enforcement of existing laws, enactment of stricter penalties, or
funding for education about drinking and driving. Whatever the case, your thesis would clearly
state the main point your paper is trying to make. Here's an example: "Drunk driving laws need to
include stricter penalties for those convicted of drinking under the influence of alcohol." Your essay
would then go on to support this thesis with the reasons why stricter penalties are needed.

Writing body paragraphs

Within an essay, body paragraphs allow a writer to expand on ideas and provide audiences with support
for a chosen topic or argument. Under most circumstances, body paragraphs can be divided into three
basic parts: a topic sentence, an illustration, and an explanation. These three parts answer three questions
for the reader: What’s your point? Can you give an example? How does your example establish your point?
If you answer these three questions, you will have also established a conclusion with which to end the
paragraph. To write an effective body paragraph, follow these steps:

101
● begin with a clear topic sentence that logically supports the thesis statement.

● support the argument by providing facts, examples, quotations, or statistics that develop or support the
central claim of the paragraph.

● explain the significance of each fact, example, quote, or statistic.

● draw a clear conclusion that is relevant to the thesis statement.

Because it develops evidence to support a claim, an effective body paragraph moves from general to
specific information. The most general information should be at the beginning of the paragraph, and, as
the paragraph moves forward, the paragraph should become more focused as it provides specific
information to reach a clear and specific conclusion. You can visualize a body paragraph as an inverted
triangle.

Parts of a Body Paragraph

The different parts of a body paragraph combine to generate clear and logical ideas for the reader. It is
important to be comfortable with how to use these parts to create a readable, usable paragraph.

Topic Sentence

In academic essays, topic sentences usually appear at the beginning of a body paragraph. Because a topic
sentence encapsulates the idea of your body paragraph, it introduces to your audience what the paragraph
will be about. It should reflect or advance the argument of your thesis statement.

Supporting Details

Following the topic sentence are sentences that introduce your supporting evidence. Evidence may include
a useful or informative sentence from a book, a journal article, or another source that supports the
102
argument of the paper. It may also include an example drawn from first-hand observation or personal
experience.

Explanation

Clarify the information you provided with your quote or example. Explain what the quote means in a
concise manner. Provide necessary details to develop your example as evidence.

Significance

Discuss the significance of the quote/example to the argument of the paper.

Conclusion/Transition

As the discussion of evidence draws to a close, you should draw on the evidence and explanation to reach
a conclusion: a new understanding of the thesis. This conclusion may set up the transition to the next
paragraph.

Sample Body Paragraph

Below is an example of a well-constructed body paragraph that uses all of the parts of a body
paragraph mentioned above to demonstrate the thematic significance of a beloved character? These
parts are labeled. Notice how all the parts combine to express the author‟s point effectively.

[Topic Sentence] In Harry Potter and the Sorcerer’s Stone, J.K. Rowling suggests through her portrayal
of Hagrid that appearance can be misleading. [Supporting Details] Although he is oneof the most loved
characters in the Harry Potter universe, his initial confrontation with Harry strikes intimidation in the
young boy‟s heart. The narrator notes that Hagrid‟s face is “almost completely hidden by a long, shaggy
mane of hair and a wild, tangled beard,” and his eyes, “glint[ing] like black beetles,” peer down at Harry
(Rowling 46). [Explanation] Harry‟s anxietyseeps through the page as he is confronted by an
intimidating figure, and because this descriptionpaints such a scary picture, the reader might expect
Hagrid to be a rather frightening person as the book progresses. [Significance] This initial description,
however, is contrasted by Hagrid‟s true character. Besides serving as a trusted informant to Dumbledore
and a crucial ally to Harry and his friends, Hagrid displays an immense compassion for animals, affection
so powerful it sometimes reduces the giant to tears. [Conclusion/Transition] Hagrid‟s humble actions
and earnest compassion show that appearances can be misleading, a theme that many characters mirror in
the book.

Concluding the essay

A conclusion works to remind your reader of the main points of your paper and summarizes what you
want your reader to “take away” from your discussion. Consider these tips when writing your conclusion:

 Begin with your rephrased thesis statement to remind your reader of the point of your paper.
 Summarize the points you made in your paper and show how they support your argument; tie all
the pieces of your paper together.
 Tell your reader what the significance of your argument might be. Why is the discussion
important? Do you want your reader to think differently, question something, or perform some

103
action? Make a recommendation of what your reader should "do" with the information you just
gave them, or share the importance of the topic.

Useful phrases for Essay writing


Developing the argument
 The first aspect to point out is that…
 Let us start by considering the facts.
 The novel portrays, deals with, revolves around…
 Central to the novel is…
 The character of xxx embodies/ epitomizes…
The other side of the argument
 It would also be interesting to see…
 One should, nevertheless, consider the problem from another angle.
 Equally relevant to the issue are the questions of…
Conclusion
 The arguments we have presented… suggest that…/ prove that…/ would indicate that…
 From these arguments one must…/ could…/ might… conclude that…
 All of this points to the conclusion that…
 To conclude…
Ordering elements
 Firstly,…/ Secondly,…/ Finally,… (note the comma after all these introductory words.)
 As a final point…
 On the one hand, …. on the other hand…
 If on the one hand it can be said that… the same is not true for…
 The first argument suggests that… whilst the second suggests that…
 There are at least xxx points to highlight.
Adding elements
 Furthermore, one should not forget that…
 In addition to…
 Moreover…
 It is important to add that…
Accepting other points of view
 Nevertheless, one should accept that…
 However, we also agree that…
Personal opinion
 We/I personally believe that…
 Our/My own point of view is that…
 It is my contention that…
 I am convinced that…
 My own opinion is…
Others’ opinions
 According to some critics…
Critics:
104
 believe that
 say that
 suggest that
 are convinced that
 point out that
 emphasize that
 contend that
 go as far as to say that
 argue for this
Introducing examples
 For example…
 For instance…
 To illustrate this point…
Introducing facts
 It is… true that…/ clear that…/ noticeable that…
 One should note here that…
Saying what you think is true
 This leads us to believe that…
 It is very possible that…
 In view of these facts, it is quite likely that…
Certainty
 Doubtless,…
 One cannot deny that…
 It is (very) clear from these observations that…
Doubt
 All the same, it is possible that…
 It is difficult to believe that…
Accepting other points to a certain degree
 One can agree up to a certain point with…
 Certainly,… However,…
 It cannot be denied that…
Emphasizing particular points
 The last example highlights the fact that…
 Not only… but also…
 We would even go so far as to say that…
Moderating, agreeing, disagreeing
 By and large…
 Perhaps we should also point out the fact that…
 It would be unfair not to mention the fact that…
 One must admit that…
 We cannot ignore the fact that…
 One cannot possibly accept the fact that…
Consequences
 From these facts, one may conclude that…
105
 That is why, in our opinion, …
 Which seems to confirm the idea that…
 Thus,…/ Therefore,…
Comparison
 Some critics suggest…, whereas others…
 Compared to…
 On the one hand, there is the firm belief that… On the other hand, many people are convinced
that…
Phrases For Balanced Arguments
Introduction
It is often said that…
It is undeniable that…
It is a well-known fact that…
One of the most striking features of this text is…

Thesis
The first thing that needs to be said is…
First of all, let us try to analyze…
One argument in support of…
We must distinguish carefully between…
The second reason for…
An important aspect of the text is…
It is worth stating at this point that…

Antithesis
On the other hand, we can observe that…
The other side of the coin is, however, that…
Another way of looking at this question is to…

Conclusion
What conclusions can be drawn from all this?
The most satisfactory conclusion that we can come to is…
To sum up… we are convinced that…/ …we believe that…/ …we have to accept that…

106
107
TUTORIAL
Exercise 1

Q1. Review the following body paragraph. 1) Eliminate any sentences that do not relate to the topic
sentence and main idea of the body paragraph. 2) Identify each of the five elements of a body
paragraph defined above. This exercise will strengthen your ability to stay focused when you write
your next paper.

One of the most important aspects of ice hockey is speed because players must skate around
defensemen and get to open areas of the rink. The fastest players are able to sneak around an
opposing defense and go on breakaways, creating scoring chances. Being big is also important
because size allows players to hit hard. One of the fastest players in the NHL is Teemu Selanne of the
Anaheim Ducks. Because of his iconic speed and Finish heritage, Teemu is known as the “Finnish
Flash.” He has used his speed to score more than 600 goals during his career. He can usually be seen
streaking down the boards, flying by helpless defensemen, and crashing the net to score goals.
“Teemu Selanne‟s impressive career resulted in 10 All Star Game appearances.” Ultimately, such
speed is what makes a player extraordinary, even though it is just one of many attributes an ice
hockey player must have to succeed.

Q2. What is the most effective way of trying to use the linking words in essay writing?

A. Overusing links words that are simple

B. Learning only a few linking words and phrases to use

C. Between each sentences

D. Learning a lot of linking words and phrases to use

Q3.Which one of these points do you include in an introduction?

A. Final results

B. Evidence

C. Introduce topic areas in one or two sentences

Q4.Which one of these points is included in the body?

A. Summary of the topic

B. A topic sentence that states the main idea

108
C. Thesis statement

Q5.What does a conclusion include?

A. Synthesizing your major points

B. Describing the topic

C. The positive and the negative points

Q6. What do you need to include in a conclusion?

A. New ideas

B. Restating the thesis statement

C. Introduce new details

Exercise 2

Q1. Many old buildings protected by law are part of a nation’s history. Some people think they should be
knocked down and replaced by news ones. How important is it to maintain old buildings? Should history
stand in the way of progress?

Q2. Some people believe the aim of university education is to help graduates get better jobs. Others
believe there are much wider benefits of university education for both individuals and society. Discuss both
views and give your opinion.

Q3. Formal examinations are the only effective way to assess a student's performance. Continual
assessment such as course work and projects is not a satisfactory way to do this. To what extent do you
agree or disagree with this statement?

Q4. Stress is now a major problem in many countries around the world. What are some of the factors in
modern society that cause this stress, and how can we reduce it?

Q5. Do children behave better when they are physically punished or rewarded?

Q6. Nowadays the way many people interact with each other has changed because of technology. In what
ways has technology affected the types of relationships people make? Has this become a positive or
negative development?

Give reasons for your answer and include any relevant examples from your own knowledge or experience.

Q7. Every year several languages die out. Some people think that this is not important because life will be
easier if there are fewer languages in the world. To what extent do you agree or disagree with this
opinion?
109
Q8. In many countries around the world young people decide to leave their parents’ home once they finish
school. They start living on their own or sharing a home with friends. Is this a positive or a negative
development? Give reasons for your answer and include relevant examples.

Q9. Nowadays online shopping is extremely popular. Discuss the impact of it on the environment and on
people, who lost their jobs because of it.

Q10. Many people find it difficult to speak in front of and to present to an audience. It is believed that this
skill should be taught in school. Why is this important? To what extent do you agree or disagree with it?

Exercise 3

1. Should plastic be banned?

2. Pollution due to Urbanization

3. Education should be free

4. Should Students get limited access to the Internet?

5. Selling Tobacco should be banned

6. Smoking in public places should be banned

7. Facebook should be banned

8. Students should not be allowed to play PUBG

9. Does Indian cinema shape our popular culture or merely reflect it?

10. Is the growing level of competition good for the youth?

11. Fulfilment of ‘new woman’ in India is a myth.

12. Be the change you want to see in others.

13. Science and technology is the panacea for the growth and security of the nation.

14. Farming has lost the ability to be a source of subsistence for the majority of farmers in India.

15. Technology cannot replace manpower

110
Reading Comprehension
What is reading comprehension?

Reading comprehension is the ability to understand and interpret what you read. In order to successfully
comprehend written material, you need to be able to decode efficiently as well as interpret the language
being read at a high level. Look at the figure to the right to see the skills necessary to master
comprehension.

The Purpose of Reading

The purpose of reading is to connect the ideas on the page to what you already know. If you don't know
anything about a subject, then pouring words of text into your mind is like pouring water into your hand.
You don't retain much. For example, try reading these numbers:

7516324 This is hard to read and remember.

751-6324 This is easier because of chunking.

123-4567 This is easy to read because of prior knowledge and structure.

Similarly, if you like a sport, then reading the sports page is easy. You have a framework in your mind for
reading, understanding and storing information.

Improving Comprehension

Reading comprehension requires motivation, mental frameworks for holding ideas, concentration and
good study techniques. Here are some suggestions.
111
Develop a broad background.

Broaden your background knowledge by reading newspapers, magazines and books. Become interested in
world events.

Know the structure of paragraphs.

Good writers construct paragraphs that have a beginning, middle and end. Often, the first sentence will
give an overview that helps provide a framework for adding details. Also, look for transitional words,
phrases or paragraphs that change the topic.

Identify the type of reasoning.

Does the author use cause and effect reasoning, hypothesis, model building, induction or deduction,
systems thinking?

Anticipate and predict.

Really smart readers try to anticipate the author and predict future ideas and questions. If you're right, this
reinforces your understanding. If you're wrong, you make adjustments quicker.

Look for the method of organization.

Is the material organized chronologically, serially, logically, functionally, spatially or hierarchical? See
section 10 for more examples on organization.

Create motivation and interest.

Preview material, ask questions, and discuss ideas with classmates. The stronger your interest, the greater
your comprehension.

Pay attention to supporting cues.

Study pictures, graphs and headings. Read the first and last paragraph in a chapter, or the first sentence in
each section.

Highlight, summarize and review.

Just reading a book once is not enough. To develop a deeper understanding, you have to highlight,
summarize and review important ideas.

Build a good vocabulary.

For most educated people, this is a lifetime project. The best way to improve your vocabulary is to use a
dictionary regularly. You might carry around a pocket dictionary and use it to look up new words. Or, you
can keep a list of words to look up at the end of the day. Concentrate on roots, prefixes and endings.

Use a systematic reading technique like SQR3.

112
Develop a systematic reading style, like the SQR3 method and make adjustments to it, depending on
priorities and purpose. The SQR3 steps include Survey, Question, Read, Recite and Review.

Monitor effectiveness.

Good readers monitor their attention, concentration and effectiveness. They quickly recognize if they've
missed an idea and backup to reread it.

Question types :-
1. main idea
2. Inference
3. assumption
4. vocabulary

Main Idea
All writing has a main idea its writer wants to communicate. Sometimes called the central thought or
theme, the main idea inspires the title, is illustrated by the rest of the writing, and conveys the author’s
purpose for writing the passage. The most important hint for selecting the main idea is to ask yourself, “Is
this statement supported by all or most of the material in the selection?”

Often them main idea is directly stated in a single topic sentence which summarizes what the entire
passage is about. Sometimes the main idea is unstated, for the author has decided to let the details
suggest the central point. In such a case, you, the reader, must figure you what the implied main idea is.
Again, the main idea can be tested by asking, “Is this statement supported by all or most of the material in
the selection?”

On reading tests, main idea questions may assume a variety of formats. These include the following:

The main idea of this passage is that…

Which sentence best expresses the main idea?

The paragraph could be entitled…

The point of the paragraph is…

What idea does the paragraph develop?

Example : Read the following passages and select the best answer to each main idea question.

Almost everyone would agree that cleaning up the environment and keeping it that way is a good thing.
The obvious negative impacts on human health and the ecosystems on Earth are well understood. Yet,
there are underlying consequences to compliance to this most basic need that need to be studied, as well.
The government of the U.S. has requested voluntary action from citizens and industry, and it has also
legislated action in order to start the clean-up process. An example of this is the Clean Air Act. The good
news is that this law requires that emissions into the air meet certain standards that will help clean up the
environment. Under this act, cars have to reduce emissions of nitrous oxides by 60%, and "clean coal"
technologies have to reduce harmful emissions by 20%. Industry is required to reduce emissions of over
113
200 compounds that are known to cause health problems. The bad news is the cost of compliance.
Estimates of consumer costs for a cleaner environment go as high as $50 billion a year, and that study does
not even take into account the loss of jobs from reshaping industries to meet the new standards.

Which of the following statements best expresses the main idea of the paragraph?

A. Industries in nations across the United States need to unite to maintain the standards created by the
Clean Air Act despite the costs to the country.

B. Industries are doing their part to reduce emissions that are known to have ill effects on the
environment and human health.

C. Although legislation that supports cleaning up the environment is a good idea, the costs of such efforts
need to be evaluated.

D. Cleaning up the environment is best accomplished through both legislation like the Clean Air Act and
volunteerism from humans impacted by emissions.

Answer- The correct answer is C. Choice A is too opinionated. The paragraph does not give a call to action in any
way. Choice B is too narrow, as it fails to mention the negative impact that cleaning up the environment can have.
Choice D is off topic, although it's tricky because it uses verbiage from the paragraph. The paragraph does not give a
lesson in cleaning up the environment. Choice C is correct because it encompasses the gist of the entire paragraph
without being too narrow or too broad.

Inference
Writers make direct statements, but adult readers “read between the lines” by applying reasoning to what
is said to understand more completely what is meant. In daily reading, your interpretation is based on the
facts presented and your own experiences; however, on a test passage, when you evaluate inferences,
implications, and conclusions, you must limit your logical guesses to the information given.

Inference questions are demanding because they ask you to show that you understand what you read
AND that you can apply logical thought to its content. To choose the best answer, select the inference,
implication, or conclusion that is most logical, based on the information provided.

Inference questions contain characteristic language including: implies, suggests, apparently, probably, may
be, appears, seems, should, could, may be attributed to, can conclude, can be inferred, and implies that.

Example –

On the surface, the conquest of the Aztec empire by Herman Cortes is one of the most amazing military
accomplishments in history. With a small fighting force numbering in the hundreds, Cortes led the Spanish
explorers into victory against an Aztec population that many believe topped 21 million. In light of such a
seemingly impossible victory, the obvious question is: how did a small group of foreign fighters manage to
topple one of the world's strongest, wealthiest, and most successful military empires?

Several factors led to Cortes' success. First, the Spanish exploited animosity toward the Aztecs among rival
groups and convinced thousands of locals to fight. In one account of a battle, it is recorded that at least
200,000 natives fought with Cortes. Next, the Spanish possessed superior military equipment in the form

114
of European cannons, guns, and crossbows, leading to effective and efficient disposal of Aztec defenses.
For example, Spanish cannons quickly defeated large Aztec walls that had protected the empire against big
and less technically advanced armies.

Despite the Spanish advantages, the Aztecs probably could have succeeded in defending their capital city
of Tenochtitlan had they leveraged their incredible population base to increase their army's size and
ensured that no rogue cities would ally with Cortes. In order to accomplish this later goal, Aztec leader
Motecuhzoma needed to send envoys to neighbouring cities telling their inhabitants about the horrors of
Spanish conquest and the inevitability of Spanish betrayal.

In addition, the Aztecs should have exploited the fact that the battle was taking place on their territory. No
reason existed for the Aztecs to consent to a conventional battle, which heavily favoured the Spanish.
Motecuhzoma's forces should have thought outside the box and allowed Cortes into the city, only to
subsequently use hundreds of thousands of fighters to prevent escape and proceed in surprise "door-to-
door" combat. With this type of battle, the Aztecs would have largely thwarted Spanish technological
supremacy. However, in the end, the superior weaponry of the Spanish, the pent-up resentment of Aztec
rivals, the failure of Aztec diplomacy, and the lack of an unconventional Aztec war plan led to one of the
most surprising military outcomes in the past one thousand years.

Q. The passage implies that which of the following constituted the reason for Cortes' success seeming so
impressive?

A) Utilization of technological supremacy

B) Ability to form a coalition of local tribes to fight the Aztecs

C) The vast quantities of gold and other precious metals obtained in conquest

D) The large disparity in the size of the Aztec and Spanish militaries

E) Spanish speed and brutality in battle

Answer- Correct Answer: D

The relevant portion of the passage is: "one of the most amazing military accomplishments in history. With
a small fighting force..."

A. The passage states that this was a reason that Cortes' victory was not as impressive as it originally
appeared.

B. The passage states that this is an action that the Aztecs should have taken to protect against Cortes.

C. The passage makes no mention of this.

D. This matches the author's description of why Cortes' success seemed so impressive.

E. The author does not mention or imply this as a reason for Cortes' success seeming impressive.

Vocabulary

115
Almost all readers encounter words that are unfamiliar to them but find that the meanings of these words
are apparent by how they are used in the passage. The context, the information in surrounding words and
sentences, can give clues to the meaning of word. For example, when you read,

“The scientist alluded to ethical issues surrounding cloning, but he never discussed any of them directly,”

You can reasonably guess that alluded to means made indirect acknowledgment of. Furthermore, how the
word is used, its part of speech, sometimes supplies the clue to its meaning. For example, consider this
sentence

“The proletariat seem satisfied, but just three days later the workers demonstrated again.”

Proletariat is used as a subject; the next subject is workers; and, as the second word group is continuing
the thought of the first, the structural context suggests the two words “proletariat” and “workers” have
virtually the same meaning.

Example - Read the passage and selects the best answer, the meaning of the word as used by the writer
in the passage.

The parade streamed down the broad avenue, and the urchins screamed with glee. While adults enjoy a
good display children adore one.

The term urchins means

a) party-goers.

b) Assembled watchers.

c) Marchers.

d) Playful kids

Answer - d

a. As the passage is about a parade, not a party, this is NOT the best answer.

b. As the writer differentiates between at least two types of parade watchers, this is NOT the best answer.

c. As the writer refers to observing rather than participating in a parade, this is NOT the best answer.

d. As the second sentence makes a distinction between the enjoyment level of children and adults, this IS
the best answer.

116
Tutorial
Intermediate

Read each passage and select the best answer.

Q1. No one should hold high elective office with first consenting to have lived a public, public life and a
public private life. By this I mean that all worthy candidates should be willing to have their public service
scrutinized for corrupt dealings from the very start and that their private lives should be a similar open
book: what we the voters see should be what the candidates truly are. Any office holder who refuses, as
well as aspirant who fails the test should be subject to immediate recall proceedings or be otherwise
disqualified.

The passage implies that

a) Corrupt politicians are in power.

b) Some people have public and private records adequate to have high elective office.

c) The country is in trouble.

d) The country is not in trouble.

Q2. I’d followed that streaming, rolling wreck down the lonesome highway, ever alert to dodging the bits
and pieces that friskily flew off it. I’d rented a new V-8, but when my speedometer hit 100, and I still
couldn’t gain on them, I’d given up hopes of passing, of leaving that disaster in my dust, and settled into
monitoring. Muffler parts exited as we left Phoenix. Chrome broke free of rusted rivets fifty miles north.
The hood ornament nearly got me just south of the Grand Canyon. I praised the skies when the rubber
started flying my way as we approached the canyon, niftily eluded the blowout rubble, and smiled as the
FINALLY gave up and pulled over, losing more of their muffler in the process.

It can be inferred that

a) The lead car has been totally neglected for years.

b) The narrator is afraid of speed.

c) The lead car has an engine that has been taken care of.

d) The narrator das a lot more money that the lead car owners do.

Q3. A child with Asperger's Syndrome may move into the personal space of others, failing to recognize
body language and even verbal cues that he or she has transgressed. Friends and new acquaintances alike
may be acknowledged with tight and enthusiastic hugs. Bypassing typical greetings like "Hi, how are you?"
an older child with Asperger's may launch into discourse on the latest topic of concern. This narrow special
interest (e.g., Civil War maritime strategies, accuracy of train schedules) may be age-inappropriate (e.g.,

117
"Power Rangers" cartoons at age 16) or boring, but a child with this disorder will often continue to clarify,
oblivious to disinterest, "looks," or snickers from the people close by.

Which of the following statements best expresses the main idea of the paragraph?

A. Asperger's Syndrome is a disorder that affects an afflicted child's social interactions.

B. Asperger's Syndrome is a disorder that affects many different aspects of an afflicted child's life.

C. Asperger's Syndrome is a disorder causing an afflicted child to have narrow special interests that are
often age-inappropriate.

D. Asperger's Syndrome is a disorder causing an afflicted child to act especially reserved around new
experiences and people.

Q4. Recently, a $114 million bond was approved which will be used in the upcoming year to renovate
existing schools and build new schools in the North Point school district. Additions will also be built onto
the three high schools, three new elementary schools will be built, and a Career Center will also be
constructed for high school students on the campus of North Point Community College. In addition,
technology in the district will be brought up to date, teacher's will receive bonuses denied last year, and
two new vice principals will be hired for Smithson Middle School and Harry P. Shulte Middle School.

Which of the following statements best expresses the main idea of the paragraph?

A. North Point School District is going to be going through major changes in the upcoming year.

B. North Point School District has received bond money to build new schools and update technology in the
upcoming year.

C. North Point School District is going to be going through changes which will take place in the spring of
next year.

D. North Point School District has received bond money which will be used to upgrade the entire school
district in many ways this upcoming year.

Q5. Often, students are not capable of performing within a designated range of expectations in school.
Those students who hover below the normative range due to physical or cognitive impairments must be
evaluated to determine if special services or accommodations are needed. High schools are required to
offer a wide array of special services to those students who are evaluated as incapable of staying within
the normative range of performance standards in the classroom. The special services offered can help
these students reach their maximum potential, but before they can be administered, a teacher has to
identify that there is a problem.

Which of the following statements best expresses the main idea of the paragraph?

A. High schools are required to help students with special needs achieve their maximum potential.

B. High schools are required to help students who have been identified by a teacher as having special
needs, achieve their maximum potential.

118
C. High schools serve the needs of various types of students, including those who have been identified as
having special needs.

D. High schools serve the needs of various types of students, but only offer special services to those who
have been identified as having cognitive impairments.

Q6. Legends make children dream of princesses riding bare back through fields of fragrant heather. They
immortalize noble knights, champions for humanity, who fought tyranny as they raced through a village
with sword brandished and banner waving in the afternoon breeze. Legends awaken imagination in the
young and spur creativity in the old. The famous and trustworthy story of King Arthur is one of those great
legends made famous through repeated telling. According to the stories about King Arthur, he was a man
of valor, integrity, and honor in the face of challengers and opposition. He strove to lead all of England
down the path of goodness and prosperity without any foolish moves or attacks of cowardice. He
supposedly did all of these wonderful things, but as it seems, not one person knows how this legend came
to be, and some even debate that he was once alive.

Which of the following statements best expresses the main idea of the paragraph?

A. King Arthur's legends have been inspirational for both the old and the young.

B. Legends, like that of King Arthur, can be inspirational despite their muddled beginnings or deficient
proof.

C. Legends can inspire both the young and the old.

D. Legends, like that of King Arthur, are inspirational to those who want to believe in fairy tales.

Q7. While some readers find Hemingway terse to the point of being Unliterary, most entirely disagree.
They believe a few words, if they are the best words, tell the most.

Terse means

a) profane.

b) brief .

c) slangy .

d) tense .

Q8. . The discovery of the missing Benjamin Franklin manuscript on the Fourth of the July was pure
serendipity, for John was rummaging in the attic only to look for his father’s flag. Nevertheless, the town
fathers were ecstatic over the timing.

Serendipity means

a) an action productive of demanding a very great price.


119
b) aptitude for making accidental and timely discoveries.

c) a magnificent display of genuine patriotism.

d) a heartfelt contribution to community history.

Q9. The clever criminals lived off the land ever after to the great puzzlement of all. They succeeded in
eluding capture every time lawmen appeared by hiding in their elaborate tunnels.

Eluding means

a) aiding.

b) publicizing.

c) escaping.

d) excavating.

Q10. “Uncle” said Luke to the old Sean “You seem to be well fed, though I know no one looks after you.
Nor have I seen you leave your residence at any time. Tell me how you manage it?” “Because” Sean
replied, “I have a good feed every night at Emperor’s orchard. After dark, I go there myself and pick out
enough fruits to last a fortnight.”Luke proposed to accompany his uncle to the orchard. Though reluctant
because of Luke’s habit of euphoric exhibition of extreme excitement, Sean agreed to take him along. At
the orchard while Sean hurriedly collected the fruits and left, Luke on the other hand at the sight of
unlimited supply of fruits was excited and lifted his voice which brought Emperor’s men immediately to his
side. They seized him and mistook him as the sole cause of damage to the orchard. Although Luke
reiterated that he was a bird of passage, they pounded him mercilessly before setting him free.

Which of the following can be inferred from the passage?

a. Luke did not take good care of his uncle

b. Emperor was a wicked man

c. Lack of self-control had put Luke into trouble

d. Luke had a habit of speaking loudly

Q11. “Uncle” said Luke to the old Sean “You seem to be well fed, though I know no one looks after you.
Nor have I seen you leave your residence at any time. Tell me how you manage it?” “Because” Sean
replied, “I have a good feed every night at Emperor’s orchard. After dark, I go there myself and pick out
enough fruits to last a fortnight.”Luke proposed to accompany his uncle to the orchard. Though reluctant
because of Luke’s habit of euphoric exhibition of extreme excitement, Sean agreed to take him along. At
the orchard while Sean hurriedly collected the fruits and left, Luke on the other hand at the sight of
unlimited supply of fruits was excited and lifted his voice which brought Emperor’s men immediately to his
side. They seized him and mistook him as the sole cause of damage to the orchard. Although Luke
reiterated that he was a bird of passage, they pounded him mercilessly before setting him free.

Choose the word which is most nearly the same in meaning as the word “Reluctant” used in the passage.

120
a. Against

b. Resistant

c. Opposed

d. Disinclined

Q12. It is important for shipping companies to be clear about the objectives for maintenance and materials
management – as to whether the primary focus is on service level improvement or cost minimization.
Often when certain systems are set in place, the cost minimization objective and associated procedure
become more important than the flexibility required for service level improvement. The problem really
arises since cost minimization tends to focus on out of pocket costs which are visible, while the opportunity
costs, often greater in value, are lost sight of.

Choose the summary (four given below) that best captures the essence of the paragraph.

A: Shipping companies have to either minimize costs or maximize service quality. If they focus on cost
minimization, they will reduce quality. They should focus on service level improvement, or else opportunity
costs will be lost sight of.

B: Shipping companies should focus on quality level improvement rather than cost cutting. Cost cutting will
lead to untold opportunity costs. Companies should have systems in place to make the service level
flexible.

C: Shipping companies should determine the primary focus of their maintenance and materials
management. Focus on cost minimization may reduce visible costs, but ignore greater invisible costs and
impair service quality.

D: Any cost minimization program in shipping is bound to lower the quality of service. Therefore, shipping
companies must be clear about the primary focus of their maintenance and materials management before
embarking on cost minimization.

Q13. As the economic role of multinational, global corporations expands, the international economic
environment will be shaped increasingly not by governments or international institutions, but by the
interaction between governments and global corporations, especially in the United States, Europe, and
Japan. A significant factor in this shifting world economy is the trend toward regional trading blocs of
nations, which has a potentially large effect on the evolution of the world trading system. Two examples of
this trend are the United States-Canada Free Trade Agreement (FTA) and Europe 1992, the move by the
European Community (EC) to dismantle impediments to the free flow of goods, services, capital, and labor
among member states by the end of 1992. However, although numerous political and economic factors
were operative in launching the move to integrate the EC’s markets, concern about protectionism within
the EC does not appear to have been a major consideration. This is in sharp contrast to the FTA; the
overwhelming reason for that bilateral initiative was fear of increasing United States protectionism.
Nonetheless, although markedly different in origin and nature, both regional developments are highly
significant in that they will foster integration in the two largest and richest markets of the world, as well as
provoke questions about the future direction of the world trading system.

121
The primary purpose of the passage as a whole is to propose its continuance

(A) describe an initiative and

(B) chronicle a development and illustrate its inconsistencies

(C) identify a trend and suggest its importance

(D) summarize a process and question its significance

Q14. Disequilibrium at the interface of water and air is a factor on which the transfer of heat and water
vapour from the ocean to the air depends. The air within about a millimetre of the water is almost
saturated with water vapour and the temperature of the air is close to that of the surface water.
Irrespective of how small these differences might be, they are crucial, and the disequilibrium is maintained
by air near the surface mixing with air higher up, which is typically appreciably cooler and lower in water
vapour content. The turbulence, which takes its energy from the wind mixes the air. As the speed of wind
increases, so does the turbulence, and consequently the rate of heat and moisture transfer. We can arrive
at a detailed understanding of this phenomenon after further study. The transfer of momentum from wind
to water, which occurs when waves are formed, is an interacting-and complicated phenomenon. When
waves are made by the wind, it transfers important amounts of energy-energy, which is consequently not
available for the production of turbulence.

This passage principally intends to:

A. resolve a controversy

B. attempt a description of a phenomenon

C. sketch a theory

D. reinforce certain research findings

E. tabulate various observations

Q15. Some modern anthropologists hold that biological evolution has shaped not only human morphology
but also human behaviour. The role those anthropologists ascribe to evolution is not of dictating the
details of human behaviour but one of imposing constraints - ways of feeling, thinking, and acting that
''come naturally'' in archetypal situations in any culture. Our ''frailties'' - emotions and motives such as
rage, fear, greed, gluttony, joy,lust, love-may be a very mixed assortment quality: we are, as we say, ''in the
grip'' of them. And thus they give us our sense of constraints.

Unhappily, some of those frailties our need for ever-increasing security among them is presently
maladaptive. Yet beneath the overlay of cultural detail, they, too, are said to be biological in direction, and
therefore as natural to us as are our appendixes. We would need to comprehend thoroughly their adaptive
origins in order to understand how badly they guide us now. And we might then begin to resist their
pressure.

It can be inferred that in his discussion of maladaptive frailties the author assumes that

122
A. Evolution does not favor the emergence of adaptive characteristics over the emergence of
maladaptive ones

B. Any structure or behavior not positively adaptive is regarded as transitory in evolutionary theory

C. Maladaptive characteristics, once fixed, make the emergence of other maladaptive characteristics
more likely

D. The designation of a characteristic as being maladaptive must always remain highly tentative

E. Changes in the total human environment can outpace evolutionary change.

Q16. Some modern anthropologists hold that biological evolution has shaped not only human morphology
but also human behaviour. The role those anthropologists ascribe to evolution is not of dictating the
details of human behaviour but one of imposing constraints - ways of feeling, thinking, and acting that
''come naturally'' in archetypal situations in any culture. Our ''frailties'' - emotions and motives such as
rage, fear, greed, gluttony, joy,lust, love-may be a very mixed assortment quality: we are, as we say, ''in the
grip'' of them. And thus they give us our sense of constraints.

Unhappily, some of those frailties our need for ever-increasing security among them is presently
maladaptive. Yet beneath the overlay of cultural detail, they, too, are said to be biological in direction, and
therefore as natural to us as are our appendixes. We would need to comprehend thoroughly their adaptive
origins in order to understand how badly they guide us now. And we might then begin to resist their
pressure.

The primary purpose of the passage is to present

A. A position on the foundations of human behaviour and on what those foundations imply

B. A theory outlining the parallel development of human morphology and of human behaviour

C. A diagnostic test for separating biologically determined behaviour patterns from culture - specific
detail

D. An overview of those human emotions and motive's that impose constraints on human behaviour

E. A practical method for resting the pressures of biologically determined drives.

Q17. More than a century later, the Earth seems to be literally falling to pieces — recent environmental
setbacks include billions of tones of ice shelves breaking off in the Antarctic and unusually warm
temperatures in different parts of the world. Panic reactions range from predictions of sinking islands to
lamenting the ill-effects of global warming induced by release of greenhouse gases into the atmosphere.
The trouble is that we are too obsessed with the climate change problem to even acknowledge the fact
that the state of the planet hinges on much more. Climate change is at best a symptom of a far more
complex malaise, just as a fever is most often only an indicator of something that's gone awry in our body.
It's time for a complete and comprehensive planetary health check, that will examine the impacts of
change in land use, loss of biodiversity, use of fertilizers and pesticides and consistent pollution of water
bodies. This would overcome the limitations of evaluating how ecosystems work by reacting to just one
major environmental concern as is happening in the case of global warming. These considerations have
123
been responsible for the setting up of an international panel, the Millennium Ecosystem Assessment.
Financed by four major international bodies, including the UN and the World Bank, the eco-panel was set
up without much fuss last June, and is expected to determine, over a period of four years and at a cost of
$21 million, the state of the Earth's-ecosystems. The eco-panel will source inputs from more than 2,000
natural and social scientists the world over. Put simply, the Earth will go through the equivalent of a
thorough physical, so that biological, economic and social information can be collated to help scientists
arrive at a final diagnosis. The newly constituted eco -panel will have to ensure that data collection is more
representative of the regions of the world. Today, we have the advantage of sourcing data from remote
sensing satellites as well. The information thus gathered would have to be sorted out and analysed by
specialists and also by generalists.

Which of the following word is similar to the word ―lamenting?

(A) Applauding (B) Rejoicing (C) Contended (D) Grieving (E) None of these

Q18. More than a century later, the Earth seems to be literally falling to pieces — recent environmental
setbacks include billions of tones of ice shelves breaking off in the Antarctic and unusually warm
temperatures in different parts of the world. Panic reactions range from predictions of sinking islands to
lamenting the ill-effects of global warming induced by release of greenhouse gases into the atmosphere.
The trouble is that we are too obsessed with the climate change problem to even acknowledge the fact
that the state of the planet hinges on much more. Climate change is at best a symptom of a far more
complex malaise, just as a fever is most often only an indicator of something that's gone awry in our body.
It's time for a complete and comprehensive planetary health check, that will examine the impacts of
change in land use, loss of biodiversity, use of fertilizers and pesticides and consistent pollution of water
bodies. This would overcome the limitations of evaluating how ecosystems work by reacting to just one
major environmental concern as is happening in the case of global warming. These considerations have
been responsible for the setting up of an international panel, the Millennium Ecosystem Assessment.
Financed by four major international bodies, including the UN and the World Bank, the eco-panel was set
up without much fuss last June, and is expected to determine, over a period of four years and at a cost of
$21 million, the state of the Earth's-ecosystems. The eco-panel will source inputs from more than 2,000
natural and social scientists the world over. Put simply, the Earth will go through the equivalent of a
thorough physical, so that biological, economic and social information can be collated to help scientists
arrive at a final diagnosis. The newly constituted eco -panel will have to ensure that data collection is more
representative of the regions of the world. Today, we have the advantage of sourcing data from remote
sensing satellites as well. The information thus gathered would have to be sorted out and analysed by
specialists and also by generalists.

Which of the following word is similar to the word ―obsessed?

(A) Unconcerned (B) Possessed (C) Uncontrolled (D) Unenthusiastic (E) None of these

Q19. The need for knowledge of the constitution and functions, in the concrete, of human nature is great
just because the teacher's attitude to subject matter is so different from that of the pupil. The teacher
presents in actuality what the pupil represents only in posse. That is, the teacher already knows the things
which the student is only learning. Hence the problem of the two is radically unlike. When engaged in the
direct act of teaching, the instructor needs to have subject matter at his fingers' ends; his attention should

124
be upon the attitude and response of the pupil. To understand the latter in its interplay with subject
matter is his task, while the pupil's mind, naturally, should be not on itself but on the topic in hand. Or to
state the same point in a somewhat different manner: the teacher should be occupied not with subject
matter in itself but in its interaction with the pupils' present needs and capacities. Hence simple
scholarship is not enough.

It can be inferred from the paragraph that:

A. simple scholarship is lop-sided in its focus

B. simple scholarship might overlook the subject matter and focus purely on the student

C. simple scholarship, with a narrow focus, might not place enough emphasis on the student’s needs

D. simple scholarship will always be lopsided in nature because of its ambiguous stand on problems

Q20. An increase in the level of serotonin levels in the human body is known to significantly enhance the
mood of the person and in some cases, help people overcome depression. Serotonin taken orally does not
pass into the pathways of the central nervous system, because it does not cross the blood–brain barrier.
However, tryptophan and its metabolite 5-hydroxytryptophan (5- HTP), from which serotonin is
synthesized, does cross the blood–brain barrier. These agents are available as dietary supplements, and
may be effective serotonergic agents.

Which of the following can be correctly inferred from the statements above?

A. Individuals who do not consume enough tryptophan can develop depressive tendencies

B. Individuals who consume tryptophan can metabolize enough serotonin and do not run the risk of
depression

C. Consumption of serotonin laced foods can help cure depression in some cases

D. Tryptophan is an effective agent that can help cure depression in some cases

Advance

PASSAGE 1- Marketing executives in television work with a relatively stable advertising medium. In many
ways, the television ads aired today are similar to those aired two decades ago. Most television ads still
feature actors, still run 30 or 60 seconds, and still show a product. However, the differing dynamics of the
Internet pose unique challenges to advertisers, forcing them to adapt their practices and techniques on a
regular basis.

In the early days of Internet marketing, online advertisers employed banner and pop-up ads to attract
customers. These techniques reached large audiences, generated many sales leads, and came at a low
cost. However, a small number of Internet users began to consider these advertising techniques intrusive
and annoying. Yet because marketing strategies relying heavily on banners and pop-ups produced results,
companies invested growing amounts of money into purchasing these ad types in hopes of capturing
market share in the burgeoning online economy. As consumers became more sophisticated, frustration
125
with these online advertising techniques grew. Independent programmers began to develop tools that
blocked banner and pop-up ads. The popularity of these tools exploded when the search engine Google, at
the time an increasingly popular website fighting to solidify its place on the Internet with giants Microsoft
and Yahoo, offered free software enabling users to block pop-up ads. The backlash against banner ads
grew as new web browsers provided users the ability to block image-based ads such as banner ads.
Although banner and pop-up ads still exist, they are far less prominent than during the early days of the
Internet.

A major development in online marketing came with the introduction of pay-per-click ads. Unlike banner
or pop-up ads, which originally required companies to pay every time a website visitor saw an ad, pay-per-
click ads allowed companies to pay only when an interested potential customer clicked on an ad. More
importantly, however, these ads circumvented the pop-up and banner blockers. As a result of these
advantages and the incredible growth in the use of search engines, which provide excellent venues for pay-
per-click advertising, companies began turning to pay-per-click marketing in droves. However, as with the
banner and pop-up ads that preceded them, pay-per-click ads came with their drawbacks. When
companies began pouring billions of dollars into this emerging medium, online advertising specialists
started to notice the presence of what would later be called click fraud: representatives of a company with
no interest in the product advertised by a competitor click on the competitor's ads simply to increase the
marketing cost of the competitor. Click fraud grew so rapidly that marketers sought to diversify their online
positions away from pay-per-click marketing through new mediums.

Although pay-per-click advertising remains a common and effective advertising tool, marketers adapted
yet again to the changing dynamics of the Internet by adopting new techniques such as pay-per-
performance advertising, search engine optimization, and affiliate marketing. As the pace of the Internet's
evolution increases, it seems all the more likely that advertising successfully on the Internet will require a
strategy that shuns constancy and embraces change.

Q1. The author implies what about the future of pay-per-performance advertising?

A) Although it improves on pay-per-click advertising, it is still vulnerable to click fraud

B) It will one day become extinct as Internet users discover drawbacks with it

C) Internet users will develop free software to block its effectiveness

D) It will eventually become less popular with advertisers as the Internet evolves and drawbacks
emerge

E) It will not face drawbacks due to its differing approach to online marketing

Q2. Which of the following most accurately states the main idea of the passage?

A) Although pay-per-click advertising remains a wide-spread and effective online advertising medium,
its popularity is likely to diminish as the Internet evolves.

B) Internet advertising is not well received by Internet users, causing independent programmers to
subvert advertisers.
126
C) Unlike the television, the Internet has experienced dramatic changes in short periods of time.

D) Unlike the television, the Internet has evolved rapidly, forcing online marketers to develop new
advertising strategies and mediums.

E) The pace of the Internet’s evolution is increasing and will only increase in the future.

PASSAGE 2- Shortly after September 11, 2001, the United States began requesting additional financial
information about persons of interest by subpoenaing records located at the SWIFT banking consortium.
SWIFT, which routes trillions of dollars a day, faced an ethical dilemma: fight the subpoenas in order to
protect member privacy and the group's reputation for the highest level of confidentiality, or, comply and
provide information about thousands of financial communications in the hope that lives will be saved.
SWIFT decided to comply in secret, but in late June 2006, four major U.S. newspapers disclosed SWIFT's
compliance. This sparked a heated public debate over the ethics of SWIFT's decision to reveal ostensibly
confidential financial communications.

Analyzing the situation in hindsight, three ethical justifications existed for not complying with the Treasury
Department's requests. First, SWIFT needed to uphold its long-standing values of confidentiality, non-
disclosure, and institutional trust. The second ethical reason against SWIFT's involvement came with
inadequate government oversight as the Treasury Department failed to construct necessary safeguards to
ensure the privacy of the data. Third, international law must be upheld and one could argue quite strongly
that the government's use of data breached some parts of international law.

Although SWIFT executives undoubtedly considered the aforementioned reasons for rejecting the
government's subpoena, three ethical justifications for complying existed. First, it could be argued that the
program was legal because the United States government possesses the authority to subpoena records
stored within its territory and SWIFT maintained many of its records in Virginia. Second, it is entirely
possible that complying with the government's subpoena thwarted another catastrophic terrorist attack
that would have cost lives and dollars. Third, cooperating with the government did not explicitly violate
any SWIFT policies due to the presence of a valid subpoena. However, the extent of cooperation certainly
surprised many financial institutions and sparked some outrage and debate within the financial
community.

While SWIFT had compelling arguments both for agreeing and refusing to cooperate with the U.S.
government program, even in hindsight, it is impossible to judge with certitude the wisdom and ethics of
SWIFT's decision to cooperate as we still lack answers to important questions such as: what information
did the government want? What promises did the government make about data confidentially? What, if
any, potentially impending threats did the government present to justify its need for data?

Q3. The primary purpose of the passage is to:

A) Explain the extent of SWIFT's cooperation with the U.S. government in tracking financial
transactions

B) Trace the origin of public outrage, especially among international financial institutions, over
SWIFT's cooperation with U.S. authorities
127
C) Argue that the absence of information and the presence of ethical complexities make judging
SWIFT's decision difficult at best

D) Argue that SWIFT's decision amounted to a compromise of SWIFT's principles

E) Provide historical background on a difficult ethical dilemma

Q4. Which of the following can be inferred from the passage?

A) No clear cut answer as to the legality of SWIFT's cooperation existed

B) SWIFT failed to adequately consult its legal staff before deciding to cooperate

C) The volume of money routed through SWIFT declined after its cooperation became public

D) U.S. authorities threatened criminal charges if SWIFT refused their subpoenas

E) Treasury Department officials objected to the publication of information about its classified
program

Q5. The author most likely used the word "ostensibly" near the end of the first paragraph to emphasize
that:

A) The ethical decision facing SWIFT seemed complex

B) The U.S. government saw no problems in the request it made

C) Four major U.S. newspapers thought on the surface that the decision to publish the story was
simple

D) Users of SWIFT believed their financial data and communications were private

E) Members of the public reacted with anger upon learning of SWIFT’s cooperation

PASSAGE 3- Although websites such as Facebook and MySpace experienced exponential growth during the
middle of the first decade of the 21st century, some users remain oblivious to the fact that the information
they post online can come back to haunt them. First, employers can monitor employees who maintain a
blog, photo diary, or website. Employers can look for controversial employee opinions, sensitive
information disclosures, or wildly inappropriate conduct. For example, a North Carolina newspaper fired
one of its features writers after she created a blog on which she anonymously wrote about the
idiosyncrasies of her job and coworkers.

The second unintended use of information from social networking websites is employers who check on
prospective employees. A June 11, 2006 New York Times article reported that many companies recruiting
on college campuses use search engines and social networking websites such as MySpace, Xanga, and
Facebook to conduct background checks. Although the use of MySpace or Google to scrutinize a student’s
background is somewhat unsettling to many undergraduates, the Times noted that the utilization of
Facebook is especially shocking to students who believe that Facebook is limited to current students and
recent alumni.
128
Corporate recruiters and prospective employers are not the only people interested in college students’
lives. The third unintended use of social networking websites is college administrators who monitor the
Internet—especially Facebook—for student misconduct. For example, a college in Boston’s Back Bay
expelled its student Government Association President for joining a Facebook group highly critical of a
campus police sergeant. In addition, fifteen students at a state university in North Carolina faced charges in
court for underage drinking because of photos that appeared on Facebook.

Although more users of websites such as Facebook are becoming aware of the potential pitfalls of online
identities, many regular users still fail to take three basic security precautions. First, only make your
information available to a specific list of individuals whom you approve. Second, regularly search for
potentially harmful information about yourself that may have been posted by mistake or by a disgruntled
former associate. Third, never post blatantly offensive material under your name or on your page as,
despite the best precautions, this material will likely make its way to the wider world. By taking these
simple steps, members of the digital world can realize the many benefits of e-community without
experiencing some of the damaging unintended consequences.

Q6. The primary purpose of the passage is to:

A) Explain the growth of the digital world through the lens of privacy

B) Discuss the risks of the digital world and suggest ways to protect yourself

C) Evaluate the pros and cons of active participation in the digital world

D) Propose steps Facebook, MySpace, and Google can take to protect user privacy

E) Illustrate potential unintended uses of private information

Q7. Based upon the passage, the author implies which of the following:

A) Information obtained unwillingly from the Internet is permissible in court

B) It is impossible to protect yourself from unintended uses of information online

C) Making information available only to people whom you trust compromises your online community

D) Even if you restrict who can view your data, the government may still access it

E) Done properly, posting prurient information about oneself poses no substantial risk

PASSAGE 4- The new financial secretary Donald Tsang has said he is committed to the principle and
philosophy of financial management enshrined by his predecessor Sir Hamish Macleod. It was really not
surprising to hear such a message as our Government has consistently reiterated its noninterventionist
policy. Our Government is proud of the low taxation policy, free market operations and the enterprising
nature of the economy, as it believes that all these lay the strong financial foundations that make possible
the economic boom in Hong Kong. Hence, the role of the Government is to interfere, as little as possible, in
the public sector, for fear of damaging the "normal" operation of the market. Government spending in
money terms had never exceeded 20 per cent of the Gross Domestic Product (GDP), since 1945. Many
people in Hong Kong are scared by the spectre of unemployment and yet the Government has done

129
nothing to solve the problem, because administration officials do not want to spoil the miraculous market
mechanism. There is talk of "self-help" by entrepreneurs involved in the market. Meanwhile, the
unemployment rate is rocketing; I believe the Government should act to combat this problem. Owing to
the lack of a comprehensive unemployment security scheme and unemployment insurance which exist in
western industrial countries (for example, the US and Germany), Hong Kong's jobless suffer more
psychological and social pressure than their Western Counterparts. The unemployed have a heavy
psychological burden to bear. This can lead to them having feeling of low self-esteem and a feeling that
they are worthless. Those who seek help from the Social Welfare Department (SWD) have to endure the
complicated application procedure and the inhuman and harsh means test. The procedure to apply for
assistance is not as easy as one would imagine. It puts the unemployed under great pressure. I do not
believe that maintaining the policy of minimal government intervention will stem rising unemployment.
The Government should make its "invisible" hand in the market become visible by, for example, creating
more job opportunities for the unemployed; ensuring age and sex discrimination do not exist in the
workplace, restraining employers from exploiting employees, in the way that some employers do by taking
on casual staff and making them work long hours and, finally,. bringing under control the "crazy" property
speculation of capitalists

Q8. What is the main point of paragraph?

(A) To analyze then problem

(B) To give reasons for the call for action

(C) To describe the pattern of government spending

(D) To describe the operation of the free market.

(E) None of these

Q9. In passage 'endure' could be replaced by which of the following?

(A) Enjoy

(B) Wait for

(C) Demanding

(D) Put up with

(E) None of these

Q10. In passage ' reiterated ' could be replaced by which of the following?

(A) Take back

(B) Frequent

(C) Restate

(D) Rearrange

130
(E) None of these

PASSAGE 5- The economic transformation of India is one of the great business stories of our time. As
stifling government regulations have been lifted, entrepreneurship has flourished, and the country has
become a high-powered center for information technology and pharmaceuticals. Indian companies like
Infosys and Wipro are powerful global players, while Western firms like G.E. and I.B.M. now have major
research facilities in India employing thousands. India's seemingly endless flow of young, motivated
engineers, scientists, and managers offering developed-world skills at developing-world wages is held to be
putting American jobs at risk, and the country is frequently heralded as "the next economic superpower"

But India has run into a surprising hitch on its way to superpower status: its inexhaustible supply of
workers is becoming exhausted. Although India has one of the youngest workforces on the planet, the
head of lnfosys said recently that there was an "acute shortage of skilled manpower and a study projects
that this year salaries for skilled workers will rise fourteen and a half per cent, a sure sign that demand for
skilled labor is outstripping supply.

How is this possible in a country that every year produces two and a half million college graduates and four
hundred thousand engineers? Start with the fact that just ten per cent of Indians get any kind of post-
secondary education, compared with some fifty per cent who do in the U.S. Moreover, of that ten per cent,
the vast majority go to one of India's seventeen thousand colleges, many of which are closer to community
colleges than to four-year institutions. India does have more than three hundred universities, but a recent
survey by the London Times Higher Education Supplement put only two of them among the top hundred in
the world. Many Indian graduates therefore enter the workforce with a low level of skills. A current study
led by Vivek Wadhwa, of Duke University, has found that if you define "engineer' by U.S. standards, India
produces just a hundred and seventy thousand engineers a year, not four hundred thousand. Infosys says
that, of 1.3 million applicants for jobs last year, it found only two per cent acceptable.

India has taken tentative steps to remedy its skills famine-the current government has made noises about
doubling spending on education, and a host of new colleges and universities have sprung up since the mid-
nineties. But India's impressive economic performance has made the problem seem less urgent than it
actually is, and allowed the government to defer difficult choices. (In a country where more than three
hundred million people live on a dollar a day, producing college graduates can seem like a low priority.)
Ultimately, the Indian government has to pull off a very tough trick, making serious changes at a time when
things seem to be going very well. It needs, in other words, a clear sense of everything that can still go
wrong. The paradox of the Indian economy today is that the more certain it’s glowing future seems to be,
the less likely that future becomes.

Q11. Which of these could you infer according to the passage?

a. Wages in the developing countries are less as compared to wages in the developed countries

b. Wages in the developing countries are more as compared to wages in the developed countries

c. Wages in the developing countries are same as wages in the developed countries

d. None of these

Q12.What can you infer as the meaning of stifling from the passage ?
131
a. Democratic

b. Liberal

c. Impacting

d. Undemocratic

PASSAGE 6- The study of history traditionally has had fixed boundaries and focal points—periods,
countries, dramatic events, and great leaders. It also has had clear and firm notions of scholarly procedure:
how one inquiries into a historical problem, how one presents and documents one’s findings, what
constitutes admissible and adequate proof. Anyone who has followed recent historical literature can testify
to the revolution that is taking place in historical studies. The currently fashionable subjects come directly
from the sociology catalog: childhood, work, leisure. The new subjects are accompanied by new methods.
Where history once was primarily narrative, it is now entirely analytic. The old questions “What
happened?” and “How did it happen?” have given way to the question “Why did it happen?” Prominent
among the methods used to answer the question “Why” is psychoanalysis, and its use has given rise to
psychohistory. Psychohistory does not merely use psychological explanations in historical contexts.
Historians have always used such explanations when they were appropriate and when there was sufficient
evidence for them. But this pragmatic use of psychology is not what psychohistories intend. They are
committed, not just to psychology in general, but to Freudian psychoanalysis. This commitment precludes
a commitment to history as historians have always understood it. Psychohistory derives its “facts” not from
history, the detailed records of events and their consequences, but from psychoanalysis of the individuals
who made history, and deduces its theories not from this or that instance in their lives, but from a view of
human nature that transcends history. It denies the basic criterion of historical evidence: that evidence be
publicly accessible to, and therefore assessable by, all historians. And it violates the basic tenet of historical
method: that historians be alert to the negative instances that would refute their theses. Psychohistorians,
convinced of the absolute rightness of their own theories, are also convinced that theirs is the “deepest”
explanation of any event that other explanations fall short of the truth. Psychohistory is not content to
violate the discipline of history (in the sense of the proper mode of studying and writing about the past); it
also violates the past itself. It denies to the past an integrity and will of its own, in which people acted out
of a variety of motives and in which events had a multiplicity of causes and effects. It imposes upon the
past the same determinism that it imposes upon the present, thus robbing people and events of their
individuality and of their complexity. Instead of respecting the particularity of the past, it assimilates all
events, past and present, into a single deterministic schema that is presumed to be true at all times and in
all circumstances.

Q13. Which of the following best states the main point of the passage?

(A) The approach of psychohistorians to historical study is currently in vogue even though it lacks the rigor
and verifiability of traditional historical method.

(B) Areas of sociological study such as childhood and work are of little interest to traditional historians.

(C) The psychological assessment of an individual’s behavior and attitude is more informative than the
details of his or her daily life.

132
(D) History is composed of unique and non repeating events that must be individually analyzed on the basis
of publicly verifiable evidence.

(E) None of the above.

Q14. It can be inferred from the passage that the methods used by psychohistorians probably prevent
them from ___________

(A) presenting their material in chronological order

(B) producing a one-sided picture of an individual’s personality and motivations

(C) uncovering alternative explanations that might cause them to question their own conclusions

(D) offering a consistent interpretation of the impact of personality on historical events

(E) recognizing connections between a government’s political actions and the aspirations of government
leaders

PASSAGE 7- On the surface, the conquest of the Aztec empire by Herman Cortes is one of the most
amazing military accomplishments in history. With a small fighting force numbering in the hundreds,
Cortes led the Spanish explorers into victory against an Aztec population that many believe topped 21
million. In light of such a seemingly impossible victory, the obvious question is: how did a small group of
foreign fighters manage to topple one of the world's strongest, wealthiest, and most successful military
empires?

Several factors led to Cortes' success. First, the Spanish exploited animosity toward the Aztecs among rival
groups and convinced thousands of locals to fight. In one account of a battle, it is recorded that at least
200,000 natives fought with Cortes. Next, the Spanish possessed superior military equipment in the form
of European cannons, guns, and crossbows, leading to effective and efficient disposal of Aztec defenses.
For example, Spanish cannons quickly defeated large Aztec walls that had protected the empire against big
and less technically advanced armies.

Despite the Spanish advantages, the Aztecs probably could have succeeded in defending their capital city
of Tenochtitlan had they leveraged their incredible population base to increase their army's size and
ensured that no rogue cities would ally with Cortes. In order to accomplish this later goal, Aztec leader
Motecuhzoma needed to send envoys to neighboring cities telling their inhabitants about the horrors of
Spanish conquest and the inevitability of Spanish betrayal.

In addition, the Aztecs should have exploited the fact that the battle was taking place on their territory. No
reason existed for the Aztecs to consent to a conventional battle, which heavily favored the Spanish.
Motecuhzoma's forces should have thought outside the box and allowed Cortes into the city, only to
subsequently use hundreds of thousands of fighters to prevent escape and proceed in surprise "door-to-
door" combat. With this type of battle, the Aztecs would have largely thwarted Spanish technological
supremacy. However, in the end, the superior weaponry of the Spanish, the pent-up resentment of Aztec
rivals, the failure of Aztec diplomacy, and the lack of an unconventional Aztec war plan led to one of the
most surprising military outcomes in the past one thousand years.

133
Q15. The author implies which of the following about the Aztec view toward an unconventional military
confrontation of the Spanish?

A) The Aztecs did not consider it

B) The Aztecs considered it, but rejected it out of beliefs about how battles ought to be fought

C) The Aztecs considered this, but it was too late

D) The Aztecs were certain a victory could be achieved via traditional combat

E) The Aztecs felt the geography of Tenochtitlan did not favor this strategy

Q16. Which of the following best characterizes the main point the author is trying to convey in the
passage?

A) Aztec failure to fight an unconventional war led to an unnecessary defeat

B) Spanish victory was neither as impressive nor as surprising as it may first appear

C) Resentment toward the Aztecs led to their demise

D) Herman Cortes masterminded an amazing military accomplishment

E) The myopic vision of the Aztecs led to their unnecessary downfall

PASSAGE 8- On August 22, 1939, Adolf Hitler summoned his top military generals to Obersalzberg, where
he delivered a speech explaining his plans for war, first with Poland, then with the rest of Europe. Despite
resistance from those both inside and outside Germany, Hitler felt exceedingly confident that he could
defy the will of the international community and conquer vast amounts of land. In his speech at
Obersalzberg, he laid out numerous factors he believed would contribute to the success of his war plans.

Chief among Hitler's sources of confidence in Germany's brazen war plans was German military quickness.
Hitler said, "Our strength lies in our quickness." On the advice of Colonel-General von Brauchitsch, Hitler
believed Poland could be captured in a few weeks, an astonishingly short amount of time given the recent
history of trench warfare and the long history of protracted European military engagements that resulted
in minimal land gains and high casualty counts.

Hitler's confidence in the ability of the German military to inflict considerable brutality further
strengthened his determination to pursue an exceedingly ambitious plan of territorial aggrandizement. He
said, "I shall shoot everyone who utters one word of criticism" and noted that "the goal to be obtained in
the war is not that of reaching certain lines but of physically demolishing the opponent."In this vein, Hitler
ordered his military to "be hard, be without mercy, *and+ act more quickly and brutally than others…for it
scares the others off." Hitler believed that enemies, not used to this type of brutality, would surrender
quickly.

In addition to speed and brutality, Hitler believed that, in the end, history would overlook his inhumane
conduct. To support this view, which turned out to be anything but prescient, Hitler invoked a pollyannaish
view of Asian leader Genghis Kahn. In Hitler's eyes, Kahn "sent millions of women and children into death
knowingly and with a light heart," yet "history sees in him only the great founder of States."
134
Although Hitler brimmed with confidence and experienced initial yet widely-expected success in Poland
and then in Denmark, he overlooked important considerations. In many ways, Hitler made the same
mistake Napoleon Bonaparte made years earlier. Hitler believed he could advance further and conquer
Britain, yet, like Napoleon, Hitler did not adequately foresee the insurmountable barrier posed by Britain's
island status. Despite the damage inflicted at the hands of the German Luftwaffe during the Battle of
Britain (1940), British forces eventually won this important battle. Nevertheless, Hitler pressed on and, in
an even more fateful decision that carried echoes of a Napoleonic tactical misstep, invaded the USSR
where his forces suffered the decisive defeat of World War II at Stalingrad in 1943. In the end, Hitler's
reputation in history proved to be as brutal and decisive as the battle plans and philosophy he announced
at Obersalzberg.

Q17. The author of the passage is primarily concerned with explaining:

A) The logistics of Hitler's war strategy and the mechanics of its failure

B) The philosophy of Hitler's war strategy and the world's reaction

C) Why Hitler believed his war plans would succeed and why they eventually failed

D) Hitler's plans and their failure with an eye to pre-1900 history

E) Explaining the source of Hitler's brutality and the reasons for its failure

Q18. Which of the following best characterizes the author's view of the relationship between Hitler and
Napolean?

A) Governed with similar styles

B) Fought military conflicts with similar ideologies

C) In general, shared a legacy as overly ambitious leaders

D) At a high-level, some similarities in military missteps existed

E) Both suffered final defeats by impetuously charging east

Q19. According to the passage, why did Hitler believe he could conquer Poland in a few weeks?

A) The inaction of European neighbors

B) The example of Napoleon

C) The philosophy of Genghis Kahn

D) The counsel of a military general

E) The small size of Poland

PASSAGE 9- Prior to the fall of the Union of Soviet Socialist Republics (USSR), Mikhail Gorbachev, seeing a
country falling behind its Western rival and a people increasingly clamoring for change, addressed the
growing internal unrest in the summer of 1987 by introducing a series of reforms known as perestroika

135
(literally, restructuring). In Perestroika: New Thinking for Our Country and the World, Mikhail Gorbachev
discussed his analysis of the problems facing the USSR and his plans to solve them.

Perhaps the most pressing and visible problem facing the USSR in the last 1980s came in the form of the
country’s consistently mediocre economic performance, despite its vast natural resource wealth and large
labor force. Gorbachev flatly admitted that economic failures were increasing and current policies were
failing to offer a sustainable remedy. Failing to take advantage of the numerous scientific and technological
advancements available, the USSR relied on inefficient and outdated business models. As a result,
Gorbachev said, "in the last fifteen years the national income growth rates had declined by more than a
half and by the beginning of the eighties had fallen to a level close to economic stagnation." With business
executives focused on using more resources (in order to employ more people) instead of becoming more
efficient, the country produced poor quality products unable to compete in a global economy. Further, this
inefficiency led to shortages: "the Soviet Union, the world’s biggest producer of steel, raw materials, fuel
and energy, has shortfalls in them due to wasteful or inefficient use."

The decrepit economy engendered social unrest and woe that only compounded economic difficulties and
societal misery. Gorbachev wrote of "a gradual erosion of the ideological and moral values of our people"
and noted the considerable growth in "alcoholism, drug addiction and crime." Accentuating these
difficulties, the Communist government often ignored the needs of the average citizen, causing distrust
and resentment. Perhaps the most destructive element of the social unraveling and inadequate
government response was the mediocre education system. Gorbachev said, "Creative thinking was driven
out from the social sciences, and superfluous and voluntarist assessments and judgments were declared
indisputable truths."

Although Gorbachev also opined about the growing public disbelief in the content of the immense
government propaganda campaigns, the extent to which economic underdevelopment and social deviance
gripped Soviet culture made the collapse of the USSR virtually inevitable in the minds of many observers.
When combined with glasnost (literally, openness), Gorbachev’s plan that allowed greater transparency,
perestroika actually served to hasten the collapse of the USSR. Contrary to its purpose, perestroika
ensured that the fall of the USSR would occur sooner rather than later. Only a few years after Gorbachev
implemented changes that would have been unthinkable and antithetical to the philosophy of previous
leaders like Lenin, Stalin, and Khrushchev, the USSR fell.

Q20. Which of the following best describes the primary objective of the passage?

A) Argue that the implementation of perestroika caused the fall of the Soviet Union

B) Explain perestroika along with its roots and consequences

C) Analyze the pros and cons of Mikhail Gorbachev's decision to implement perestroika

D) Explain the short-falls of a communist system and offer remedies

E) Discuss the role of Mikhail Gorbachev in propelling the USSR towards ceasing to exist

Q21. The passage implies that which of the following was most true of the Soviet economy prior to
perestroika:

136
A) Suffered from underperformance due to excessive government regulation and micro-management

B) Failed to meet its potential as a result of corruption and bureaucratic overhead

C) Lacked adequate natural resources to grow efficiently, regardless of business management

D) Focused on achieving high-employment rather than export-capable products

E) With declining growth and stagnation, stood in the worst shape ever in USSR history

Q22. Based upon the passage, the author would likely agree most with which of the following
characterizations of the impact of the USSR's troubled economy during the days leading up to
perestroika?

A) Cause for renewed determination in communist philosophy

B) Reason that natives looked increasingly to the West and capitalism

C) Source of frustration and discomfort among citizens that fueled social friction

D) Justification for the USSR's neglect of the needs of many citizens

E) Primary cause of the USSR's poor educational system

Q23. In the context of the passage, the author most likely uses the word "unthinkable" (in the last
sentence) to help convey which of the following points about the changes Gorbachev implemented in
perestroika?

A) They would have never crossed the mind of Lenin as being conceptually possible, let alone
desirable or feasible

B) They would have been difficult for the mind of Lenin to comprehend intelligibly

C) They would have been seen by Lenin as undesirable and poor choices

D) They would have been seen by Lenin as incomprehensible yet appealing

E) They would have been considered highly desirable

Q24. Gorbachev offers all of the following as evidence of the need for perestroika EXCEPT:

A) Shortages in natural resources due to inefficiency

B) Declines in economic output and growth

C) Slides in moral values of citizens

D) Erosion of new and ingenious thinking

E) Frustrations with the results of past reform efforts

137
Unit 5

Narration
What is direct and indirect speech?

Direct and indirect speech is used frequently, both in writing and in everyday speech. Journalists use
quoted speech in their articles, and everyone uses the odd quote when relating an interesting story.

Reported speech can be found in business writing, journalistic writing, and again, in everyday speech.

Direct/Quoted speech uses quotation marks and the exact words that a person has said. Reported speech
(also called indirect speech) relates what the person said, but does not use the exact words. You often
need to change verbs and pronouns to keep the original meaning.

Quoted Speech (Direct Speech) Reported Speech (Indirect Speech)

“What time is the meeting?” Tom Tom asked what time the meeting was. Abdul
asked. Abdul replied, “It’s at 1 o’clock.” said that the meeting was at 1 o’clock. Sue
asked Pat what she had done on Sunday. Pat
Sue asked Pat, “What did you do on
said that she had visited her brother.
Sunday?” “I visited my brother,” Pat
answered

Glossary of Terms

Quotation Marks

Punctuation marks used to indicate that the text represents the exact words (direct speech) someone has
spoken. These punctuation marks vary from language to language.

Direct Speech

The exact words someone says, set within quotation marks.

e.g. “Will you run for President in the next election?” the reporter asked Hillary Clinton.

Indirect Speech

The reporting of what someone has said, not using his or her exact words.

e.g. The reporter asked Hillary Clinton if she would run for President in the next election.

Quoted Speech

138
Another term for direct speech.

Reported Speech

Another term for indirect speech.

Formal Reported Speech

Formal reported speech requires that you make specific changes to verb tenses that were spoken. In
general, the verbs move one step to the past. Like becomes liked; liked becomes had liked, for example.
Formal reported speech is used most often in writing and sometimes in speaking.

Reporting Verbs

Verbs used to indicate quoted or reported speech. There are several reporting verbs but in journalism the
most common is say.

e.g. A White House spokesman said Clinton would appear on CNN tomorrow.

Practice Exercise - Identifying Quoted and Reported Speech

Read the following paragraph and underline all instances of reported speech. Circle any quoted speech.

Maria recently returned from a conference in Dubai and told her colleagues about her trip there. She said
that the architecture of Dubai was fascinating, with many new buildings of glass and steel. Of course, she
also saw the Burj Khalifa, the world’s tallest building. Some of her friends went up to the top, but Maria
said she didn’t because she has a fear of heights. “I don’t even like being on a second-story balcony, so the
Burj Khalifa was definitely out of the question for me!” Maria said that there were many shopping centers
in Dubai, but that they were expensive. The weather was “unbelievably hot.” On her last night in Dubai,
Maria and her friends went on a dinner cruise on a small boat in the harbor. There was music and great
food, and they could see the lights of the city as the boat cruised through the harbor. “It was the perfect
ending to two weeks in Dubai,” she said.

Answer

Reported speech is underlined; quoted is highlighted.

Maria recently returned from a conference in Dubai and told her colleagues about her trip there.
She said that the architecture of Dubai was really fascinating, with many new buildings of glass
and steel. Of course, she also saw the Burj Khalifa, the world’s tallest building. Some of her
friends went up to the top, but Maria said she didn’t because she has a fear of heights. “I don’t
even like being on a second-story balcony, so the Burj Khalifa was definitely out of the question
for me!” Maria said that there were many shopping centers in Dubai, but that they were
expensive. The weather was “unbelievably hot.” On her last night in Dubai, Maria and her friends
went on a dinner cruise on a small boat in the harbor. There was music and great food, and they
could see the lights of the city as the boat cruised through the harbor. “It was the perfect ending

139
totwo weeks in Dubai,” she said.

RULES FOR INDIRECT SPEECH

Rule No. 1. Words of the speaker (reported speech) are not enclosed in Inverted Commas or
Quotation Marks in Indirect Speech.

Rule No. 2. Usage of word “that”: The conjunction “that” is always used between
reporting verb and reported speech in indirect speech.
Example:
 Direct Speech: He said, “I write a letter”.
 Indirect Speech: He said that he wrote a letter.

Rule No. 3. Change in tense of the reported speech: A change is made in tense of reportedspeech
for changing a direct speech into indirect speech. Rules for tense change are given below:

Examples:
 Direct Speech: She said, “I am watching a movie”. Indirect
Speech: She said that she was watching a movie
 Direct Speech: He says, “I am playing cricket”. Indirect
Speech: He says that he was playing cricket.

Rule No. 4. Changes in Pronoun: The pronoun (or subject) of reported speech is sometime
changed according to the pronoun (or subject) or Object of the reported verb (first sentence of
Direct speech). The possessive pronouns (i.e. his, her, my, their, your etc.) may also change
according to subject or object of the first sentence.

Examples:
 Direct Speech: He said, “I eat two apples”. Indirect
Speech: He said that he ate two apples.
 Direct Speech: She said to me, “I like your book”. Indirect
Speech: She said to me that she liked my book.

Rule No. 5. Change in Time: If there is time mentioned in the sentence of Direct speech, the time
will be changed in Indirect Speech. There are certain rules changing the time. i.e. not into then,
tomorrow into the next day, today into that day, yesterday into the previous day.

Examples:
 Direct Speech: She said, “I am buying a laptop today”.
Indirect Speech: She said that she was buying a laptop that day.
 Direct Speech: He said, “I need your now”.
Indirect Speech: He said that he needed my help then.

Direct Speech Indirect Speech


I He/ She
We They
Change of pronouns My His/ Her
Your My
Our Their

140
Me Him/ Her
Us Them
Here there
Today That day
Now Then
This That
These Those
Change of place and time Thus So
words This morning That morning
Yesterday The day before
Tomorrow The next day
Next week The following week
Next month The following month

DIRECT SPEECH CHANGES - INTO INDIRECT SPEECH (TENSE CHANGE)


(i) Present Simple Tense into Past Simple Tense
(ii) Present Progressive Tense into Past Progressive Tense
(iii) Present Perfect Tense into Past Perfect Tense
(iv) Present Perfect Progressive Tense into Past Perfect Tense
(v) Past Simple Tense into Past Perfect Tense
(vi) Past Progressive Tense into Perfect Continuous Tense
(vii) Past Perfect Tense (The tense remains unchanged)
(viii) Past Perfect Progressive Tense (The tense remains unchanged)
(ix) Future Simple Tense (e.g. will) into “would”
(x) Future Progressive Tense (e.g. will be) into “would be”
(xi) Future Perfect Tense (e.g. will have) into “would have”
(xii) Future Perfect Progressive Tense (e.g. will have been) into “would have been”

RULES FOR AFFIRMATIVE/ POSITIVE SENTENCES

1. PRESENT TENSE

PRESENT SIMPLE changes into PAST SIMPLE

She said, “I work in a hospital”. She said that she worked in a hospital.

They said, “We play Football”. They said that they played Football.

He said, “I love my parents”. He said that he loved his parents.

She said, “I don’t waste time”. She said that she didn’t waste time.

PRESENT PROGRESSIVE changes into PAST PROGESSIVE

141
He said, “I am waiting for someone”. He said that he was waiting for someone.

I said, “She is driving a car”. I said that she was driving a car.

She said, “They are enjoying the music”. She said that they were enjoying the music.

David said, “I am not going to College”. David said that he was not
going to College.

PRESENT PERFECT changes into PAST PERFECT


She said, “ I have completed the She said that she had completed the
work”. work.
John said, “I have won a prize”. John said that he had won a prize.
He said, “She has washed the He said that she had washed the
shirts” shirts.
David said, “I have not met her”. David said that he had not met her.

PRESENT PERFECT PROGRESSIVE changes into PAST PERFECT PROGRESSIVE

She said, “I have been working in a factory She said that she had been working in a factory
for two years. for two years.

2. PAST TENSE

PAST SIMPLE changes into PAST PERFECT

He said, “I started a Job”. He said that he had started a job.

She said, “I bought a new car”. She said that she had bought a new car.

They said, “We went to a market” They that they had gone to a market.

He said to me, “You didn’t help me” He said to me that I had not helped him.

PAST PROGRESSIVE changes into PAST PERFECT PROGRESSIVE

142
He said, “I was writing a poem” He said that he had been writing a poem.

The kids said, “We were playing a game”. The kids said they had been playing a game.

They said, “We were watching a movie”. They said they had been watching a movie.

She said, “I was not making a noise”. She said that she had not been making a noise.

PAST PERFECT changes into PAST PERFECT (No change in tense)

They said, “We had won the game” They said that they had won the game.

He said, “I had gone to home”. He said that he had gone to home.

David said, “I had passed the exam”. David said that he had passed the exam.

She said, “I had not received the letter”. She said that she had not received the letter.

3. FUTURE TENSE

FUTURE SIMPLE TENSE


WILL changes into WOULD

She said, “I will go to London”. She said that she would go to London.

He said, “I will start a new job”. He said that he would start a new job.

They said to me, “We will help you”. They said to me that they would help me.

She said, “I will not waste time”. She said that she would not waste time.

FUTURE CONTINUOUS TENSE

WILL BE changes into WOULD BE

143
He said to me, “I will be waiting for you”. He said to me that he would be waiting for me.

David said, “I will be making tea”. David said that he would be making tea.

She said, “I will be feeding my kids”. She said that she would be feeding her kids.

They said, “We will not be driving a car”. They said that they would not be driving a car.

FUTURE PERFECT TENSE

WILL HAVE changes into WOULD HAVE

She said, “I will have cooked the food”. She said that she would have cooked the food.

He said, “I will have cleaned the room”. He said that he would have cleaned the room.

David said, “I will have called a Doctor”. David said that he would have called a Doctor.

RULES FOR INTERROGATIVE/ QUESTION SENTENCES

The basic rules for converting Direct Speech (question sentences) into Indirect Speech are as follows:

(i) Comma and inverted commas will be removed.

(ii) The conjunction “that” will not be used in Indirect Speech (in question sentence). The conjunction “that” is
used in Indirect speech of all positive sentences.

(iii) In indirect speech, the question sentence will be expressed in assertive (positive) form instead of
interrogative form.

(iv) Question mark (?) will not be used in indirect speech of interrogative sentence. .

(v) The verb such as ‘say’ or ‘tell’ (e.g. he said or she said) in direct speech is replaced with verb such ‘ask’ or
‘inquire’ (e.g. he asked, she inquired) in indirect speech.

The above rules are common for all question-sentences. Apart from these common rules, there are a few
specific rules which apply according to the type of question in sentence.

There are two types of question-sentences:

1. Question which can be answered with YES or NO

144
2. Question which cannot be answered only with YES or NO, but requires a detailed answer. Such questions
usually start with “what, why, how, when etc.”

Examples:

• Do you like a mango? (Such a question can be answered by saying only YES or No)

• What are you doing? (Such a question cannot be answered by Yes or No but it requires a bit explanation
i.e. I am listening to music.)

The specific rules according to type of question are as follows:

Questions that CAN be answered with “YES or NO”

To make indirect speech of such questions, the word “if” or “whether” will be used in Indirect Speech. Both
the words “if” or “whether” can be used interchangeably.

Examples:

Direct Speech Indirect Speech

David said to me, “Do you like coffee?” David asked me if I liked coffee.

He said to me, “Will you help me?” He asked me if I would help him.

She said to me, “Can I use your Phone?” She asked me if she could use my Phone.

They told me, “Have you gone to London?” They asked me if I had gone to London.

He said to me, “Did you meet your friend?” He asked me if I had met my friend.

John said to her, “Will you buy the book?” John asked her whether she would buy the book.

She said, “Are they waiting for me?” She asked whether they were waiting for her.

Questions that CANNOT be answered with only “YES or NO”

To make indirect speech of such questions, the word “if” or “whether” will not be used. The question is
changed into assertive(positive) form and is simply placed after the reporting verb.

Examples:
Direct Speech Indirect Speech

145
They said to me, “where have you gone?” They asked me where I had gone.

Teacher told her, “Why did you fail the exam? Teacher asked her why she had failed the exam.

I said to her, “What were you eating?” I asked her what she had been eating.

He said to her, “How will you help me?” He asked her how he would help her.

The student said, “How can I buy this book?” The student asked how he could buy this book.

They said, “How is the weather?” They asked how the weather was.

She said, “Who was that fantastic man?” She asked me who that fantastic man had been.

INDIRECT SPEECH FOR MODALS

For converting direct speech into indirect speech, Present Modals (e.g. Must, Can, May) are changed into
Past Modals (i.e. Could, Might, Had to).

Direct Speech Indirect Speech


CAN changes into COULD
He said, “I can sing a song”. He said that he could sing a song.
She said, “I can write a poem”. She said that she could write a poem.
David said, “I can play a Guitar”. David said that he could play a Guitar.
MAY changes into MIGHT
She said, “I may go to Paris”. She said that she might go to Paris.
They said, “We may buy a car”. They said that might buy a car.
He said, “I may call a Doctor”. He said he might call a Doctor.
MUST (not changed)
He said, “I must study this book”. He said that he must study that book.

She said, “I must eat a healthy diet”. She said that she must eat a healthy diet.

Modals like Should, Might, Could, Would, Ought to are not changed in Indirect Speech.

Direct Speech Indirect Speech


He said, “I should start a Job”. He said that he should start a Job.
She said, “I might eat a cake”. She said that she might eat a cake
David said, “I could buy a Camera”. David said that he could buy a Camera.

146
Tutorial
EXERCISE 1

Q1. "What did you see at the South Pole?" Ashok asked Anil.

A. Ashok asked Anil if he saw anything at the South Pole.

B. Ashok asked Anil what he had seen at the South Pole.

C. Ashok asked Anil what did he see at the South Pole.

D. Ashok asked Anil that he saw anything at the South Pole.

Q2. Suresh asked, “How long will it take to travel from Germany to South Africa ?"

A. Suresh asked how long it will take to travel from Germany to South Africa.

B. Suresh asked how long it would take to travel from Germany to South Africa.

C. Suresh asked how long it would take to travel from Germany to South Africa.

D. Suresh was asking how long it must take to travel from Germany to South Africa.

Q3. He said to me, "I expect you to attend the function."

A. He told me that he had expected me to attend the function.

B. He told me that he expected me to attended the function.

C. He told me that he expected me to have attended the function.

D. He told me that he expected me to attend the function.

Q4. Manna asked Rohan, "Have you sat in a trolley bus before ?"

A. Manna asked Rohan had he sat in a trolley bus earlier.

B. Manna asked Rohan had he sat in a trolley bus before.

C. Manna asked Rohan if he sat on a trolley bus before.

D. Manna asked Rohan if he has ever sat in a trolley bus.

Q5. Vissu said, "We passed by a beautiful lake when we went on a trip to Goa."

A. Vissu said that they passed by a beautiful lake when they had gone on a trip to Goa.

B. Vissu said that they has passed by a beautiful lake when they went on a trip to Goa.

C. Vissu said that they had passed by a beautiful lake when they had gone on a trip to Goa.

147
D. Vissu said they passed by a beautiful lake when they went on a trip to Goa.

Q6. The teacher said to Ram, “Congratulations! Wish you success in life."

A. The teacher congratulated Ram and said wish you success in life.

B. The teacher wished congratulations and success in life to Ram.

C. The teacher wished congratulations to Ram and wished him success in life.

D. The teacher congratulated Ram and wished him success in life.

Q7. The poor examine said, "O God, take pity on me."

A. The poor examine prayed God to take pity on him.

B. The poor examine, involving God, implored him to take pity on him.

C. The poor examine exclaimed that God take pity on him.

D. The poor examine asked God to take pity on him.

Q8. The father warned his son that he should be beware of him.

A. The father warned his son, "beware of him !"

B. The father warned his son, "Watch that chap !"

C. The father warned his son, "Be careful about him."

D. The father warned his son, "Don't fall into the trap."

Q9. He said,"I cannot help you at present because I am myself in difficulty."

A. He said that I cannot help you at present because I myself in difficulty.

B. He said that he could not help you at present because he was himself in difficulty.

C. He told that he could not help you at present because he was himself in difficulty.

D. He asked that he could not help you at present because he was himself in difficulty.

Q10. He says, ''I don't want to play anymore."

A. He says that he doesn't want to play anymore.

B. He says that I don't want to play anymore.

C. He says that I didn't want to play anymore.

D. He says that he didn't want to play anymore.

Q11. The government has announced, "Taxes will be raised".

A. The government has announced that taxes would be raised.


148
B. The government has announced that taxes would raised.

C. The government has announced that taxes will be raised.

D. The government has announced taxes will be raised.

Q12. You said, "It gives me great pleasure to be with my friend, we have great time ever."

A. You said that it gave him great pleasure to be with his friend we had great time ever.

B. You said that it gave you great pleasure to be with your friend you had great time ever.

C. You said it gives me great pleasure to be with my friend we have great time ever.

D. You said that it has given you great pleasure to be with your friend you have great time ever.

Q13. She said, "If you work hard, you will take good marks."

A. My teacher advised if I work hard I could take good marks.

B. My teacher advises if you work hard you will take good marks.

C. My teacher advises if I work hard I will take good marks.

D. My teacher advised that I worked hard I would take good marks.

Q14. Mother said to child, "You must do your homework, after that you can go for playing."

A. Mother asked to child that he must did his homework after that he could go for playing.

B. Mother said to child that he must done his homework after that he could be go for playing.

C. Mother was requested to the child that he does his homework after that he can go for playing.

D. Mother is ordered to the child that he did his homework after that he can gone for playing.

Q15. They said, "We really enjoyed a movie, we have never seen such a movie in a life."

A. They said that they really had enjoyed a movie they had never seen such a movie in a life.

B. They said that they really enjoyed a movie, they never saw such a movie in a life.

C. They said that they really had enjoyed a movie they have never seen such a movie in a life.

D. They said that we really had enjoyed a movie we had never seen such a movie in a life.

Q16. "Did you know about Mr. Ahmed", I said. "No, I did not know about him", She said.

A. I asked if she had known about Mr.Ahmed she replied no she had not known about him.

B. I asked she knew about Mr.Ahmed she replied no she not know about him.

C. I asked she had known about Mr. Ahmed she replied no she had not know about Mr. Ahmed.

D. I was asked that did she know about Mr. Ahmed she said that no she did not know about him.
149
Q17. A leader says, "A country cannot be peaceful until good education is provided to each child."

A. A leader says that a country could not be peaceful until good education is provided to each child.

B. A leader says that a country cannot been peaceful until good education was provided to each child.

C. A leader says that a country cannot peaceful until good education will be provided to each child.

D. A leader says that a country cannot be peaceful until good education is provided to each child.

Q18. He bade his love goodbye.

A. He said, “Goodbye”.

B. He wished his love, “Goodbye”.

C. He exclaimed, “Goodbye, love”.

D. He said. “Goodbye, my love”.

Q19. His father said that it was time he visited his uncle.

A. “It was time you visited your uncle” said his father.

B. “It is time you visited my uncle” said his father.

C. “It is time you visit your uncle” said his father.

D. “It is time you visited your uncle” said his father.

Q20. He exclaimed with joy that their team had won the tournament.

A. He said, “Our team has won the tournament”

B. He said, “Wow I Our team won the tournament”

C. He exclaimed, “Hurrah! Our team has won the tournament!”

D. He said, “Our team won the tournament.”

EXERCISE 2

1. Read the dialogue and complete the passage given below.

Interviewer: So, why do you want to be a computer programmer?

Ravi: Well, I have always been interested in computers.

Interviewer: I see. Do you have any experience?

Ravi: No, but I’m a fast learner.

Interviewer: What kind of a computer do you use?

Ravi: Computer? Uhm, let me see. I can use a Mac. I also used Windows 10 once.
150
Interviewer: That’s good.

Q. Ravi recently attended an interview for the selection of a computer programmer. At the interview, he was
asked (a) ……………………….. To this question he replied that he wanted to change his job because (b)
……………………….. When the interviewer asked him (e) ………………………. he replied that he (d) ………………………..
Finally, the interviewer wanted to know (e) ………………………. . Ravi replied that he could use a Mac and had
also used Windows 10 once in the,.past. The interviewer seemed to be pleased with his answers.

2. Read the dialogue and complete the passage given below.

Sunita: Tomorrow is your birthday, what do you want as a gift?

Neetu: That is a lovely thought but I don’t want anything.

q. Sunita asked Neetu since the next day was her birthday, (a) …………………… Neetu replied that (b)
…………………… but (C) ………………….. .

3. Read the dialogue and complete the passage given below.

Gardener: Did you water the plant today?

Dev: No, but I will, today.

Gardener: Then tomorrow I will get a sapling of sunflower.

Q. The Gardener asked Dev (a) …………………… Dev replied negatively but (b) …………………… Then the gardener
said that (c) ………………….. ..

4. Read the dialogue and complete the passage given below.

Electrician: When did your electricity go?

Mohan: It is not working since evening.

Electrician: Sorry sir, in this case, I will have to check the fuse now.

Q. The electrician asked Mohan (a) …………………… Mohan replied that (b) …………………… The electrician
apologetically said that in that case (c) ……………………

5. Read the dialogue and complete the passage given below.

Doctor : You should take this medicine every day.

Patient : Should I take it before dinner or after dinner?

Doctor : No, you should take it after breakfast.

Q. The Doctor advised the patient that (a) …………………… The patient further asked (b) …………………… The
doctor replied negatively and then said (c) ………………….. .
151
6. Read the dialogue and complete the passage given below.

The Scenario

There is a reception for Professor Hazin, a respected author, who has just announced he will publish a new
book. A student asks him a few questions for the student newspaper.

Reporter: “What is the title of your book?”

Mr. Hazin: “The title is The River at Midnight.”

Reporter: “What is the topic?”

Mr.Hazin: “The book is a collection of new poetry that I wrote last year.”

Reporter: “Is this your first book?”

Mr. Hazin: “It is my first book of poems. I have previously published two novels.”

Reporter: “Do you have any advice for aspiring writers?”

Mr. Hazin: “They should read a lot and write a lot – every day if possible.”

Reporter: “Do you have anything else you’d like to say?”

Mr. Hazin: “Yes. Please join me for some refreshments at the reception!”

EXERCISE 3

Q1. I reiterated, “I don’t care about the job.”

A. I reiterated that I did not care about the job.

B. I repeatedly said that I cared about the job.

C. I said again and again I didn't care about the job.

D. I reiterated I didn’t care about the job.

Q2. The new student asked the old one, “Do you know my name?"

A. The new student asked the old one did he know his name

B. The new student asked the old one If he knows his name

C. The new student asked the old one that whether he knew his name.

D. The new student asked the old one if he knew his name.

Q3. Tania said to her friend, ‘Can you lend me an umbrella ?’

A. Will you lend me your umbrella, Tania asked her friend.

152
B. Tania requested her friend to lend her an umbrella

C. Tania asked her friend to give her an umbrella.

D. Please give me an umbrella Tania requested her friend.

Q4. The doctor says, "It is better you undergo a surgery next week."

A. The doctor says that it was better I Underwent a surgery the coming week.

B. The doctor advises that it is better I underwent a surgery the following week.

C. The doctor says that it is better I undergo a surgery the following week.

D. The doctor advised me to undergo a surgery the following week.

Q5. Where will you be tomorrow," I said, “in case I have to ring you"?

A. I said to him where he will be in case I have to ring him.

B. I enquired about his where about puts the next day in case I would have to ring up.

C. I asked where he would be the next day in case I had to ring him.

D. I asked where you will be the next day in case I will ring him.

Q6. The Prime Minister said that no one would be allowed to disturb the peace.

A. The Prime Minister said “No one will disturb the peace.”

B. The Prime Minister said “No one can disturb the peace.”

C. The Prime Minister said “We would not allow no one to disturb the peace

D. The Prime Minister said “We shall not allow any one to disturb the peace."

Q7. “Would you mind taking off your shoes before entering the house?” He said to the foreigner.

A. He said the foreigner that to take off his shoes before entered the house.

B. Before entering the house he said that shoes must be taken off.

C. He told the foreigner that he must take off his shoes before entered the house.

D. He requested the foreigner to take off his shoes before entering the house.

Q8. The receptionist said to Umesh, “Please be seated and wait for your turn.”

A. The receptionist told Umesh to please be seated and wait for his turn.

B. The receptionist requested Umesh that he should be seated and wait for his turn.

C. The receptionist requested Umesh to be seated and to wait for his turn.

D. The receptionist requested Umesh that be seated and wait for his turn.
153
Q9. “Alas ! How I have wasted my fortune!” said he.

A. He confessed with regret that he had been very extravagant.

B. He confessed with regret that I have wasted my fortune.

C. He confessed with regret that he had wasted his fortune.

D. He confessed with regret that he had been wasting his fortune.

Q10. he project manager asked Vivek, “How much time will you take to finish this project ?”

A. The project manager enquired of Vivek how much time he would take to finish that project.

B. The project manager enquired of Vivek how much time he will be taking to finish that project.

C. The project manager asked Vivek how much time he would take to finish this project.

D. The project manager asked Vivek how much time would he take to finish that project.

Q11. I said to him, “Where have you lost the pen I brought for you yesterday?"

A. I asked him where he had lost the pen I had brought for him the previous day.

B. I asked him where had he lost the pen I had brought him the previous day.

C. I asked him where he had lost the pen I have brought for him the previous day

D. I asked him where he had lost the pen I had brought him yesterday.

Q12. I said to my friend, "Good Morning. Let us go for a picnic today."

A. I wished my friend good morning and proposed that they should go for a picnic that day.

B. I told good morning to my friend and suggested to go for a picnic today.

C. I wished my friend good morning and proposed that we should go for a picnic that day.

D. I told good morning to my friend and asked to go for a picnic that day.

Q13. Can you see a woman seated On the bench in a park ? Alice asked her.

A. Alice asked her If she could see a woman seated on the bench in a park.

B. Alice asked her if she could have seen a woman seated on the bench in a park.

C. Alice told her if she could see a woman seated on the bench in a park.

D. Alice asked her if she can see a woman seated on the bench in a park.

Q14. 'From one of the windows of my flat I can see the Qutub Minar’, said my friend.

A. My friend said that from one of the windows of his flat he could see the Qutub Minar.

B. My friend told that from one of the windows of his flat he can see the Qutub Minar.
154
C. My friend said that from one of the windows of his flat he can see the Qutub Minar.

D. My friend told that from one of the windows of his flat he could see the Qutub Minar.

Q15. The lady said to the servant, "If you don’t wash the clothes properly, I will dismiss you."

A. The lady cautioned the servant that she must wash the clothes properly .

B. The lady advised the servant to wash the clothes properly

C. The lady told the servant that she would dismiss her on the event of bad work.

D. The lady warned the servant that she would dismiss her if she didn’t wash the clothes property.

COVER LETTER
The Purpose of a Cover Letter
The cover letter, or letter of application, gives you the opportunity to introduce yourself to an employer
and make connections between the position you are applying for, the skills or experiences you offer, and
the organization. While a resume is a summary of your experiences, achievements, and skills, a cover
letter highlights key elements that you want a potential employer to note. Be sure to relay relevant
academic and work experience as it connects to the position and organization. A cover letter should
introduce you and answer any questions employers may have.

Key Elements of a Successful Cover Letter


1. Uniqueness

Many hiring managers will trash a cover letter if it’s not customized. Putting in the extra time and effort to
write a unique cover letter can make all the difference in the success of your application. There’s nothing
wrong with following a formula, but use custom language for each job and company. This simple step
shows you’re taking the opportunity seriously.

2. Personalization

Skip “To Whom It May Concern” and personalize your greeting instead by using the name of the hiring
manager. Check the job listing for their name and contact details. If the information isn’t included, search
the company website or call the human resources office.

3. Ease of Use

It’s still accepted practice to include your mailing address in the header of your cover letter, but also
remember to include your email address, cellphone number, and website (if you have one). Put this
information front and center so a hiring manager can quickly find it when they’re ready to contact you.

4. Persuasion

155
The first one or two sentences after your greeting should demonstrate why you’re the best candidate for
the job. Giving a very specific reason will grab the hiring manager’s attention and encourage them to keep
reading.

5. Concision

Hiring managers tend to prefer a cover letter of half a page or less in length. Your aim is to grab their
attention and highlight your experience – not recite everything in your resume. Include a few key points
that will entice the hiring manager to read on (and request an interview). When you send a cover letter
digitally, be sure to add links to your website, portfolio, social media channels, and any other sites that
apply.

6. Relevance

Don’t simply list your skills. Instead, describe how they apply to this particular job. You need to do this
even if your past work experience is not in the same industry! You’re trying to convince the hiring manager
you’re worth interviewing. It’s the “why,” not the “what,” that sells.

7. Optimization

Hiring managers are often pressed for time. They may not read every word of your cover letter, so you’ll
want to include industry-relevant keywords that will help them quickly understand that you’re a match for
position.

8. Eagerness

Show genuine interest in not only the position, but also in the company as a whole. Do your research and
find out about its values, mission, and notable achievements. Again, putting in the extra effort will help
your application stand out and show that you’re not just sending the same cover letter to every job you are
applying for.

9. Straightforwardness

End your cover letter with a strong closing statement. Thank the hiring manager for taking the time to
review your application and include a call to action – most likely a request for an interview.

10. Grammatical Correctness

Check (and double-check) your spelling and grammar. Simple mistakes can land your cover letter in the
trash bin. Don’t rely on autocorrect or spell check, since these programs don’t always pick up all errors.
Read and reread the letter yourself, then ask a friend to do the same.

Useful words and phrases for cover letter


Words

1. Communication Skills
156
Strong communication skills — written, verbal and on the phone — are essential in most jobs. In a cover
letter, emphasize your excellent communication skills by using words such as “negotiated,” “addressed,”
“persuaded” and “encouraged.” To describe your writing abilities, use words such as “authored,”
“corresponded” and “publicized.” Choose words that are logical, concise and reader-friendly.

2. Management Skills

When applying for administrative and management jobs choose words which describe your qualifications
carefully. Instead of saying, “Was in charge of” or “Was responsible for,” use words such as “managed,”
“oversaw,” “supervised” and “administered.” One power word is often more effective than four less-
impactful words.

3. Research Skills

Describe your research skills logically, using powers words to highlight the work you did. Instead of saying,
“Looked at data,” use power words such as “examined,” “surveyed,” “investigated,” “analyzed,” and
“evaluated” to describe your research qualifications. Other effective words include “systematized,”
“inspected,” and “extracted.”

4. Creative and Technical Skills

To best summarize your creative skills, use words such as “introduced,” “designed,” “founded,” and
“integrated.” For example, “I designed and implemented new strategies for data transfer.” For technical
skills, use words such as “assembled,” “programmed,” “calculated,” “engineered,” and “upgraded.”

Phrases

Use these sentence starters as examples when drafting the three parts of your cover letter: the
introduction, the body, and the closing paragraph.

INTRODUCTION

 It is with great interest that I am applying for the position of______.


 (Name of contact) provided me with your name and suggested I contact you in reference to the
_____ position.
 As a recent graduate of --------- with a degree in _______ and an avid (reader, writer, sports
enthusiast, etc.), I believe I am a strong applicant for the position of ________.
 As a (research, congressional, etc.) intern with (name of organization) I gained valuable experience
in _______ that makes me an ideal candidate for the position of _______.
 I am eager to apply for the position of ________ because I have the _______ that it takes to be
successful in the position.
 Thank you for speaking with me on (date) and discussing the ______ position. I am very interested
in ________.
 Your organization impresses me because of (the performance of your product, the integrity of your
support staff, its customer satisfaction, etc.).

BODY

157
 My experience as a ______ will help me to contribute _______.
 Throughout my four years of undergraduate studies I have demonstrated skills and abilities that are
exactly what the position of _______ demands.
 I can contribute to your organization’s effectiveness by __________.
 My previous success in _______ has proven my ability to ________.
 To highlight some of my accomplishments I have _________.
 Working with _______ has strengthened and improved my (analytical, research, interpersonal,
organizational, etc.) and (analytical, research, interpersonal, organizational, etc.) skills.
 I am confident that I can be a valuable asset to your organization because _________.

CLOSING

 I am eager to learn more about _____ and would like to discuss my qualifications and interests with
you.
 I am interested in the position and would appreciate the opportunity to discuss my background and
your requirements in greater detail.
 I feel that my ______ and ______ make me a strong candidate for this position, and therefore look
forward to the possibility of employment with (name of organization).
 I would appreciate the opportunity to further discuss my qualifications with you. I can be reached
at (phone number) or at (email).
 I will contact you within (10 days, 1 week, etc.) to (verify my application materials are in order,
introduce myself, inquire as to the next steps, etc.).

SENTENCES THAT CAN BE USED IN MULTIPLE PARAGRAPHS

 I would like the opportunity to put


my skills, drive and enthusiasm to work as a _____ for ______.
 Being (a team player, results driven,
a quick learner, etc.) who loves challenges, I will _______.
 My career goal is to ______.
Therefore I welcome the opportunity to make a contribution to (organization name here).
 The ______ position described in
_____ sounds exactly like the opportunity I’m looking for.

Format of cover letter


 Contact Section- How you include your contact information will vary based on how you are sending
your cover letter. If you are uploading or mailing a letter, put the information at the top of the
page.

With an email cover letter, your contact information should be below your signature.

 Employer Address-How you address the cover letter will depend on how much information you
have about the employer.

158
 Salutation- A salutation is the greeting you include at the beginning of a cover letter. Here's how to
write a salutation, including what to use if you don't have the name of a contact person to list.

 Body- The body of a cover letter includes the sections where you explain why you are interested in
and qualified for the job for which you are applying. Here's what to include in each section or your
letter.

 Paragraphs and Bulleted Lists- A more traditional cover letter contains written paragraphs
describing your qualifications.

 Closings- When you're writing a cover letter or sending an email message to apply for a job it's
important to close your cover letter in a professional manner.

What to Include in Each Section

Your Contact Information


Name
Address
City, State Zip Code
Phone Number
Email Address

Date

Employer Contact Information (if you have it)


Name
Title
Company
Address
City, State Zip Code

Salutation

Dear Mr./Ms. Last Name,

Body of Cover Letter


The body of your cover letter lets the employer know what position you are applying for, why the
employer should select you for an interview, and how you will follow-up. Organize the body of your
cover letter into the following paragraphs:

 First Paragraph
The first paragraph of your letter should include information on why you are writing. Mention the
position you are applying for and where you found the job listing. Include the name of a mutual
contact, if you have one.

 Middle Paragraph(s)
The next section of your cover letter should describe what you have to offer the employer. Mention
specifically how your qualifications match the job you are applying for. Think of this section of the
cover letter as where you're making a pitch for your fit as an employee and show what makes you a
159
great candidate. Make the connection between your qualifications and the job requirements clear.
Use this section to interpret your resume—don't repeat from it verbatim.

 Final Paragraph
Conclude your cover letter by thanking the employer for considering you for the position. Include
information on how you will follow-up. Optionally, you can briefly restate why you would be a good
fit for the position.

Complimentary Close
respectfully yours,

Signature

Handwritten Signature (for a hard copy letter)

Typed Signature

Cover Letter Closing Examples


 Sincerely

 Sincerely yours

 Regards

 Best

 Best regards

 With best regards

 Kind regards

 Yours truly

 Most sincerely

 Respectfully

 Respectfully yours

Crafting an Effective Cover Letter

Paragraph 1: Introduce Yourself

• Identify the position you are applying for, and how you learned of it.

160
• Explain why you are interested in applying for this role.

• You may want to include your year (e.g. junior), college, and major.

• Be sure to include something specific about the company (it’s important to tailor your letter; don’t use
generic language that could apply to any company in that industry).

• Answer the questions: Why is this position right for me? Why do I want to work for this specific
organization?

Paragraphs 2 & 3: Sell Yourself

• Indicate that you understand the position requirements and illustrate that you posses s the skills
required to add value in the role.

• Highlight character traits and/or specific experiences that prove you are a good fit f or the organization
and the role.

• Do not simply repeat what is on your resume.

• DO elaborate on skills and stories – be specific!!

• Look at the required skills section of the job posting for ideas of what to highlight

• Answer the question: Why am I right for this position?

Paragraph 4: Closing

• Keep this paragraph short and sweet.

• Summarize your qualifications for the position and reiterate why you are a good fit.

• Provide contact information (cellphone and email).

• Say thank you!

• Answer the questions: Does the recruiter know how to contact me, and the type of position I am
seeking?

Sample 1 – Electrical Engineering

November 8, 2020

Ms. Carol McConnell


HR Manager
ABC Company
55 Victory Dr.
Sometown, FL 55555

Dear Ms. McConnell:

161
I read your posting on Monster for an electrical engineer with great interest. This challenging position
focused on heavy industrial projects is a perfect match to my qualifications.

Since earning a bachelor’s degree in electrical engineering (BSEE), I have worked for the last 5 years as an
electrical engineer for XYZ Corporation, a leading U.S. shipbuilder of Navy aircraft carriers and submarines.
In this role, I maintain, troubleshoot, upgrade and repair plant electrical equipment. This experience has
given me a strong foundation within the heavy industrial manufacturing sector in addition to proven
success in:

 Improving preventive-maintenance programs and inventory systems to minimize unscheduled


equipment downtime by up to 34%.

 Extending the average life of electronic industrial systems and components by up to 25%.

 Cutting machinery repair and replacement costs by $250K in a single year.

 Troubleshooting circuitry, wiring and other electrical/electromechanical issues and implementing


design improvements to prevent future problems.

During my tenure at XYZ Corporation, I developed expertise in an array of cutting-edge engines/motors


and plant-wide electrical/control systems, including digital signal processors (DSPs) and programmable
logic controllers (PLCs). I am also proficient in using electrical design tools and software such as ETAP, SPEL,
SKM Systems Analysis and EasyPower.

I am very interested in being part of your integrated engineering team. Please call me at (555) 555-5555 to
set up an interview. Thank you for your time.

Sincerely,

Cindy Jones
Enclosure: Résumé

Sample 2 - Electrical Engineering

*Today’s Date+

*Hiring Manager’s Name+


[Company Name]
[Company Address]
[City, State, Zip Code]
[Telephone Contacts]

*Dear Mr. /Mrs. /Ms. *Manager’s Name+,

Re: Application for the position of an Electrical Engineer

It is with much joy and pleasure that I write to you to consider me for the position of Electrical Engineer
with your organization. I base my confidence on my skill-set, academic accomplishments, and exceptional
career track record.
162
After high school, I did pursue a degree course at the famous YTR Institute of Technology, where I
graduated with flying colors. After my graduation, I moved ahead to obtain government certification in the
said field. I have since worked as an Electrical Engineer for the last seven years for two companies whose
identities I have disclosed in the resume.

Among my core competencies is the ability to work for long hours with minimal supervision,
unprecedented attentiveness to details, and the ability to multitask. So effective, I have been that I have
won numerous accolades from my past employers and clients alike.

When invited for an interview, I hope to discuss my qualifications in further depth and greater clarity.

Sincerely,
[Your Signature]

Sample 3- Computer science cover letter

October 4, 2020

Ms. Phoebe Hopkins


VP of Information Technology
ABC Corporation
245 Maple St.
Sometown, NJ 55555

Dear Ms. Hopkins:

I found your posting on Monster for a network administrator and knew I wanted to apply immediately.
ABC Corporation’s reputation for responsive service and proactive support differentiates your company
from the rest, and I would like to be part of your dynamic team.

Since graduating from XYZ College with a BS in computer science, I have worked for three years as a
network administrator for DEF Company. In this role, I am valued as a solutions-focused manager of
networks, IT systems, user support functions and technology projects. I have experience in all areas
covered in your job ad, including:

 Network and Server Design and Administration

 LAN, WAN, VPN, SAN and VoIP Infrastructure

 Network Analysis and Optimization

 System Security, Disaster Recovery and Business Continuity Plans

 Cloud Computing and Data Storage

 Enterprise System and Software Implementations

163
Known as a keen technical trouble-shooter, I am dedicated to providing cost-effective and expedient
solutions optimizing network stability and security; improving performance of systems and equipment; and
resolving issues such as network crashes, system slowdowns and virus outbreaks.

At DEF Company, I provide world-class service technical support in an enterprise environment, and I am
confident I would quickly become a productive network administrator at ABC Corporation. If you agree,
please call me at (555) 555-5555 to set up an interview.

I look forward to your response.

Sincerely,

Barbara Holt

Enclosure: Résumé

Sample 4- Computer science cover letter

Your name:
Address:
Contact details:
Email address:

Date: MM/ DD/ YYYY

Employer's name:
Designation:
Company's name:
Address:

Dear Mr. / Mrs. (Employer's name),

From you job posting on the website (mention the source of the job vacancy), I understand that you are in
need of an experience computer software engineer with a minimum of a year experience. I meet all your
requirements and have enclosed my resume for your further attention.

I have earned my masters in information technology and bachelors of Science in engineering from XYZ
University. My extensive knowledge of the industry's trends and recent accomplishments can give your
organization some innovative ideas and creations that will suit the company's needs. I believe my
exemplary accomplishments are something I can share with your company and team of engineers.

I am working currently as a computer engineer with (mention your current company) wherein I am in
charge of developing systems for client database transfers. Here I have managed a team of creative
engineers to produce essential code procedures for the company's intense programming projects. In
addition to this, I have assisted many multi-national companies in creating programs to enrich their
learning procedures and electronics developments. With all these inputs, I have improved the standards of
my company's operations in research and development by 10%.

164
I have enclosed my resume and references for your review. I can be contacted at (insert contact details) to
discuss this opportunity in detail. I look forward to your response.

Thank you so much.

Sincerely,
Your name:

Enclosure: resume, work experience certificate, reference letter.

Sample 5- Mechanical Engineering

Today’s Date

*Hiring Manager’s Name+


[Company Address]
[Company City, State, xxxxx]
[(xxx)-xxx-xxxx]
[hiring.manager@gmail.com]

Dear *Mr./Mrs./Ms.+ *Manager’s Name+,

My name is [your name] and I have been working with [previous employer] for the last [number of years].
The description you’ve outlined for the mechanical engineer position at [company name] strongly matches
my experience and qualifications, especially prototype design of industrial machine components. As such, I
am highly interested in joining your team.

I have a B.S. in Mechanical Engineering from U.C. Berkeley and have worked as a mechanical engineer for
[number of years]. In my time at [previous employer] I was able to:

 Develop three prototype components of the SuperDrill 9000 that was commercially employed;

 Act as lead machinist and component designer on four separate engineering products for [previous
employer];

 Win “Best Machinist – Western Region” - nominated twice

I am aware that [company name] has been engaged in some exciting projects and initiatives in the industry
in recent years, especially with [project name or activity]. As you can see, I have been involved in similar
work and know I could greatly contribute to *company name+’s goals in the future.

Thank you for taking the time to read my application and for your consideration. I would love to come in
for an interview and am looking forward to hearing from you. I understand you’ve probably received a
number of applications, so I will check in next week if I haven’t heard anything by then. Thank you again for
your time and consideration.

Sincerely,

[Your Name]

Sample 6- Mechanical Engineering


165
[Date]

[Name of HR Manager]
[Position Title]
[Company Name]
[Company Address]
[City, State, Zip]

Dear Mr./Ms. [...]:

Please accept this letter and the accompanying resume as an expression of my interest in a position with
your organization.

As my resume indicates, in June 2006, I expect to receive a Bachelor of Science degree in mechanical
engineering from California Polytechnic State University. I would then like to begin employment with an
organization in which my effective performance will be met with recognition and growth.

I have acquired a sound overall knowledge of leading edge engineering principles, tools, and practices, with
emphasis on designing, building and testing of mechanical systems. I am proficient in the use of various
automated solutions including current releases of AutoCAD and SolidWorks.

I have applied classroom learning to innovative and successful projects in which I served as sole or principal
designer. Additionally, I have been a responsible leader in a family owned small business.

My personal attributes include leadership and sound judgment as well as creativity, analytical and
troubleshooting skills. I interact productively with people from diverse backgrounds. I have a career history
of achieving employment goals. I have a history of quality work carried to timely completion.

I am certain I could make significant contributions to your organization, and I would welcome the
opportunity to meet with you to discuss how my education and abilities might best be employed by your
organization.

Thank you for your consideration. I look forward to your response.

Sincerely,

Mark Gunlogson

Encl: Resume

Sample 7- Civil Engineering

November 8, 2020

Ms. Diane Smith


HR Manager
ABC Company
55 Circle Point
Sometown, AZ 55555

166
Re: Civil Engineer Trainee III u Advertised on Monster

Dear MS Smith:

Your civil engineer trainee position is an exciting opportunity. ABC Company’s stellar reputation for
providing quality service and project support to your clients is well-known, and I am motivated to join your
team.

Highlights of my credentials:

 BS in civil engineering and Engineer in Training (EIT) certification from NCEES.

 Two years of experience as a civil engineer trainee and intern for premier design firms.

 Hands-on experience providing project design and site civil engineering leadership for water,
wastewater, roadway and other municipal infrastructure projects in Sometown, AZ.

 Delivery of on-schedule, on-budget and high-quality completion of infrastructure projects with


varying degrees of complexity and budgets ranging from $500K to $6.8M.

 Proficient in preparing drawings and technical specs using AutoCAD Civil 3D and MicroStation.

In addition, I have worked closely with licensed PEs to save our clients and employers tens of thousands of
dollars while maintaining compliance with regulatory requirements. I have provided creative, cost-effective
design solutions to issues including erosion, corrosive groundwater, insufficient water inflow/drainage, soil
instability, steep-slope grading and other engineering challenges.

If my credentials and strong understanding of municipal water and wastewater infrastructure systems are
well matched to your needs, please contact me at (555) 555-5555 or email lm@somedomain.com to
schedule an interview. Thanks very much.

Sincerely,

Lena Marks, EIT


Enclosure: Résumé

Sample 8 – Civil Engineering

DATE

Ruth Griffin
Construction Works
869 Badger Pond Lane
Tampa, FL 33614

Subject: Application for position of Civil Engineering

Dear Mr Griffin,

167
My name is Ruth Griffin, and I write to introduce my career qualifications in response to your
advertisement for a qualified Civil Engineering professional for your company. I am very eager to learn
more about what you are seeking, and offer my credentials as listed in this cover letter and resume.

You will find in the enclosed resume that I recently graduated with a Civil Engineering degree from
Cleveland State University, with the hopes of becoming a Civil Engineer at a top ranked company in the
area. After careful research, I have found that your company, a leader in Civil Engineering technology, is
the perfect match for these goals; and you are hiring.

During college, I was able to be a part of a number of structured and team oriented Civil Engineering
experiments and projects, and in that time, gained considerable knowledge of how theory and science can
be applied to create process improving products for our world. I am looking to your company to help me
apply this education into hands on success, and I believe that working in a plant such as yours will be a very
fulfilling and enriching job.

Please contact me after you have read this cover letter and resume.

Yours Sincerely,

(Signature hand written in blue pen ink)

Karen Lefebvre

Enclosed: Resume

Tutorial
Exercise 1

Q1. Which of the following is not a function of a cover letter?

A. To inform the employer of the job you are applying for

B. To show how well you write

C. To inform the reader of what you expect to get out of the job you're applying for

D. To entice the reader to want to get to know you better by interviewing you

Q2. When should you send a cover letter?

A. Only when an ad specifically requests it

B. Every time you send out your resume

C. When you need to list your salary requirement

D. When you need to list references

E. When you need to show off your marketability

168
Q3. Which of the following is NOT something that a cover letter writer should always do in his or her
opening paragraph?

A. Inform on why he or she is writing

B. Impress he employer with knowledge of the company

C. Be specific about the position sought and what they can can offer

D. Mention salary specifications

Q4. The effectiveness of taking risks with the opening paragraph of your letter depends greatly on the
field in which you are seeking a job.

A. True
B. False

Q5. Which of the following is NOT something that a cover letter should always contain in its closing
paragraph?

A. Request for an interview

B. Statement that you look forward to hearing from the recipient

C. Statement thanking the employer for considering the cover letter (and resume)

Q6. Which of the following is necessary for a successful cover letter?

A. Opening, body and closing paragraphs

B. Addressing the letter to a specific individual or department

C. Connecting your skills to the ones profiled in the job advertisement

D. All of the above

Q7. What's the best way to make value judgments or claims of personal attributes more credible in a
cover letter?

A. Use very positive language in making the claim

B. Substantiate the claims by backing them up with examples

C. There is no good way to make such claims, so omit them

Q8. Employers like candidates to express a willingness to perform any available job.

A. True
B. False

Q9. If you are not fully qualified, it's always best to discuss the reasons why you aren't fully qualified in
the letter.

169
A. True
B. False

Q10. It's okay to mention skills gained in school even if they have nothing to do with the job sought.

A. True
B. False

Q11. How can you make the most of your college experience in your cover letter?

A. Describe skills gained in the classroom

B. Describe sports and extracurricular activities

C. Discuss hands-on projects

D. All of the above

Q12. Transferable skills should be portrayed in the resume only, NOT in the cover letter.

A. True
B. False

Exercise 2

Cover letter - Electronics Engineering.

1. Write a cover letter for the post of embedded software engineer in the FICA Pvt ltd, Gurgaon. Write
about the experience, achievements you have attained and also about the projects done till yet.

2. Write a cover letter to the Omega Pvt Ltd ,to apply for the internship in the Artificial intelligence and
neurological networks. Also, mention your achievements and subjects of interest.

3. Write a cover letter to Polycab Limited, for the post of electronic engineer. Write this letter as a fresher
and do mention the projects you did in your bachelor degree.

Questions for computer science

1. Write a cover letter for applying to a job in Wipro, considering yourself as a fresher.

2. Write a cover letter for the post of Software Engineer, giving the details of the past experience and
achievements.

3. Write a cover letter for the post of Hardware Engineer, giving the details of the past experience and
achievements.

4. Write a cover letter to Blue River Tech,to apply for the internship in the Artificial intelligence and
neurological networks. Also, mention your achievements and subjects of interest.

170
QUESTIONS FOR MECHANICAL ENGINEERING

1. Write a cover letter for the post of Design Engineer in ABC Pvt Ltd, Pune. Write the letter to Mr. Ajay
Sharma who is the Manager of the company . Also, mention about the software you have worked on for
designing.

2. Write a cover letter to BRN products ,to apply for the internship in the field of Industrial engineering and
testing. Also, mention about the project which you were handling for testing the machines and various
projects.

3. Write a cover letter to Ms. Susan Watts from XYZ Company at Heston, regarding the vacancy for
Automobile engineer. Also, mention about your research work , achievements and projects done till yet.

COVER LETTER FOR CIVIL ENGINEERING

1. Write a cover letter to Ms. Cynthia Black from ABC Company at Ealing, regarding the vacancy for civil
engineer. Also, mention about your research work , achievements and projects done till yet.

2. Write a cover letter for the post of Design Engineer in XYZ Pvt Ltd, Bangalore. Write the letter to
Mr.Pawan Paul who is the Manager of the company . Also, mention about the softwares and tools you
have knowledge and use for designing.

3. Write a cover letter for applying to a job in Sigmatax Company, considering yourself as a fresher.

Exercise 3

Worksheet 1- Use this worksheet to gather the necessary information to create a cover letter. Once you have
answered the questions, put the information in letter format.(think about your dream job and company )

Preparation

1. Based on the job announcement key words and what you know about the job, what is the company
looking for in a candidate? (What are they buying? What problem are they trying to solve? What
experience, results, skills, approach, and abilities do they need?)

2. To whom are you sending the letter?

Name:

Title:

Company, Address:

Opening Paragraph

171
1. What is the exact title of the job? …………………………………………………

2. How did you discover the job was available? (e.g., job board, internet, placement center, professional
meeting, referral, person in hiring firm, career fair, recruiter, etc.)…………………….

3. Why are you interested in this particular job and company? (e.g., type of work, location, reputation,
opportunities, etc.)……………………………………..

Middle Paragraph

1. What do you have that matches those needs? (Experience, education, results, skills, approach, abilities)

2. Determine the three to five areas that are the highest priority to the company.

1.

2.

3.

4.

5.

3. Create a few short statements that include key words from job description. Use power verbs. Show
impact and scope.

Closing Paragraph

1. What do you want to happen after you send the resume? (An interview, a phone call, an email)
……………………..

2. What is the person expecting you to send? (e.g., resume, letter of interest, salary requirements,
references, examples of your work)………………………………………………

3. What next steps will you take after sending your resume? (e.g., a call, email, a visit)……

Contact Information

172
1. What is your phone and email?.................................

Worksheet 2 –

Worksheet 3

Re-write the following letter of application in the correct order

A. I am presently employed by a small


computer company, but I feel that I am not using my knowledge of software engineering to
the full. I am looking for a more challenging position where my field of specialisation could
be exploited in a more stimulating environment. As you will notice on my enclosed CV, the
job you are offering suits both my personal and professional interests.
B. I would be pleased to discuss my
curriculum vitae with you in more detail. Please do not hesitate to contact me if you require
further information. I look forward to hearing from you.
C. Dear Ms Johansson

173
D. 12 January 2021
E. I am writing to apply for the
position of Director of Software Development which was advertised on your company
website.
F. Carry Johansson
12 Hamilton Street
Brighton
FH2 6KX
G. Peter Sellers
H. Yours sincerely

174
Unit-6

Critical Reasoning
Critical thinking is the intellectually disciplined process of actively and skilfully conceptualizing, applying,
analyzing, synthesizing, and/or evaluating information gathered from, or generated by, observation,
experience, reflection, reasoning, or communication, as a guide to belief and action.

Critical reasoning argument

The premise (or premises) and conclusion represent the core of the argument. Remember that not all
arguments will have a conclusion, but all will have at least one premise, so we will always have at least a
partial core.

Premise

- The premise – the evidence


- Part of the core of the argument; present in every argument
- Supports the authors conclusion
- Can be a fact or an opinion; can be a description, historical information, statistical or numerical
data, or a comparison of things
- Often signalled by words or phrases such as because of since, due to, and as a result of

EXAMPLE:

Cats with long hair shed all over the house, so you should not get a long-haired cat. I have heard that they
also have lots of fleas.

P¹ Long-haired cats shed all over the house

P² Long-haired cats have a lot of fleas

C You should not get a long haired cat


Conclusion

- Part of the core of an argument; present in most arguments


175
- Represents the authors main opinion or claim; can be in the form of a prediction, a judgment of
quality or merit, or a statement of causality
- Is supported by at least one premise
- Often signalled by words such as therefore, thus, so, and consequently (though note that harder
arguments might use such a word elsewhere in the argument in an attempt to confuse us)

EXAMPLE:

The dreamer rejected the ordinary. Jay invited the ordinary. Jay is no dreamer.

P1- The dreamer rejected the ordinary

P2- Jay invited the ordinary

C- Jay is no dreamer. It is clear from the given facts that this is the conclusion.

Practice: Identify premise and conclusion in the following arguments and label them as (p) and (c)
respectively.

1, People who are red/green colour blind cannot distinguish between green and brown. Gerald cannot
distinguish between green and brown. Therefore Gerald is red/green colour blind.

2, If Max were guilty, he would not ask the police to investigate. Therefore, his asking the police to
investigate shows that he is not guilty.

3, Train service suffers when a railroad combines. By dividing its attention between its freight and
commuter customers, a railroad serves neither particularly well. Therefore, if railroad is going to be a
successful business, then it must concentrate exclusively on one of these two markets.

4. Cars drive faster on long city blocks than on short city blocks. Long blocks are thus more dangerous for
pedestrians than short blocks.

5, The store's competitors claim that the store, in selling off the shirts at those prices, neither made any
profit nor broke even. Consequently, the store's customers must have been able to buy shirts there at less
than the store's cost.

ANSWER

1, P- People who are red/green color blind cannot distinguish between green and brown.

P-Gerald cannot distinguish between green and brown.

C- Therefore Gerald is red/green color blind.

2, P- If Max were guilty

P- he would not ask the police to investigate.

C- Therefore, his asking the police to investigate shows that he is not guilty.
176
3, P- Train service suffers when a railroad combines.

P- By dividing its attention between its freight and commuter customers, a railroad serves neither
particularly well.

C- Therefore, if railroad is going to be a successful business, then it must concentrate exclusively on one of
these two markets.

4. P- Cars drive faster on long city blocks than on short city blocks.

C- Long blocks are thus more dangerous for pedestrians than short blocks.

5, P- The store's competitors claim that the store, in selling off the shirts at those prices, neither made any
profit nor broke even.

C- Consequently, the store's customers must have been able to buy shirts there at less than the store's
cost.

Premise and Conclusion Indicator Words

Words that introduce or appear in an argument premise include:

since (no temporal meaning) may be inferred from


as indicated by given that
because seeing that
for for the reason that
in that inasmuch as
as (non comparison meaning) owing to

Words that introduce or appear in an argument conclusion include:

therefore we may infer


wherefore it must be that
accordingly whence
we may conclude so
entails that it follows that
hence implies that
thus as a result
consequently

Assumptions

An assumption is something that the author must believe to be true in order to draw a certain conclusion;
however, the author does not state the assumption in the argument. The assumption itself might not
necessarily be true in the real world; the only requirement is that the author has to believe it’s true in
order to make his or her claim.

177
For example, what is the author of the below argument assuming must be true?

Amy got an A on the test. Therefore, Amy must have studied for a long time.

The author assumes that, in order to get an A on the test, it is absolutely required to study for a long time.
Note that the author is not just assuming that studying for a long time is one way to get an A on the test.
The author concludes that Amy must have studied for a long time, so that is the only way.

Assumptions fill a gap in the argument; the gap is represented by the arrow in the diagram above. If we
insert a correct assumption into the argument, it makes the argument stronger:

Amy got an A on the test. Studying for a long time is the only way to get an A. Therefore, Amy must have
studied for a long time.
Therefore Amy must have studied for a
Amy got an A on the test. long time.
Studying for a long time is
(premise) the only way to get an A (conclusion)

(assumption)

Types of Critical Reasoning Questions

Critical Reasoning Questions can be categorized into five major types.

1. Strengthen the Argument

2. Weaken the Argument

3. Find the Assumption

4. Find the Conclusion / Draw Inference

5. Paradox Questions

Strengthen Argument

In strengthen-the-argument questions, the type of premises given should be examined. The premises could
be presented in the form of

 Statistical Data – numbers, percentages, ratios, etc

 Causal Data – the data may be in the form of a cause – effect relationship

 Analogy – arriving at a conclusion through likening it to a similar situation

Here the conclusion is already given in the argument. One of the choices if true further supports or
reinforces the conclusion. This means that the conclusion is further strengthened if a choice is true. The
question is phrased in one of the following ways.

1. Which of the following, if true, strengthens the argument?


2. Which of the following, if true, least weakens the argument?
178
3. Which of the following, if true, adds credence to the argument?
4. Which of the following, if true, reinforces the argument?

Example

Astronomer: Most stars are born in groups of thousands, each star in a group forming from the
same parent cloud of gas. Each cloud has a unique, homogeneous chemical composition.
Therefore, whenever two stars have the same chemical composition as each other, they must
have originated from the same cloud of gas.

Which of the following, if true, would most strengthen the astronomer’s argument?

(A) In some groups of stars, not every star originated from the same parent cloud of gas.

(B) Clouds of gas of similar or identical chemical composition may be remote from each other.

(C) Whenever a star forms, it inherits the chemical composition of its parent cloud of gas.

(D) Many stars in vastly different parts of the universe are quite similar in their chemical
compositions.

(E) Astronomers can at least sometimes precisely determine whether a star has the same
chemical composition as its parent cloud of gas.

Solution - Let’s break it down:

Conclusion: “Therefore, whenever two stars have the same chemical composition as each other, they
must have originated from the same cloud of gas.”

Premise: “Most stars are born in groups of thousands, each star in a group forming from the same parent
cloud of gas. Each cloud has a unique, homogeneous chemical composition.”

The given conclusion considers the implication of two stars having similar chemical composition, whilst
assuming on a basic level that if he/she were to compare the chemical composition of the stars and the gas
cloud they seem to have originated from, then they would be exactly the same.

Any answer choice that supports the author’s assumption, will strengthen the argument.

Before you consider each answer choice in turn, remember that your focus should lie with the chemical
composition of the stars, and how if they’re similar to each other and to the origin gas cloud, they’re likely
to have originated in the same cluster.

Option A - cannot clearly portray a significant relevance between where stars are born (different clouds of
gas) and whether they have the same chemical composition. The ambiguity of this option is in fact rather
likely to weaken the argument, than strengthen it.

Option B- focuses on how far apart clouds of gases are from each other; something that has no relevance
to the chemical composition.

179
Option D - also deviated from the main focus of the argument by talking about stars in different parts of
the universe.

In the case of option E- any findings on the astronomers’ part could strengthen or weaken the argument.
Merely their ability to somewhat precisely determine whether a star has the same chemical composition as
its parent cloud of gas, does little to support the author’s conclusion.

Option C - is pretty clearly the winner, because it very explicitly states that “Whenever a star forms, it
inherits the chemical composition of its parent cloud of gas.”. Hence, C is the answer.

Weaken the Argument

In this question type, the conclusion is already in the argument but with a questionable assumption. One of
the choices if true weakens the conclusion. The question is phrased in one of the following ways.

1. Which of the following, if true, weakens the conclusion?


2. Which of the following, if true, least strengthens the argument?
3. Which of the following, if true, jeopardizes the argument?
4. Which of the following, if true, questions the validity of the argument?
5. Which of the following, if true, damages the argument?

Example

In many pre-schools, children commonly tend to get colds before their resistance develops,
and the colds become much less frequent. It is clear that a child requires several colds
before white blood cell concentrations rise high enough to effectively deal with colds.

Which of the following, if true, most seriously weakens this theory?

A. Children commonly spread viruses and bacteria in a small, closed environment.

B. The use of Vitamin C increases resistance to the common cold and decreases its
frequency.

C. Parents stock up on cold medicine that alleviates the symptoms of a cold after a child
gets sick.

D. There are many strains of the cold virus, and children develop resistance to individual
strains.

E. White blood cells fight infection, and their production levels are stimulated by high
infection levels.

Explanation –

The question is stating that the body’s immune system requires numerous infections to be properly
stimulated. It is a causal argument that tries to explain an observation.
180
Premise #1: Children tend to get fewer colds as they progress through pre-school.

Premise #2: (unstated assumption)

Conclusion: It takes several colds to activate a child’s immune system.

Analysis: This is After This, Therefore, Because of This fallacy. It observes that as children go through pre-
school the number of colds goes downs. From this, the creative author develops the theory that a child’s
immune system requires them to get several colds before it is fully activated.

The best way to weaken a causal argument is to suggest an alternative causal factor.

Reviewing Answer Choices

(A) Not relevant.

(B) Not relevant.

(C) This choice presents another possible reason to undermine the argument, but the medicine deals with
symptoms, not the cold per se. So it is not reducing an instance of a cold but simply decreasing its
symptoms (no more runny noses!).

(D) This choice suggests an alternative explanation for the apparent improvement in a child’s ability to fight
colds: the child simply becomes immune to individual viruses. So, the theory that a child’s immune system
needs high white blood cell concentrations isn’t the case; it is an issue of exposure to certain strains. By
suggesting a different causal process to explain the reduction in colds, it weakens the argument.

(E) This choice strengthens the argument, but the question asks for what weakens it.

Trick opposites - are sometimes used as junk answer choices on Strengthen/Weaken questions. If the stem
asks for what weakens the passage, you’ll find a perfect answer choice for what strengthens it, and vice
versa. Choice (E) in the above question about colds is an example of a trick opposite.

Assumption

Assumption questions are often considered some of the hardest among Critical Reasoning questions, and
this comes as no surprise because these questions can be tricky indeed. In Critical Reasoning problems,
an assumption is information not stated in the argument that has to be true in order for the argument’s
logic to hold. Simply put, an assumption is something the argument takes for granted in reaching its
conclusion. Look at this very simple argument to demonstrate what we have just said.

All kids are excited when they get new toys. Therefore, Tim will be thrilled when he gets the new radio-
controlled car we bought him.

This argument has a small gap (scope shift) between the evidence and the conclusion. The evidence is
about kids, and the conclusion based on this evidence is about Tim. One thing this argument expects you
to take for granted, therefore, is that Tim is a child. Without assuming this, the argument would make no
sense, so the argument must assume that Tim is a child.

181
This example demonstrates two more important characteristics of assumptions, which will help you
double-check your answers.

1. Adding the correct assumption to the argument will strengthen the argument.

2. Adding the negated assumption to the argument will weaken the argument.

Let’s first try to add our assumption to the argument:

All kids are excited when they get new toys. Tim is a child. Therefore, Tim will be thrilled when he gets
the new radio-controlled car we got him.

This argument is stronger because the new premise (assumption) eliminates the gap between the evidence
and the conclusion. Now the evidence (first premise) is about children, and the second premise includes
Tim in that category, making the evidence directly applicable to Tim.

[Note: Even though the correct assumption will strengthen the argument, not every answer that
strengthens the argument must be assumed. (Remember: all squares are rectangles but not all rectangles
are squares. Beware of logical fallacies.)]

Negate the assumption

Now, see what happens if you negate the assumption and add it to the argument:

All kids are excited when they get new toys. Tim is not a child. Therefore, Tim will be thrilled when he gets
the new radio-controlled car we got him.

The argument is weaker because its conclusion has become purely speculative. You have no logical
grounds to predict anything about Tim’s reaction, because the second premise (assumption) does not now
connect the first premise (evidence) to the conclusion.

Here’s what you must remember about assumptions, based on what’s covered here…

1. An assumption is information not stated in the argument that must be true for the argument’s
conclusion to hold. An assumption is unstated vital information that the argument takes for
granted.

2. When the correct assumption is added to the argument, it will strengthen the argument.
HOWEVER, not every answer choice that strengthens the argument must be assumed.

3. When the correct assumption is negated, adding it will weaken the argument.

Paying attention to these three rules of thumb will help you identify and double-check the correct answers
to Critical Reasoning assumption questions.

Example

182
Although many doubts have been raised as to the accuracy of age estimation for a recently
found African artefact, all this criticism has no grounds. To estimate the age of the artefact,
archaeologists used a radiocarbon dating method known to be highly accurate in estimating
the age of organic materials.

The conclusion above depends on which of the following assumptions?

A) Modern dating methods are generally more accurate than older dating methods.

B) Written records report that artefacts similar to the one recently found were being used at
the time indicated by radiocarbon dating method.

C) The recently uncovered African artefact is made of organic material.

D) Some of the scientists who doubt the accuracy of the age estimations are very young.

E) Radiocarbon dating cannot accurately estimate the age of objects and materials that are
more than 60,000 years old.

Answer explanation - This is not a very hard question, but before you try to predict a correct answer,
analyze the argument to see what was concluded and what evidence was offered to support that
conclusion. The argument concludes that criticism has no grounds, that the age estimation was accurate.
This conclusion is supported by a single piece of evidence: the radiocarbon dating method used is highly
accurate in estimating the age of organic materials.

If you were careful, you noticed that the evidence was somewhat limited; specifically, the method was said
to be accurate in dating ORGANIC materials. To conclude that the method was accurate for the African
artefact, the argument must assume that the artefact was organic OR that the method effectively used for
dating organic materials is equally effective when it comes to dating non-organic materials. The correct
answer choice therefore must be an assumption that creates one of those connections.

A) Modern dating methods are generally more accurate than older dating methods. This choice is clearly
irrelevant to the argument, since the one method discussed is not compared to any others. You can
eliminate this option without further testing it.

B) Written records report that artefacts similar to the one recently found were being used at the time
indicated by radiocarbon dating method. This one may be tempting because such information, if added to
the argument, would strengthen it. Remember, however, not every strengthening option must be
assumed. Test this option by negating it to see whether it ruins the argument’s logic.

 The criticism has no grounds.

 No written records report that artifacts similar to the one recently found were being used at the
time indicated by radiocarbon dating.

 Radiocarbon dating is highly accurate for estimating age of organic materials.


183
Well, you can see that although adding this information might strengthen the argument, it is not
information that must be assumed. Even if the information is not true, the argument could still be valid.

C) The recently uncovered African artefact is made of organic material. Correct. If you add this information
to the argument, you strengthen it by connecting the stated evidence to the stated conclusion. If you
negate this information and say that the recently uncovered African artefact is NOT made of organic
material, then your evidence becomes irrelevant, as you know only that radiocarbon dating works for
organic materials but have no idea whether it would have worked on the non-organic artefact.

D) Some of the scientists who doubt the accuracy of the age estimations are very young. Even if they all
were very old, this would not change a thing, at least as long as you don’t have information that links
scientists’ age and validity of their doubts.

E) Radiocarbon dating cannot accurately estimate the age of objects and materials that are more than
60,000 years old. You don’t know the age estimated, so this information is irrelevant.

Find the inference

In this question type, the conclusion is already given. If the statements in the argument are true, one of the
choices also must be true. If there are two premises, it is possible to make two inferences. The questions
appear as follows:

1. If the statements in the arguments are true, which of the following must also be true?
2. Which of the following statement can be inferred from the passage?
3. Which of the following is implied in the passage?

Inferences often have very little to do with the conclusion and for each premise, you can have an
inference. So, the inferences made from the premises need to be mapped with the given options to
deduce the right answer option.

Example –

A television news network has recently been criticized for failing to give as much time to
individuals who do not believe climate change is occurring as they do to scientists who
believe climate change is occurring, even though the network does give equal time to all
sides of the debates over immigration, tax reform, and gun policy. The network claims that
they only give equal time to both sides of a debate when one side cannot be definitively
proven by existing scientific research.

Which of the following can be correctly inferred from the information given above?

A. The television news network believes that the existence of climate change has been
definitely proven by existing scientific research.

184
B. The television news network believes that it is important to avoid debate on
scientific discoveries.

C. If the news network gave the same time it gave to scientists to individuals who
don’t believe in climate change, it would increase its ratings.

D. There are no effective counter arguments against climate change that might sway
viewers to believe that climate change is not occurring.

E. No individuals who spoke on the network and who did not believe climate change
was occurring were scientists.

Explanation-

Whenever you are asked to make an inference from an argument presented, remember that inferences
don't need to be interesting or surprising - they only need to be guaranteed.

The argument here states that a television network has been criticized for not giving as much time to
climate change deniers as it does to those who believe in climate change even though they do give equal
time to all sides of the debates surrounding other issues like tax reform and immigration. The network
claims that this is because they only give equal time if one side of the debate cannot be definitively proven
by science.

Since the network does not give equal time to both sides of the climate change debate, that means that
people at the network believe that it fits the exception given and that one side (the side that believes in
climate change) has been definitively proven by science, which matches answer choice A "The television
news network believes that the existence of climate change has been definitely proven by existing
scientific research.".

Among the other answers CHOICE B, "The television news network believes that it is important to avoid
debate on scientific discoveries." can be discarded because while the network does not give equal time for
all sides of the debate in some cases, it does not provide a blanket dismissal of debating all scientific
discovery.

Choice D "There are no effective counter arguments against climate change that might sway viewers to
believe that climate change is not occurring." can be eliminated since while the network believes the
science has been settled, this is not the same thing as claiming that no one will be swayed by the
arguments against climate change.

Choice E "No individuals who spoke on the network and who did not believe climate change was occurring
were scientists." is a bit harder to eliminate since the stimulus tells you that the people who believe that
climate change is occurring are scientists, but it doesn't specifically say that the individuals who don't
believe in climate change aren't scientists, so "No individuals who spoke on the network and who did not
believe climate change was occurring were scientists." can be eliminated.

185
Choice C "If the news network gave the same time it gave to scientists to individuals who don’t believe in
climate change, it would increase its ratings." can also be eliminated since even though some people
criticize the network for its current policy, it is unknown how a change in policy would affect overall
criticism of the network (and in turn how that would effect ratings).

Paradox questions

Paradox/Discrepancy questions will ask you to choose the answer choice that explains the paradox in the
given argument. A paradox refers to the coexistence of two seemingly contradictory pieces of information.
The correct answer choice will logically explain why those pieces of information are not actually
contradictory. Key words you might see are explain, paradox, or discrepancy.

A Paradox question may be worded as:

1. Which of the statements below provides the most likely explanation for the two seemingly
contradictory statements above?
2. Which of the following, if true, most helps to resolve the paradox outlined above?
3. Which of the following, if true, best explains the reason for the apparent discrepancy described
above?

EXAMPLE-

In 2009, there was a sharp increase in the number of people who were reported to have
died in the country during the first week of the year, compared with the first week of the
previous ten years. There seems to be no reason to explain this disproportionately high
number of deaths especially because the reports were widespread in the country and
there was no epidemic spreading through the country at that point in time. Moreover,
most of the deaths were not among the young and could not even be attributed to binge
drinking in celebration of the new year.

Which of the following can then explain the discrepancy?

A. The government announced in 2008 that estate taxes, payable by the heirs on
someone's death, would be abolished effective from the new year.

B. Astrologists predicted that 2009 was a good year to die in, for salvation of the
soul.

C. The statistic is an anomaly that has no logical explanation.

D. The price of some cancer drugs and chemotherapy increased by 10% in January
2009, making treatment more expensive than before.

E. An earthquake in one of the cities during December 2008 made hospital care
availability more difficult for regular illnesses.

186
EXPLANATION –

Step 1: Analyzing the Argument

For no apparent reason, there seems to have been an increase in the number of reported deaths in the
first few weeks of 2009. The argument eliminates some likely reasons, such as spreading epidemics. The
correct option must explain the reason for the abnormal number of reported deaths.

Step 2: Eliminating Options

Option (B) does not seems to imply that people voluntarily died to satisfy the astrological predictions. Even
if we were to accept that to be true, it does not explain why there was an increase in people dying right
after the year begins. If the entire year is a good year to die in, why not die in December 2009?

The question asks for an explanation and saying that there is no valid explanation is no good. Option (C)
can be eliminated.

Option (D) is an interesting, attractive option. However, the problem with the option is the timeline. If
treatment became expensive only that year, why are people dying immediately? Could they not have done
something to improve their health or sought other options?

Option (E) discusses only one city while the argument clearly states that this phenomenon was widespread
and observed throughout the country. The option therefore, does not justify why the numbers went up in
other parts of the country.

To understand why Option (A) works, understand that the argument does NOT state that the number of
deaths went up but rather that the REPORTS went up. If option (A) is true and legal heirs do not have to
pay any taxes from January 1, 2009, that would explain why the reports of deaths have gone up. Some
people could have died in the last few days of 2008 and the heirs could have waited to report it. Also
notice that the argument indicates that the reports were primarily about the older population.

Option A is the correct answer.

Tips and tricks to understand and solve critical reasoning

1) Read the question stem first.

This will allow you quickly categorize the type of Critical Reasoning question (Strengthen, Weaken,
Inference, etc.) and let you focus on identifying the premises in the stimulus. Questions such as, “Which of
the following can be correctly inferred from the statements above?” and, “If the statements above are
true, which of the following must also be true?” signify that you are dealing with an Inference question.

2) Speculate what you think the correct conclusion is.


187
Sometimes this may be difficult to verbalize, but having an outline or framework of what the “must be
true” answer should include will help to eliminate some answer choices.

3) Evaluate the answer choices using your speculated answer.

You want to carefully read all 5 answer choices. As you read the answers, compare them to the answer, or
the outline of the answer, you speculated. Some answers are obviously incorrect – either they are too
narrow in scope, too extreme to be always be true, or do not follow the criteria laid out in the stimulus.
Eliminate these answers. For other answer choices that seem attractive, keep them as possibilities. Once
you have read all of the answer choices, you can then compare your list of possible answers using the
criteria that the correct answer must be always be true.

4) Become a Defence Lawyer.

When comparing your list of possible answers, try to come up with plausible scenarios that would prove
the answer being considered not true. Just because the stimulus says that “everyone sitting in the dentist’s
office waiting room at 9:00 a.m. was a patient” does not necessarily mean that they were waiting for an
appointment. Some could have already finished their appointment, and some could have been there
dropping off another patient. Like a defence lawyer, you need to find every scenario in which an answer
choice might not be true in order to eliminate it from your options.

5) Be aware of exaggerated or extreme answers.

Because the correct answer must always be true, modifiers that exaggerate an element of the premise or
make an extreme claim usually signify an incorrect answer. If the stimulus says, “Some of the widgets
produced by Company X were defective,” an attractive, yet incorrect answer choice may exaggerate this
statement with a modifier such as “most” by claiming, “Most of Company X’s widgets were found to be
defective.” Furthermore, answers that include the terms “always”, “never”, “none” and the like are good
indicators that the answer will not be true 100% of the time.

6) Be aware of answers that change the scope of the stimulus.

On more difficult Inference questions (as if they were not difficult enough), the test makers will tempt you
to select an answer choice that slightly changes an element of the facts laid out in the stimulus. For
example, the stimulus might discuss the decrease in the violent crime rate in City A over a certain time
period.

The attractive answer that follows all of the elements of having to be true 100% of the time, but is still
incorrect might discuss decrease in the murder rate of City A over that time period. While the answer
would seem to fit the bill, the murder rate is not the same as the rate of violent crime – this changes the
scope of the initial stimulus and we can therefore rule that answer out.

The correct inference or conclusion on Critical Reasoning Inference questions is very close to what is stated
explicitly in the stimulus. Remember, the right answer choice on these question types must be true 100%
of the time.

188
Tutorial
Exercise 1

Q1. One summer, floods covered low-lying garlic fields situated in a region with a large mosquito
population. Since mosquitoes lay their eggs in standing water, flooded fields would normally attract
mosquitoes, yet no mosquitoes were found in the fields. Diallyl sulfide, a major component of garlic, is
known to repel several species of insects, including mosquitoes, so it is likely that diallyl sulfide from the
garlic repelled the mosquitoes.

Which of the following, if true, most strengthens the argument?

(A) Diallyl sulfide is also found in onions but at concentrations lower than in garlic.

(B) The mosquito population of the region as a whole was significantly smaller during the year in which the
flooding took place than it had been in previous years.

(C) By the end of the summer, most of the garlic plants in the flooded fields had been killed by waterborne
fungi.

(D) Many insect species not repelled by diallyl sulfide were found in the flooded garlic fields throughout
the summer.

(E) Mosquitoes are known to be susceptible to toxins in plants other than garlic, such as marigolds.

Q2. A study revealed that a major part of the accidents involving two wheelers takes place when they are
being driven at speeds above 50 km.hr. It is also observed that the two wheelers are driven at this speed
mostly on roads having scanty traffic.

Which of the following can be concluded from the given paragraph?

A. Accidents do not take place because of heavy traffic.

B. It is not possible to drive two wheelers at speeds above 50 km/hr on the roads having heavy traffic.

C. Two wheelers drivers tend to become reckless while driving on roads having scanty traffic.

D. Most of the accidents involving two wheelers take place on roads having scanty traffic.

E. Accidents involving four wheelers also take place at speeds above 50 km/hr.

Q3. The Supreme Court in its land mark judgement gave women the right to terminate their pregnancy as a
constitutional right.

Which of the following, if true, can go against the Supreme Court's judgement?

A. In several other countries abortion is illegal.

189
B. An unborn child should be treated as a person and every person has a fundamental right to live.

C. The elected representatives of one of the states rejected the proposal to make abortion illegal in a 30-70
vote.

D. If a foetus can be treated as a person, an embryo from which the foetus is formed should also be
treated as a person.

E. None of these

Q4. The number of instances where judges have made unwanted remarks in open courts against other
constitutional/ statutory bodies or persons who were not before them is increasing. There is a need to
bring such behaviour of judges within the purview of judicial standards.

Which of the following can be inferred/concluded from the given information?

A. There is a set of judicial standards which the judges are expected to follow

B. Till now no one has set any standards for judges.

C. It is not desirable for the judges to pass such comments on other constitutional/ statutory bodies in their
absence.

D. Freedom of expression cannot be exercised while in official capacity.

E. None of these

Q5. Insurance Company X is considering issuing a new policy to cover services required by elderly people
who suffer from diseases that afflict the elderly. Premiums for the policy must be low enough to attract
customers. Therefore, Company X is concerned that the income from the policies would not be sufficient
to pay for the claims that would be made.

Which of the following strategies would be most likely to minimize Company X's losses on the policies?

A. Attracting middle-aged customers unlikely to submit claims for benefits for many years.

B. Insuring only those individuals who did not suffer any serious diseases as children

C. Including a greater number of services in the policy than are included in other policies of lower cost

D. Insuring only those individuals who were rejected by other companies for similar policies

E. Insuring only those individuals who are wealthy enough to pay for the medical services

Q6. A conservation group in the United States is trying to change the long-standing image of bats as
frightening creatures. The group contends that bats are feared and persecuted solely because they are shy
animals that are active only at night.

Which of the following, if true, would cast the most serious doubt on the accuracy of the group's
contention?

190
A. Bats are steadily losing natural roosting places such as caves and hollow trees and are thus turning to
more developed areas for roosting.

B. Bats are the chief consumers of nocturnal insects and thus can help make their hunting territory
more pleasant for humans.

C. Bats are regarded as frightening creatures not only in the United States but also in Europe, Africa,
and South America.

D. Raccoons and owls are shy and active only at night; yet they are not generally feared and persecuted.

E. People know more about the behavior of other greatly feared animal species, such as lions, alligators,
and greatly feared animal species, such as lions, alligators, and snakes, than they do about the behavior of
bats.

Q7. Toughened hiring standards have not been the primary cause of the present staffing shortage in public
schools. The shortage of teachers is primarily caused by the fact that in recent years teachers have not
experienced any improvements in working conditions and their salaries have not kept pace with salaries in
other professions.

Which of the following, if true, would most support the claims above?

A. Many teachers already in the profession would not have been hired under the new hiring standards.

B. Today more teachers are entering the profession with a higher educational level than in the past.

C. Some teachers have cited higher standards for hiring as a reason for the current staffing shortage.

D. Many teachers have cited low pay and lack of professional freedom as reasons for their leaving the
profession.

E. Many prospective teachers have cited the new hiring standards as a reason for not entering the
profession.

Q8. Many business offices are located in buildings having 2-8 floors. If a building has more than 3 floors, it
has a lift.

If the above statements are true, which of the following must be true?

A. 2nd floors do not have lifts


B. 7th floors have lifts
C. Only floors above the 3rd floors have lifts
D. All floors may be reached by lifts

Q9. If highways were restricted to cars and only those trucks with capacity of less than 8 tons, most of the
truck traffic would be forced to run outside highways. Such a reduction in the amount of truck traffic would
reduce the risk of collisions on highways.

The conclusion drawn in the Ist sentence depends on which of the following assumptions?

A. The roads outside highway would be as convenient as highway for most drivers of trucks..
191
B. Most of the roads outside highways are not ready to handle truck traffic.
C. Most trucks that are currently running in highway have a capacity of more than 8 tons.
D. Cars are at greater risk of being
involved in collisions than are trucks.

Q10. There are numerous reasons why individuals want to run their own businesses. Some foresee more
personal satisfaction if they succeed in launching their own business, while others are mainly interested in
the prospect of larger financial rewards. Since 1980s and early 1990s, tax regulation and liberal policies
have encouraged increasing number of venture capitalists and entrepreneurs to start new enterprises.
Since 1990, one and a half million new ventures have been started. Not all have succeeded.

The above statement makes which of the following assumptions?

A. Success in starting a new business largely depends on sound financial planning


B. Venture capitalists are motivated by non-monetary gains
C. Social incentives motivate investors just as much as financial rewards
D. None of these

Q11. If highways were restricted to cars and only those trucks with capacity of less than 8 tons, most of the
truck traffic would be forced to run outside highways. Such a reduction in the amount of truck traffic would
reduce the risk of collisions on highways.

Which of the following, if true, would most strengthen the conclusion drawn in the second sentence?

A. Highways are experiencing overcrowded traffic mainly because of sharp increases in car traffic
B. Many drivers of trucks would rather buy trucks with a capacity of less than 8 tons than be excluded
from highways.
C. The number of collisions that occur near highways has reduced in recent years
D. Trucks that have a capacity of more than 8 tons cause a disproportionately large number of
collisions on highways

Q12. Offshore oil-drilling operations involve an unavoidable risk of an oil spill, but importing oil on tankers
presently involves an even greater such risk per barrel of oil. So, if we are to lessen the risk of an oil spill
without reducing our oil usage, we must invest more in offshore operations and import less oil on tankers.

Which of the following, if true, seriously weakens the argument above?

A. Importing oil on tankers is currently less expensive than drilling for it offshore.
B. Tankers can easily be redesigned so that their use entails less risk of an oil spill.
C. The impact of offshore operations on the environment can be reduced by careful management.
D. Oil spills caused by tankers have generally been more serious than those caused by offshore
operations.

Q13. By this work "light would be thrown on the origin of man and his history;" and this implies that man
must be included with other organic beings in any general conclusion respecting his manner of appearance
on this Earth. Now the case wears a wholly different aspect.
Which of the following best captures the essence of the passage?

192
A. Man must be included with other organic beings in any general conclusion respecting his manner of
appearance on this Earth

B. Man must be included with other organic beings

C. Man need not be included with other inorganic beings in any general conclusion respecting his
manner of appearance on this Earth

D. Man must be excluded from other organic beings in any general conclusion respecting his manner
of appearance on this Earth

Q14. The sole object of this work is to consider, first, whether man, like every other species, is descended
from some pre-existing form; secondly, the manner of his development; and thirdly, the value of the
differences between the so-called races of man,
Which of the following best captures the essence of the passage?

A. The work seems to locate whether man is descended from some pre-existing form

B. The work seems to locate whether man is descended from gods, the manner of his development,
and the value of the differences between the so-called races of mammals

C. The work will not locate whether man is descended from some pre-existing form, but the manner
of his development

D. The work seems to locate whether man is descended from some pre-existing form, the manner of
his development, and the value of the differences between the so-called races of man

Q15. The sense of smell is of the highest importance to the greater number of mammals. But the sense of
smell is of extremely slight service, if any, even to the dark coloured races of men, in whom it is much more
highly developed than in the white and civilised races. M. Houzeau asserts that he repeatedly made
experiments, and proved that Negroes and Indians could recognise persons in the dark by their odour.
Which of the following best captures the essence of the above psssage?

A. Sense of smell, as experiments have shown, is directly related to the colour of the skin and the
darker the skin, the greater the sense will be

B. Sense of smell, as experiments have shown, is directly related to the colour of the skin and the
darker the skin, the lesser the sense will be

C. Sense of smell, as experiments have shown, is not related to the colour of the skin at all

D. Sense of smell, theory suggests, is directly related to race and the darker the skin, the lesser the
sense will be

Q16. Persons who consume large amounts of Tablet ‘F’ often have fewer colds and more cheerful
dispositions than the average person. Therefore, I am going to start taking extra 'Tablet F' right away
because I am tired of getting depressed.
The author makes which of the following assumptions?

A. Tablet F cures colds


193
B. Tablet F is necessary in extra supplements.

C. Tablet F helps to prevent depression

D. Tablet F quickens healing from surgery.

E. It is impossible to have side effects of Tablet F and is therefore safe.

Q17. Many people argue that the death penalty deters murder. However, the notorious killer Tom Hanks
deliberately moved to a state that imposes the death penalty just before embarking on a series of
ferocious murders. Thus, it seems clear that the existence of the death penalty does not serve as a
deterrent to murder.
The argument above may best be characterized as:

A. An appeal to emotion

B. A flawed analogy

C. A general conclusion based on a specific example

D. Circular reasoning

Q18. "It was AC Milan's success in Europe in the sixties that introduced the 'Iibero' as the Italian default
and, a quarter of a century later, it was AC Milan's success in Europe that killed it off."

Which of the following was not implied in the above sentence?

A. AC Milan was successful in Europe in the sixties with the help of a ‘Libero'

B. AC Milan was successful in Europe in the late eighties without a ‘Libero'

C. The ‘Libero' was an Italian default for almost a quarter of a century

D. AC Milan was not successful in Europe for almost a quarter of a century

E. The Italians seldom used a ‘Libero' since late eighties

Q19. Alfredo, Diego and Lionel are discussing Argentinean football.


Alfredo: Argentina was a football powerhouse. Diego: Argentina is a football powerhouse.
Lionel: Argentina will be a football powerhouse.
Which of the following cannot be inferred from the above conversation?

A. Lionel is optimistic about the future

B. Alfredo, Diego and Lionel may disagree on certain things

C. Diego views the present positively

D. Alfredo completely disagrees with Diego

E. Alfredo has positive feelings about the past

194
Q20. No national productivity measures are available for underground industries that may exist but remain
unreported. On the other hand, at least some industries that are run entirely by self-employed
industrialists are included in national productivity measures.

From the information given above, it can be validly concluded that

A. there are at least some industries run entirely by self-employed industrialists that are underground
industries

B. not all industries that are run entirely by self-employed industrialists operate underground

C. not all industries that are run entirely by self-employed industrialists are included in national
productivity measures

D. there are at least some industries run entirely by self-employed industrialists that are not
underground industries

Q21. In Noida, a political candidate who used radio advertising was found to get maximum name
recognition.

The statement above logically conveys which of the following ?

A. Radio advertising is the most important factor in political campaigns in Noida

B. Maximum name recognition in Noida will help a candidate to win a higher percentage of votes cast
in the city

C. Radio advertising reaches every demographically distinct sector of the voting population of Noida

D. For maximum name recognition a candidate need not spend on media channels other than radio
advertising

Q22. Veganism is a type of vegetarian diet that excludes meat, eggs, dairy products and all other animal-
derived ingredients. Many vegans also do not eat foods that are processed using animal products, such as
refined white sugar and some wines. Most vegans also avoid the use of all products tested on animals, as
well as animal-derived non-food products, such as leather, fur and wool. A vegan diet will include all types
of grains, beans, legumes, vegetables and fruits and the nearly infinite number of dishes made by
combining them. Many vegan versions of familiar foods are available, so you can eat vegan hot dogs, ice
cream, cheese and vegan mayonnaise. Although there is some debate as to whether certain foods, such as
honey, fit into a vegan diet, if you are cooking for other vegans, it is best to err on the side of caution and
avoid them.

Q. Based on the text, which statement is definitely true?


A) Vegans do not eat honey
B) Some vegans eat honey
C) There is a vegan version of honey
D) There isn't a vegan version of honey

195
Q23. Drug addiction, like other habits such as smoking and drinking, almost always begins under peer
pressure with teenagers being influenced by those of their high school and college colleagues who are only
slightly older to join the “gang” and become adults, so to say. Even if in the initial stages, those youngsters
who have been induced to start taking drugs do not find it pleasurable, they are unwilling to break away
lest they offend seniors.

Which of the following, if true, will most weaken the above argument?

a) Some teenagers could not break away from the habit of drug addiction in fear of offending their seniors.

b) Habit of drug addiction begins under pressure.

c) Teenagers are influenced to choose drugs by slightly older people to make them join the gang.

d) Teenagers willingly choose drugs for pleasure.

Q24. The fact of the matter is that there is a strong case for increasing the farm subsidies, at least
indirectly. The Indian farm sector is now passing through a difficult period, as underscored by the suicides
of farmers reported from states like Maharashtra and Andhra.

Which of the following would strengthen the above argument?

a) Increasing farm subsidies will put fiscal pressure on the government.

b) Increasing farm subsidies will reduce the incidences of farmer suicide.

c) Increasing farm subsidies is not the ultimate solution for reducing the incidences of farmer suicide.

d) Increasing farm subsidies will help the government to increase its revenue.

Q25. There has been a high incidence of death in Rajasthan due to tobacco eating last year”. Hence, the
Rajasthan’s Chief Minister suggested that tax rates on dangerous tobacco products like cigarette and gutka
should not to be reduced as planned by the government.

Which of the following, if true, will most weaken the above argument?

a) Rajasthan Government is trying to reduce the sale of tobacco in the country.

b) Incidences of death due to tobacco eating were high last year as compared to previous year.

c) The adults and teenagers have easy access to tobacco products.

d) Rajasthan is the only state where the sale of tobacco products is the lowest.

Q26. The increase in the number of reality shows on television channels bolsters the contention that
channels owners are more interested in boosting their revenues by pandering to voyeuristic tendencies of
viewers.

The premise behind the above argument is that

A. reality shows on television channels are a recent phenomenon.

196
B. everything that a channel broadcasts should be educational.

C. reality shows on television appeal to the basic instincts of viewers.

D. reality shows make more money than other types of programs.

E. the channel owners can influence what is watched by the viewers.

Q27. Unless you dedicate your whole life to it, you will never learn to speak the language of another
country to perfection; you will never know its people and its literature with complete intimacy.

Which of the following is likely to undermine the above argument?

A. I can speak 10 foreign languages already

B. I do not travel to foreign countries.

C. I am happy with the languages I know and do not need to learn any other language.

D. I should spend time to understand my own people and literature first, only then can I appreciate other
languages and cultures.

Q28. Read the below passage carefully and answer the questions:

Giving children computers in grade school is a waste of money and teachers time. These children are too
young to learn how to use computers effectively and need to spend time on learning the basics, like
arithmetic and reading. After all, a baby has to crawl before she can walk.

Which of the following, if true, would strengthen the speaker's argument?

A. studies showing computers are expensive

B. research on the effect of computer games on children

C. examples of high school students who use computers improperly

D. proof that the cost of computers is coming down

E. evidence that using computers makes learning to read difficult

Q29. Read the below passage carefully and answer the questions:

Giving children computers in grade school is a waste of money and teachers time. These children are too
young to learn how to use computers effectively and need to spend time on learning the basics, like
arithmetic and reading. After all, a baby has to crawl before she can walk.

Which of the following, if true, would weaken the speaker's argument?

A. a demonstration that computers can be used to teach reading and arithmetic

B. analysis of the cost-effectiveness of new computers versus repairing old computers

197
C. examples of adults who do not know how to use computers

D. recent grade reports of students in the computer classes

E. a visit to a classroom where computers are being used

Q30. If you're a fitness walker, there is no need for a commute to a health club. Your neighborhood can be
your health club. You don't need a lot of fancy equipment to get a good workout either. All you need is a
well-designed pair of athletic shoes.

This paragraph best supports the statement that

A. fitness walking is a better form of exercise than weight lifting.

B. a membership in a health club is a poor investment.

C. walking outdoors provides a better workout than walking indoors.

D. fitness walking is a convenient and valuable form of exercise.

E. poorly designed athletic shoes can cause major foot injuries.

Q31. In the past, consumers would rarely walk into an ice cream store and order low-fat ice cream. But
that isn't the case today. An increasing health consciousness combined with a much bigger selection of
tasty low-fat foods in all categories has made low-fat ice cream a very profitable item for ice cream store
owners.

This paragraph best supports the statement that

A. low-fat ice cream produces more revenue than other low-fat foods.

B. ice cream store owners would be better off carrying only low-fat ice cream.

C. ice cream store owners no longer think that low-fat ice cream is an unpopular item.

D. low-fat ice cream is more popular than other kinds of ice cream.

E. consumers are fickle and it is impossible to please them

Q32. Today's workforce has a new set of social values. Ten years ago, a manager who was offered a
promotion in a distant city would not have questioned the move. Today, a manager in that same situation
might choose family happiness instead of career advancement.

This paragraph best supports the statement that

A. most managers are not loyal to the corporations for which they work.

B. businesses today do not understand their employee’s needs.

C. employees social values have changed over the past ten years.

D. career advancement is not important to today's business managers.

198
E. companies should require their employees to accept promotions.

Q33. The program to control the entry of illegal drugs into the country was a failure in 1987. If the program
had been successful, the wholesale price of most illegal drugs would not have dropped substantially in
1987.

The argument in the passage depends on which of the following assumptions?

A. The supply of illegal drugs dropped substantially in 1987.

B. The price paid for most illegal drugs by the average consumer did not drop substantially in 1987.

C. Domestic production of illegal drugs increased at a higher rate than did the entry of such drugs into the
country.

D. The wholesale price of a few illegal drugs increased substantially in 1987.

E. A drop in demand for most illegal drugs in 1987 was not the sole cause of the drop in their wholesale
price.

Q34. A factory was trying out a new process for producing one of its products, with the goal of reducing
production costs. A trial production run using the new process showed a fifteen percent reduction in costs
compared with past performance using the standard process. The production managers therefore
concluded that the new process did produce a cost savings.

Which of the following, if true, casts most doubt on the production managers' conclusion?

A. In the cost reduction project that eventually led to the trial of the new process, production managers
had initially been seeking cost reductions of fifty percent.

B. Analysis of the trial of the new process showed that the cost reduction during the trial was entirely
attributable to a reduction in the number of finished products rejected by quality control.

C. While the trial was being conducted, production costs at the factory for a similar product, produced
without benefit of the new process, also showed a fifteen percent reduction.

D. Although some of the factory's managers have been arguing that the product is out dated and ought to
be redesigned, the use of the new production process does not involve any changes in the finished
product.

E. Since the new process differs from the standard process only in the way in which the stages of
production are organized and ordered, the cost of the materials used in the product is the same in both
processes.

Exercise 2

Directions - Cause and effect In each of the following questions, two statements numbered I and II are
given. There-may be cause and effect relationship between the two statements. These two Statements
may be the effect of the same cause or independent causes. These statements may be independent causes
without having any relationship. Read both the statements in each question and mark your answer as:

199
If statement I is the cause and statement II is its effect;

If Statement II is the cause and statement I is its effect;

If both the statements I-and II are independent causes;

If both the statements I and II are effects of independent causes; and

If both the statements I and II are-effects of some common cause.

1. I. The prices of vegetables have been increased considerably during this summer.
II. There is tremendous increase in the temperature during this summer thereby damaging crops
greatly.

2. I. Police; resorted to lathi-charge to disperse the unlawful gathering of large number of people.
II. The citizens' forum called a general strike in protest against the police atrocities.

3. I. It is the aim of the city's civic authority to get the air pollution reduced by 20% in the next two
months.
II. The number of asthma cases in the city is constantly increasing.

4. I. The police authority has recently caught a group of house breakers.


II. The citizens group in the locality have started night vigil in the area.

5. I. The university authority has instructed all the colleges under its jurisdiction to ban use of all
phones inside the college premises.
II. Majority, of the teachers of the colleges signed a joint petition to the university complaining the
disturbances caused by cell phone ring-tones inside the classrooms.

6. I. The government has recently fixed the fees for professional courses offered by the unaided
institutions which are much lower than the fees charged last year.
II. The parents of the, aspiring students launched a severe agitation last year protesting against the
high fees charged by the unaided institutions.

7. I. Large number of people living in the low-lying areas have been evacuated during the last few days
to safer places.
II. The Government has rushed in relief supplies to the people living in the affected areas.

8. The performance of most of the students in final exam of class X in the schools run by the
Government was excellent.
II Many teachers of the Government schools left the school and joined private schools.

9. I. Majority of the citizens in the locality belong to higher income group.


II. if The sales in the local super market are comparatively much higher than in other localities.

10. I. There is considerable reduction in the number of people affected by water-borne diseases in City
A during this rainy season.
II. The government has opened four new civil hospitals in City A in the beginning of the year.

200
11. I. The literacy rate in the district has been increasing for the last four years.
II. The district administration has conducted extensive training programme for the S workers
involved in the literacy drive.

12. I. Most of the steel producing companies in the country have made considerable profit during the
last financial year.

II. Many Asian countries have been importing huge quantities of steel from India

13. I. Rural and semi-urban areas in the country have been suffering due to load shedding for quite
some time.

II. If the Government is not able to overcome the power crisis, load shedding will be extended
even to the urban areas.

14. I. The employees of the biggest bank in the country have given an indefinite strike call starting from
third of the next month.

II. The employees of the Central Government have withdrawn their week long demonstrations.

15. I. The school authority has asked the X Std. students to attend special classes to be conducted on
Sundays.

II. The parents of the X Std. students have withdrawn their wards from attending private tuitions
conducted on Sundays.

16. I. The farmers have decided against selling their kharif crops to the Government agencies.

II. The Government has reduced the procurement price of kharif crops starting from last month to the
next six months.

17. I. The government has decided to make all the information related to primary education available
to the general public.

II. In the past, the general public did not have access to all these information t elated to primary
education.

18 I. There is sharp decline in the production of oil seeds this year.

II. The Government has decided to increase the import quantum of edible oil.

19. I. The life today is too fast, demanding and full of variety in all aspects which at times beads to
stressful situations.

II. Number of suicide cases among teenagers is on increase.

20. I. There is unprecedented increase in the number of young unemployed in comparison to the
previous year.

II. A large number of candidates submitted applications against an advertisement for the post of
manager issued by a bank
201
Exercise 3

Q1. The Americans with disabilities act (ADA) was designed to ensure that there is no discrimination
against and unfair termination of differently-abled workers in the workplace. However, after the act was
introduced, there has been a marked increase in unemployment among the differently-abled.

Which of the following best explains this seeming discrepancy?

A. A number of differently-abled people chose not to work


B. Not willing to deal with the issues of workplace discrimination of the differently-abled, several
companies recruited fewer of them in the first place
C. Knowing that the act was about to be enforced, companies terminated some of the differently-
abled while they had a chance
D. There was no act introduced that would guarantee a job for the differently-abled
E. The unemployment among the able-bodied has remained consistently high

Q2. In an effort to curb drug abuse, the government has imposed strict laws to prosecute the dealers.
However, such an initiative is unlikely to be effective. Prosecuting dealers will lead to a shortage of drugs.
At the same time, because no efforts are being taken to curb demand, drugs will be sold at a premium,
attracting more people to the very remunerative job of drug dealing. Therefore, to effectively reduce drug
abuse, the government will have to prosecute the drug users and not dealers.

Which of the following is the most relevant information in evaluating the credibility of the argument?

A. Whether efforts have been successfully taken in any other country to regulate drug users.

B. Whether the payoff from selling drugs outweighs the severity of the punishment

C. Whether drugs will continue to be sold at a premium when there are dealers in the market again

D. Whether the majority of the users will be willing to pay a premium to continue to use the drugs

E. Whether the government will be able to keep track of new dealers as and when they enter the
market

Q3. Alan: In the last 15 years, most of the criminals who were convicted of theft or murder were from the
lower income classes and had not completed high school. Therefore, the government has to spend more
money on reducing poverty and increase funding to education. Because terrorism is the most severe of all
crimes, such measures would bring down overall crime rate and reduce threat from terrorism.

Dylan: A study that was conducted in a country known to produce a number of terrorists showed that on
average the terrorists were better educated than the overall population and that they did not necessarily
come from lower income classes. This is probably because crimes such as theft are committed for personal
gain while terrorism is for political or religious gain.

Which of the following best describes Dylan's response to Alan?

202
A. Dylan changes the direction of the argument entirely by discussing the scenario in a different
country

B. Dylan partially agrees with Alan's reasoning but refutes his recommendation to the government

C. Dylan converts a causal argument made by Alan into a generalization applicable universally

D. Dylan challenges Alan's reasoning by explaining why two situations that Alan perceives as similar
are not

E. While Alan arrives at a conclusion by drawing an analogy, Dylan arrives at the same conclusion by
refuting the analogy

Q4. During the last 50 years in England, the national football team has had at least 60℅ of its players born
during the months January to March. Similarly, in Germany, at least 50℅ of the team has been born during
the first three months of the year. In fact, this statistic holds true for most European countries. This shows
that in most European countries, parents with kids born early in the year are more likely to encourage a
football career.

Which of the following best explains why the conclusion need not be the best explanation for the
statistic?

A. Of the remaining members, 60℅ or more tend to be born between the months of April and June.

B. Parents of kids born in the second half of the year have displayed a measurable intent to promote
scholarly careers

C. The age cut-off for kids to try out for a team in most European countries is calculated as on
December 31 of any year and older the kid during trials, greater the chances of getting the right
opportunities.

D. The school year usually begins in April in most European countries and many parents are known to
encourage students to work on their academics and not sports during the school year.

E. During the last 50 years, on average 30℅ of the kids have been born during the first three months
of the year.

Q5. During medieval times, the administrative system was organized such that jobs were traditionally held
within the same family. The eldest son of the village's blacksmith will take up his father’s business and
become the next blacksmith. The other sons would join the army or serve the king in some fashion while
the daughters did what their mother did. Although the world has undergone innumerable changes, the
dynastic system has not undergone any change whatsoever. Children who have fathers who played major
league baseball are 800 times more likely than other kids to become major league players themselves.

Which of the following best refutes the author's reasoning?

A. In countries with a royal family, the eldest son of the king is destined to be the next king.

B. The blacksmith's eldest son in a mediaeval village could not choose any other profession even if he
wanted to.
203
C. A major baseball player will have better knowledge and skills and will be able to guide his son
better.

D. 60℅ of the country's doctors have at least one parent who is a doctor.

E. It is a proven fact that success in any career is determined more by skill and practice rather than by
genetic makeup.

Q6. Terrorist attacks invariably lead to tremendous losses in life, property, and morale of a country. The
effects of a terrorist attack are not just immediate and can have long-lasting, trickle-down effects as well.
The fear, for example, takes a long time to die down. However, some of these repercussions can be
beneficial to the country. Take for instance, the recent terrorist attack on our capital city. In the weeks
following the attack, the crime rate in the city came down significantly from what it was just before the
attack. This must primarily be due to the increased presence of police resources that were moved to the
area and is thus an indirect effect of the attack.

Which of the following options gives one more option as to why the crime rate decreased because of the
terrorist attacks?

A. The capital city is under increased monitoring leading to quick detection of crimes - many times
while the crime is still underway.

B. A number of people are frightened because of the terrorist attacks and have fled the capital city.

C. There was a recorded decrease in crime rate right after the terrorist attack in almost all cities of the
country.

D. Intel reports show that the terrorists who pulled off the attack had been committing various other
smaller crimes regularly to distract law enforcement from their true purposes.

E. The government had initiated schemes to decrease poverty and provide better livelihood for the
people just before the terrorist attack.

Q7. It seems that medical care is constantly improving and is far better today than it was a few years ago.
New medicines are being developed, and at a lower cost. Technological advancements are being made and
new treatments are devised. However, hospital care has not improved as much as expected. Most
hospitals do not have a large enough ER that can handle a sudden influx of patients and the number of
people who die in a hospital due to reasons other than what they got admitted for is staggeringly high.

All of the following underscore the author’s argument EXCEPT

A. Most hospitals require doctors to do so much paperwork that they spend insufficient time on direct
patient care.

B. Most hospitals recirculate air and therefore, it is very easy for diseases such as anthrax to spread
through the hospital.

C. In smaller cities, the ambulance service is on a voluntary basis and the response time for calls is
much lower than the national average, leading to a higher loss of life.

204
D. A number of doctors in private hospitals are not held accountable by the administration for the loss
of patients.

E. Doctors in most private hospitals have performance appraisals based on how much money they
have brought in for the hospital rather than based on how effectively they treated patients.

Q8. Industrial and automobile pollution have long been thought to contribute to global warming. However,
researchers have identified that the carbon dioxide and carbon monoxide released into the atmosphere
are not as potent as the methane emitted by cud-chewing animals such as cows when they fart or belch.
Therefore, if you drive a hybrid electric car to the grocery, any favor that you would do to the environment
would be offset if you end up buying beef.

Which of the following best describes the author's reasoning?

A. The author establishes a point by drawing an analogy

B. The author disproves a popular notion by providing evidence that is contrary to it

C. The author presents a new theory in response to an existing well-established theory

D. The author uses an illustration to support a new school of thought that is contrary to a popular
school of thought

E. The author makes a comparison between two theories by providing an example

Q9. There are several scientific studies and research findings that are constantly discussed and publicized
in the news media. Some of these are contrary to other research studies that are published. This conflict in
information makes people believe that either science is inaccurate or that they can ‘choose’ which
scientific result they want to believe in. However, neither of these beliefs is accurate. Science is not
inaccurate or subjective. Most of the time, scientific studies show inconsistent results either because of
inaccuracies in methodology adopted or because of misrepresentation of actual results by the news media.
For example, a recent study done on just 15 women showed that eating chocolate was not necessarily
harmful during pregnancy and the media reported that chocolate was actually beneficial to the foetus.

Which of the following best further corroborate the author’s argument?

A. A study result that shows that the effect of automobile pollution on global warming is marginal.

B. An unverified study gaining popularity because it seemingly showed that red wine reduces the risk
of cancer on a very small control group.

C. A new study that further collaborates an earlier finding that the consumption of some types of
berries lower the risk of heart failure.

D. The news media highlighting the fact that a study had clearly proven that effects of certain drugs on
rats need not be the same as on humans.

E. People not knowing what to believe because one study shows that a certain disease is caused by
genetic factors, while another shows that it also caused by lifestyle factors.

205
Q10. John: A study has identified that many soldiers received less-than-honorable discharges from the
army due to charges of misconduct that can actually be attributed to conditions such as PTSD and
traumatic brain injury. This is a military practice that is unfair and must be avoided. The army must take
responsibility for the same and stop discharging their soldiers dishonorably.

Christy: While it is true that it is unfair to the soldiers to be discharged so from the army, it has also been
found that there are no sufficient measures or systems that have been created to identify whether the
misconduct was due to trauma or due to other reasons. Moreover, to enforce discipline in the ranks, it is
important for the army to follow a uniform rule for all the soldiers.

Which of the following statements can Christy further add to her argument?

A. If there is no proper system, then the army should have developed a proper system by now.

B. Complete elimination of dishonorable discharges would make it more difficult for the army to
monitor and evaluate the soldiers.

C. It is not the army’s fault that there has been no system developed to help those with PTSD

D. Studies have also found that showing consideration for some soldiers has led to others taking
advantage, indirectly promoting misconduct.

E. There have been instances when the army has been able to identify the reasons for the misconduct
and acted accordingly.

Q11. Technological improvements and reduced equipment costs have made converting solar energy
directly into electricity far more cost-efficient in the last decade. However, the threshold of economic
viability for solar power (that is, the price per barrel to which oil would have to rise in order for new solar
power plants to be more economical than new oil-fired power plants) is unchanged at thirty-five dollars.

Which of the following, if true, does most to help explain why the increased cost-efficiency of solar
power has not decreased its threshold of economic viability?

(A) The cost of oil has fallen dramatically.

(B) The reduction in the cost of solar-power equipment has occurred despite increased raw material costs
for that equipment.

(C) Technological changes have increased the efficiency of oil-fired power plants.

(D) Most electricity is generated by coal-fired or nuclear, rather than oil-fired, power plants.

(E) When the price of oil increases, reserves of oil not previously worth exploiting become economically
viable.

Q12. A recent archeological expedition in Northern Asia revealed a great number of skeletons of animals that died
about 1000 years ago. Further research indicated that all the skeletons had been subjected to temperatures in
excess of 300 degrees Celsius. This fact provided grounds for speculation that they were killed and cooked by tribes
that lived in Northern Asia at that time. However, some of the skeletons belonged to animals that were considered
sacred by those tribes and were never hunted or eaten.

206
Which of the following best explains the apparent discrepancy above?

A) Some of the skeletons found during the expedition belonged to animals that no longer inhabit the area.

B) Skeletons of most animals did not have damages typical of the skeletons of animals that had been killed,
cut, and cooked.

C) Tribes that inhabited Northern Asia 1000 years ago used fire to cook food.

D) Another study indicated that there was a great famine in Northern Asia at that time, which forced local
tribes to look for alternative sources of food.

E) A further study indicated evidence of a large fire that had occurred in Northern Asia approximately 1000
years ago.

Q13. Humidity refers to the presence of water vapour in the atmosphere around us. It is surprising that the higher
the humidity at a place, the more the people sweat.

Which of the following if true, can be a logical explanation for the above phenomenon?

A. A higher temperature evaporates the water vapours in the atmosphere

B. People sweat only when they exert themselves physically

C. Presence of water vapour in the atmosphere is always negligible

D. People sweat even when the humidity is low

E. The pressure of water vapour at a saturation level in the atmosphere prevents sweat from human
bodies from evaporating

Q14. Only a member of the progressive party would oppose the bill to stop with all the testing of nuclear bombs.
Maria cannot be a member of the progressive party as she supports the bill.

Which of the following would show that the conclusion is not validly drawn?

A. All the parties other than the progressive party support the bill

B. The progressive party generally opposes all efforts at war preparation

C. Maria has, on an earlier occasion supported a bill introduced by a member belonging to a different party

D. It is possible that some members of the progressive party may not oppose the bill to stop testing of
nuclear bombs

E. Maria is not known to take a view independent of her party’s on critical issues

Q15. Despite the efforts of a small minority of graduate students at one university to unionize, the majority of
graduate students there remain unaware of the attempt. Most of those who are aware believe that a union would
not represent their interests or that, if it did, it would not effectively pursue them. Thus, the graduate students at
the university should not unionize, since the majority of them obviously disapprove of the attempt.

The reasoning in the argument is most vulnerable to criticism on the grounds that the argument:
207
A. tries to establish a conclusion simply on the premise that the conclusion agrees with long-standing
practice.

B. fails to exclude alternative explanations for why some graduate students disapprove of unionising

C. presumes that simply because a majority of a population is unaware of something, it must not be a good
idea

D. Ignores the possibility that although a union might not effectively pursue graduate student interests,
there are other reasons for unionizing

E. blurs the distinction between active disapproval and mere lack of approval

208

You might also like